Week 1 Prequiz - Growth and Development; Hospitalized Child

¡Supera tus tareas y exámenes ahora con Quizwiz!

Parents ask the nurse, "Should we be concerned our preschooler has an imaginary friend, and how should we react?" Which responses should the nurse give to the parents? (Select all that apply.) a. "The imaginary playmate is a sign of health." b. "You can acknowledge the presence of the imaginary companion." c. "It is normal for a preschool-aged child to have an imaginary friend." d. "If your child wants a place setting at the table for the child, it is best to refuse." e. "It is OK to allow the child to blame the imaginary playmate to avoid punishment."

ANS: A - "The imaginary playmate is a sign of health." B - "You can acknowledge the presence of the imaginary companion." C - "It is normal for a preschool-aged child to have an imaginary friend." Parents should be reassured that the child's fantasy is a sign of health that helps differentiate between make-believe and reality. Parents can acknowledge the presence of the imaginary companion by calling him or her by name and even agreeing to simple requests such as setting an extra place at the table, but they should not allow the child to use the playmate to avoid punishment or responsibility.

What factors can negatively affect parents' reactions to their child's illness? (Select all that apply.) a. Additional stresses b. Previous coping abilities c. Lack of support systems d. Seriousness of the threat to the child e. Previous experience with hospitalization

ANS: A - Additional stresses C - Lack of support systems D - Seriousness of the threat to the child The factors that can negatively affect parents' reactions to their child's illness are additional stresses, lack of support systems, and the seriousness of the threat to the child. Previous coping abilities and previous experience with hospitalization would have a positive effect on coping.

The nurse is teaching parents of a school-age child how to cleanse small wounds. What should the nurse advise the parents to avoid using to cleanse a wound? (Select all that apply.) a. Alcohol b. Normal saline c. Tepid water d. Povidone-iodine e. Hydrogen peroxide

ANS: A - Alcohol D - Povidone-iodine E - Hydrogen peroxide Caution caregivers to avoid cleansing the wound with povidone-iodine, alcohol, and hydrogen peroxide because these products disrupt wound healing. Normal saline and tepid water are safe to use when cleansing wounds.

A parent tells the nurse, "My toddler tries to undo the car seat harness and climb out of the seat." What strategies should the nurse recommend to the parent to encourage the child to stay in the seat? (Select all that apply.) a. Allow your child to hold a favorite toy b. Allow your child out of the seat occasionally. c. Avoid using rewards to encourage cooperative behavior. d. When child tries to unbuckle the seat harness, firmly say, "No." e. It may be necessary to stop the car to reinforce the expected behavior.

ANS: A - Allow your child to hold a favorite toy D - When child tries to unbuckle the seat harness, firmly say, "No." E - It may be necessary to stop the car to reinforce the expected behavior. Strategies to encourage a child to stay in a car seat include allowing the child to hold favorite toy, firmly saying "No" if the child begins to undo the harness, and stopping the car to reinforce the expected behavior. Rewards, such as stars or stickers, can be used to encourage cooperative behavior. The child should stay in the car seat at all times, even for short trips.

What menstrual disorders are indications for a pelvic examination? (Select all that apply.) a. Amenorrhea b. Dyspareunia c. Impaired fertility d. Irregular uterine or vaginal bleeding e. Dysmenorrhea unresponsive to therapy

ANS: A - Amenorrhea D - Irregular uterine or vaginal bleeding E - Dysmenorrhea unresponsive to therapy Indications for a pelvic examination include amenorrhea, irregular uterine or vaginal bleeding, and dysmenorrhea unresponsive to therapy. Impaired fertility is not an indication for a pelvic examination; it can be a result of endometriosis. Dyspareunia (painful intercourse) is not an indication for a pelvic examination but may be a sign of endometriosis.

A school-age child has been a victim of bullying. What characteristics does the nurse assess for in this child? (Select all that apply.) a. Anxiety b. Outgoing c. Low self-esteem d. Psychosomatic complaints e. Good academic performance

ANS: A - Anxiety C - Low self-esteem D - Psychosomatic complaints Victims of bullying are at increased risk for low self-esteem; anxiety; depression; feelings of insecurity and loneliness; poor academic performance; and psychosomatic complaints such as feeling tense, tired, or dizzy.

What are signs and symptoms of the stage of detachment in relation to separation anxiety in young children? (Select all that apply.) a. Appears happy b. Lacks interest in the environment c. Regresses to an earlier behavior d. Forms new but superficial relationships e. Interacts with strangers or familiar caregivers

ANS: A - Appears happy D - Forms new but superficial relationships E - Interacts with strangers or familiar caregivers Manifestations of the stage of detachment seen in children during a hospitalization may include appearing happy, forming new but superficial relationships, and interacting with strangers or familiar caregivers. Lacking interest in the environment and regressing to an earlier behavior are manifestations seen in the stage of despair.

Parents tell the nurse that siblings of their hospitalized child are feeling "left out." What suggestions should the nurse make to the parents to assist the siblings to adjust to the hospitalization of their brother or sister? (Select all that apply.) a. Arrange for visits to the hospital. b. Limit information given to the siblings. c. Encourage phone calls to the hospitalized child. d. Make or buy inexpensive toys or trinkets for the siblings. e. Identify an extended family member to be their support system.

ANS: A - Arrange for visits to the hospital. C - Encourage phone calls to the hospitalized child. D - Make or buy inexpensive toys or trinkets for the siblings. E - Identify an extended family member to be their support system. Strategies to support siblings during hospitalization include arranging for visits, encouraging phone calls, giving inexpensive gifts, and identifying a support person. Information should be shared with the siblings not limited.

The nurse is teaching parents of a 3-year-old child about language developmental milestones. What milestones should the nurse include in the teaching session? (Select all that apply.) a. Asks many questions b. Names one or more colors c. Repeats sentence of six syllables d. Uses primarily "telegraphic" speech e. Has a vocabulary of 1500 words or more

ANS: A - Asks many questions C - Repeats sentence of six syllables D - Uses primarily "telegraphic" speech The language milestones of a 3-year-old child include asking many questions, repeating a sentence of six syllables, and using primarily "telegraphic" speech. Naming one or more colors and having a vocabulary of 1500 words or more footing are language milestones seen at the age of 4 years.

The nurse is planning strategies to assist a slow-to-warm child to try new experiences. What strategies should the nurse plan? (Select all that apply.) a. Attend after-school activities with a friend. b. Suggest the child move quickly into a new situation. c. Avoid trying new experiences until the child is ready. d. Allow the child to adapt to the experience at his or her own pace. e. Contract for permission to withdraw after a trial of the experience.

ANS: A - Attend after-school activities with a friend. D - Allow the child to adapt to the experience at his or her own pace. E - Contract for permission to withdraw after a trial of the experience. The nurse should encourage slow-to-warm children to try new experiences but allow them to adapt to their surroundings at their own speed. Pressure to move quickly into new situations only strengthens their tendency to withdraw. After-school activities can be a cause for reaction, but attending with a friend or contracting for permission to withdraw after a trial of a specified number of times may provide them with sufficient incentive to try

What are common causes of speech problems? (Select all that apply.) a. Autism b. Prematurity c. Hearing loss d. Developmental delay e. Overstimulated environment

ANS: A - Autism C - Hearing loss D - Developmental delay Common causes of speech problems are hearing loss, developmental delay, autism, lack of environmental stimulation, and physical conditions that impede normal speech production. Prematurity and an overstimulated environment are not causes of speech problems.

The nurse is planning care for an infant with eczema. Which interventions should the nurse include in the care plan? (Select all that apply.) a. Avoid giving the infant a bubble bath. b. Avoid the use of a humidifier in the infant's room. c. Avoid overdressing the infant. d. Avoid the use of topical steroids on the infant's skin. e. Avoid wet compresses on the infant's most affected areas.

ANS: A - Avoid giving the infant a bubble bath. C - Avoid overdressing the infant. Guidelines for care of an infant with eczema include avoiding a bubble bath and harsh soaps and avoiding overdressing the infant to prevent perspiration, which can cause a flare-up. The care plan should include using a humidifier in the infant's room, topical steroids, and wet compresses on the most affected areas.

What preventive measures should the nurse teach parents of toddlers to prevent early childhood caries? (Select all that apply.) a. Avoid using a bottle as a pacifier. b. Eliminate bedtime bottles completely. c. Place juice in a bottle for the child to drink. d. Wean from the bottle by 18 months of age. e. Avoid coating pacifiers in a sweet substance.

ANS: A - Avoid using a bottle as a pacifier. B - Eliminate bedtime bottles completely E - Avoid coating pacifiers in a sweet substance. Prevention of dental caries involves eliminating the bedtime bottle completely, feeding the last bottle before bedtime, substituting a bottle of water for milk or juice, not using the bottle as a pacifier, and never coating pacifiers in sweet substances. Juice in bottles, especially commercially available ready-to-use bottles, is discouraged; these beverages are especially damaging because the sugar is more readily converted to acid. Juice should always be offered in a cup to avoid prolonging the bottle-feeding habit. Toddlers should be encouraged to drink from a cup at the first birthday and weaned from a bottle by 14 months of age, not 18 months.

The nurse is teaching parents of preschool-aged children strategies to prevent sexual abuse. What should the nurse include in the teaching session? (Select all that apply.) a. Back up a child's right to say no. b. Don't take what your child says too seriously. c. Take a second look at signals of potential danger. d. Don't be too detailed about examples of sexual assault. e. Remind children that even "nice" people sometimes do mean things.

ANS: A - Back up a child's right to say no. C - Take a second look at signals of potential danger. E - Remind children that even "nice" people sometimes do mean things. To provide protection and preparation from sexual abuse, parents should back up a child's right to say no, take a second look at signals of potential danger, and remind children that even "nice" people sometimes do mean things. Parents should take what children say seriously and they should give specific definitions and examples of sexual assault

What are classified as corrosive poisons? (Select all that apply.) a. Batteries b. Paint thinner c. Drain cleaners d. Mineral seed oil e. Mildew remover

ANS: A - Batteries C - Drain cleaners E - Mildew remover Batteries, drain cleaners, and mildew removers are classified as corrosive poisons. Paint thinner and mineral seed oil are classified as hydrocarbon poisons.

The nurse is preparing to feed a 10-month-old child diagnosed with failure to thrive (FTT). Which actions should the nurse plan to implement? (Select all that apply.) a. Be persistent. b. Introduce new foods slowly. c. Provide a stimulating atmosphere. d. Maintain a calm, even temperament. e. Feed the infant only when signs of hunger are exhibited.

ANS: A - Be persistent. B - Introduce new foods slowly. D - Maintain a calm, even temperament. Feeding strategies for children with FTT should include persistence; introducing new foods slowly; and maintaining a calm, even temperament. The environment should be unstimulating, and a structured routine should be developed with regard to feeding, not just when the infant shows signs of hunger

The nurse is teaching an adolescent about the use of tretinoin (Retin-A). What should the nurse include in the teaching session? (Select all that apply.) a. Begin with a pea-sized dot of medication. b. Apply additional medication to the throat. c. Use sunscreen daily and avoid the sun when possible. d. Divide the medication into the three main areas of the face. e. Apply the medication immediately after washing the face.

ANS: A - Begin with a pea-sized dot of medication. C - Use sunscreen daily and avoid the sun when possible. D - Divide the medication into the three main areas of the face. Tretinoin is available as a cream, gel, or liquid. This drug can be extremely irritating to the skin and requires careful patient education for optimal usage. The patient should be instructed to begin with a pea-sized dot of medication, which is divided into the three main areas of the face and then gently rubbed into each area. The avoidance of the sun and the daily use of sunscreen must be emphasized because sun exposure can result in severe sunburn. The medication should not be applied for at least 20 to 30 minutes after washing to decrease the burning sensation. The medication should not be applied to the throat.

The nurse is teaching parents of preschool children consequences of inadequate sleep. What should the nurse include in the teaching session? (Select all that apply.) a. Behavior changes b. Increased appetite c. Difficulty concentrating d. Poor control of emotions e. Impaired learning ability

ANS: A - Behavior changes C - Difficulty concentrating D - Poor control of emotions E - Impaired learning ability Consequences of inadequate sleep include daytime tiredness, behavior changes, hyperactivity, difficulty concentrating, impaired learning ability, poor control of emotions and impulses, and strain on family relationships. Increased appetite is not a consequence of inadequate sleep

What are characteristics of middle adolescence (15-17 years) with regard to relationships with peers? (Select all that apply.) a. Behavioral standards set by peer group b. Acceptance of peers extremely important c. Seeks peer affiliations to counter instability d. Exploration of ability to attract opposite sex e. Peer group recedes in importance in favor of individual friendship

ANS: A - Behavioral standards set by peer group B - Acceptance of peers extremely important D - Exploration of ability to attract opposite sex Characteristics of middle adolescence relationships with peers include behavioral standards set by the peer group, acceptance of peers is extremely important, and exploration of the ability to attract opposite sex. Seeking peer affiliations to counter instability is a characteristic of early adolescence relationships with peers. Peer groups receding in importance in favor of individual friendships is characteristic of late adolescence relationships with peers

The clinic nurse is assessing an adolescent on a topical antibacterial agent. The nurse should assess for which side effects that can be seen with topical antibacterial agents? (Select all that apply.) a. Burning b. Dryness c. Dry eyes d. Erythema e. Nasal irritation

ANS: A - Burning B - Dryness D - Erythema Side effects of topical antibacterial medications include erythema, dryness, and burning; using the medications every other day will decrease the adverse effects. Dry eyes and nasal irritation are seen with use of isotretinoin, 13-cis-retinoic acid (Accutane).

What growth and development milestones are expected between the ages of 8 and 9 years? (Select all that apply.) a. Can help with routine household tasks b. Likes the reward system for accomplished tasks c. Uses the telephone for practical purposes d. Chooses friends more selectively e. Goes about home and community freely, alone or with friends f. Enjoys family time and is respectful of parents

ANS: A - Can help with routine household tasks B - Likes the reward system for accomplished tasks E - Goes about home and community freely, alone or with friends Children between the age of 8 and 9 years accomplish many growth and development milestones, including helping with routine household tasks, liking the reward system when a task is accomplished well, and going out with friends or alone more independently and freely. Using the telephone for practical reasons, choosing friends more selectively, and finding enjoyment in family with new-found respect for parents are tasks accomplished between the ages of 10 and 12 years.

Parents of an adolescent ask the school nurse, "It is OK for our adolescent to get a job?" The nurse should answer telling the parents the effects of adolescents who work more than 20 hours a week are what? (Select all that apply.) a. Can lead to fatigue b. Can lead to poorer grades c. Improves an interest in school d. Enhances development and identity e. Can reduce extracurricular involvement

ANS: A - Can lead to fatigue B - Can lead to poorer grades E - Can reduce extracurricular involvement Detrimental effects are likely for adolescents who work more than 20 hours a week. Greater involvement in work can lead to fatigue, decreased interest in school, reduced extracurricular involvement, and poorer grades. Involvement in work may take time away from other activities that could contribute to identity development. Adolescent work as it exists today may negatively affect development.

The nurse is teaching a school-age child about factors that can delay wound healing. What factors should the nurse include in the teaching session? (Select all that apply.) a. Deficient vitamin C b. Deficient vitamin D c. Increased circulation d. Dry wound environment e. Increase in white blood cells

ANS: A - Deficient vitamin C B - Deficient vitamin D D - Dry wound environment Factors that delay wound healing are a dry wound environment (allows epithelial cells to dry), deficient vitamin C (inhibits formation of collagen fibers), and deficient vitamin D (regulates growth and differentiation of cell types). Decreased, not increased, circulation delays healing. An increase in the white blood cell count may occur but does not delay healing

Parents are worried that their preschool-aged child is showing hyperaggressive behavior. What are signs of hyperaggresive behavior? (Select all that apply.) a. Disrespect b. Noncompliance c. Infrequent impulsivity d. Occasional temper tantrums e. Unprovoked physical attacks on other children

ANS: A - Disrespect B - Noncompliance E - Unprovoked physical attacks on other children Hyperaggressive behavior in preschoolers is characterized by unprovoked physical attacks on other children and adults, destruction of others' property, frequent intense temper tantrums, extreme impulsivity, disrespect, and noncompliance.

The nurse is providing anticipatory guidance to the parents of a 1-month-old infant on preventing a suffocation injury. Which should the nurse include in the teaching? (Select all that apply.) a. Do not place pillows in the infant's crib. b. Crib slats should be 4 inches or less apart. c. Keep all plastic bags stored out of the infant's reach. d. Plastic over the mattress is acceptable if it is covered with a sheet. e. A pacifier should not be tied on a string around the infant's neck.

ANS: A - Do not place pillows in the infant's crib. C - Keep all plastic bags stored out of the infant's reach. E - A pacifier should not be tied on a string around the infant's neck. Anticipatory guidance for a 1-month-old infant to prevent a suffocation injury takes into account that the infant will have increased eye-hand coordination and a voluntary grasp reflex as well as a crawling reflex that may propel the infant forward or backward. Pillows should not be placed in the infant's crib, plastic bags should be kept out of reach, and a pacifier should not be tied on a string around the neck. Crib slats should be 2.4 inches apart (4 inches is too wide), and the mattress should not be covered with plastic even if a sheet is used to cover it.

What can the nurse suggest to families to reduce blood lead levels? (Select all that apply.) a. Do not store food in open cans. b. Ensure the child eats regular meals. c. Mix formula with hot water from the tap. d. Vacuum hard-surfaced floors and window wells. e. Wash and dry the child's hands and face frequently.

ANS: A - Do not store food in open cans. B - Ensure the child eats regular meals. E - Wash and dry the child's hands and face frequently. To reduce blood lead levels, the family should ensure the child eats regular meals because more lead is absorbed on an empty stomach. The child's hands and face should be washed and dried frequently, especially before eating. Food should not be stored in open cans, particularly if cans are imported. Hot water dissolves lead more quickly than cold water and thus contains higher levels of lead. Hot water should not be used to mix formula. Hard-surfaced floors or window sills or wells should not be vacuumed because this spreads dust

The nurse is preparing a staff education program about growth and development of an 18-month-old toddler. Which characteristics should the nurse include in the staff education program? (Select all that apply.) a. Eats well with a spoon and cup b. Runs clumsily and can walk up stairs c. Points to common objects d. Builds a tower of three or four blocks e. Has a vocabulary of 300 words f. Dresses self in simple clothes

ANS: A - Eats well with a spoon and cup B - Runs clumsily and can walk up stairs C - Points to common objects D - Builds a tower of three or four blocks Tasks accomplished by an 18-month-old toddler include eating well with a spoon and cup, running clumsily, walking up stairs, pointing to common objects such as shoes, and building a tower with three or four blocks. An 18-month-old toddler has a vocabulary of only 10 words, not 300. Toddlers cannot dress themselves in simple clothing until 24 months of age

The nurse is teaching an adolescent girl strategies to relieve dysmenorrhea. What should the nurse include in the teaching session? (Select all that apply.) a. Effleurage b. Diet high in fat c. Limiting exercise d. Use of a heating pad e. Massaging the lower back

ANS: A - Effleurage D - Use of a heating pad E - Massaging the lower back Dysmenorrhea can be relieved by heat (heating pad or hot bath), which minimizes cramping by increasing vasodilation and muscle relaxation and minimizing uterine ischemia. Also, massaging the lower back can reduce pain by relaxing paravertebral muscles and increasing the pelvic blood supply. Soft, rhythmic rubbing of the abdomen (effleurage) is useful because it provides a distraction and an alternative focal point. A low-fat, not a high-fat, diet can help with dysmenorrhea, and exercise should not be limited because exercise can be beneficial.

What guidelines should the nurse use when interviewing adolescents? (Select all that apply.) a. Ensure privacy. b. Use open-ended questions. c. Share your thoughts and assumptions. d. Explain that all interactions will be confidential. e. Begin with less sensitive issues and proceed to more sensitive ones.

ANS: A - Ensure privacy. B - Use open-ended questions. E - Begin with less sensitive issues and proceed to more sensitive ones. Guidelines for interviewing adolescents include ensuring privacy, using open-ended questions, and beginning with less sensitive issues and proceeding to more sensitive ones. The nurse should not share thoughts but maintain objectivity and should avoid assumptions, judgments, and lectures. It may not be possible for all interactions to be confidential. Limits of confidentiality include a legal duty to report physical or sexual abuse and to get others involved if an adolescent is suicidal

The nurse is providing anticipatory guidance to parents of an 8-month-old infant on preventing a drowning injury. Which should the nurse include in the teaching? (Select all that apply.) a. Fence swimming pools. b. Keep bathroom doors open. c. Eliminate unnecessary pools of water. d. Keep one hand on the child while in the tub. e. Supervise the child when near any source of water.

ANS: A - Fence swimming pools C - Eliminate unnecessary pools of water. D - Keep one hand on the child while in the tub. E - Supervise the child when near any source of water. Anticipatory guidance to prevent drowning for an 8-month-old infant takes into account that the child will begin to crawl, cruise around furniture, walk, and climb. Fences should be placed around swimming pools, unnecessary pools of water should be eliminated, one hand should be kept on the child when bathing, and the child should be supervised when near any source of water. The bathroom doors should be kept closed.

The nurse is teaching an adolescent female about the symptoms of premenstrual syndrome (PMS). What symptoms should the nurse include in the teaching session? (Select all that apply.) a. Headaches b. Fluid retention c. Increased energy d. Emotional changes e. Premenstrual cravings

ANS: A - Headaches B - Fluid retention D - Emotional changes E - Premenstrual cravings Symptoms of PMS include fluid retention (abdominal bloating, pelvic fullness, edema of the lower extremities, breast tenderness, and weight gain), behavioral or emotional changes (depression, crying spells, irritability, panic attacks, and impaired ability to concentrate), premenstrual cravings (sweets, salt, increased appetite, and food binges), headache, and backache. Fatigue rather than increased energy occurs

An adolescent asks the nurse about the "safety of getting a tattoo." The nurse explains to the adolescent that it is important to find a qualified operator using proper sterile technique because an unsterilized needle or contaminated tattoo ink can cause what? (Select all that apply.) a. Hepatitis C virus b. Hepatitis B virus c. Hepatitis E virus d. Human immunodeficiency virus (HIV) e. Mycobacterium chelonae skin infections

ANS: A - Hepatitis C virus B - Hepatitis B virus D - Human immunodeficiency virus (HIV) E - Mycobacterium chelonae skin infections Using the same unsterilized needle to tattoo body parts of multiple teenagers presents the same risk for human immunodeficiency virus (HIV), hepatitis C virus, and hepatitis B virus transmission as occurs with other needle-sharing activities. Contaminated tattoo ink can cause nontuberculous M. chelonae skin infections. The hepatitis E virus is transmitted via the fecal-oral route, principally via contaminated water, not by contaminated needles.

The nurse is evaluating a 7-month-old infant's cognitive development. Which behaviors should the nurse anticipate evaluating? (Select all that apply.) a. Imitates sounds b. Shows interest in a mirror image c. Comprehends simple commands d. Actively searches for a hidden object e. Attracts attention by methods other than crying

ANS: A - Imitates sounds B - Shows interest in a mirror image E - Attracts attention by methods other than crying A 7-month-old infant is in the secondary circular reactions (4-8 months) stage of cognitive development. Behaviors in this stage include imitating sounds, showing interest in a mirror image, and attracting attention by methods other than crying. Comprehending simple commands and actively searching for a hidden object are behaviors seen in the coordination of secondary schemas (9-12 months).

The nurse is preparing to administer danazol (Danocrine) to a patient with endometriosis. What are the side effects of this medication? (Select all that apply.) a. Insomnia b. Hot flashes c. Amenorrhea d. Increased libido e. Vaginal secretions

ANS: A - Insomnia B - Hot flashes C - Amenorrhea The side effects of danazol are amenorrhea, hot flashes, vaginal dryness, insomnia, and decreased libido.

The nurse is providing anticipatory guidance to parents of a 4-month-old infant on preventing an aspiration injury. What should the nurse include in the teaching? (Select all that apply.) a. Keep baby powder out of reach. b. Inspect toys for removable parts. c. Allow the infant to take a bottle to bed. d. Teething biscuits can be used for teething discomfort. e. The infant should not be fed hard candy, nuts, or foods with pits.

ANS: A - Keep baby powder out of reach. B - Inspect toys for removable parts. E - The infant should not be fed hard candy, nuts, or foods with pits. Anticipatory guidance to prevent aspiration for a 4-month-old infant takes into account that the infant will begin to be more active and place objects in the mouth. Toys should be checked for removable parts; baby powder should be kept out of reach; and hard candy, nuts, and foods with pits should be avoided. The infant should not go to bed with a bottle. Teething biscuits should be used with caution because large chunks may be broken off and aspirated

The nurse is teaching an adolescent with premenstrual syndrome (PMS) dietary measures to relieve the symptoms of PMS. What should the nurse include in the teaching session? (Select all that apply.) a. Limit salt in the diet. b. Limit legumes in the diet. c. Include red meat in the diet. d. Include whole grains in the diet. e. Limit consumption of refined sugar.

ANS: A - Limit salt in the diet. D - Include whole grains in the diet. E - Limit consumption of refined sugar. Dietary treatment for PMS includes limiting consumption of refined sugar, salt, red meat, alcohol, and caffeinated beverages. Women can be encouraged to include whole grains, legumes, seeds, nuts, vegetables, fruits, and vegetable oils in their diet.

The nurse has administered a dose of epinephrine to a 12-month-old infant. For which adverse reactions of epinephrine should the nurse monitor? (Select all that apply.) a. Nausea b. Tremors c. Irritability d. Bradycardia e. Hypotension

ANS: A - Nausea B - Tremors C - Irritability Epinephrine increases activation of the sympathetic nervous system. Adverse effects include nausea, tremors, and irritability. Tachycardia would occur, not bradycardia, and hypertension, not hypotension, would occur

A parent asks the nurse about the "characteristics of a nightmare." What response should the nurse give to the parent? (Select all that apply.) a. Nightmares are scary dreams. b. The child can describe the nightmare. c. The child is reassured by your presence. d. Nightmares occur usually 1 to 4 hours after falling asleep. e. Nightmares take place during non-rapid eye movement sleep

ANS: A - Nightmares are scary dreams. B - The child can describe the nightmare. C - The child is reassured by your presence. Nightmares are scary dreams, the child can describe the nightmare, and the child is reassured by a parent's presence. Sleep terrors occur usually 1 to 4 hours after falling asleep, but nightmares occur in the second half of sleep. Sleep terrors occur during non-rapid eye movement sleep, but nightmares occur during rapid eye movement sleep.

The school nurse is assessing a child's severely scraped knee for infection. What are signs of a wound infection? (Select all that apply.) a. Odor b. Edema c. Dry scab d. Purulent exudate e. Decreased temperature

ANS: A - Odor B - Edema D - Purulent exudate Signs of wound infection are odor, edema, and purulent exudate. Increased, not decreased, temperature indicates infection. A dry scab over the wound is part of the healing process.

The nurse is teaching parents about potential causes of colic in infancy. Which should the nurse include in the teaching session? (Select all that apply.) a. Overeating b. Understimulation c. Frequent burping d. Parental smoking e. Swallowing excessive air

ANS: A - Overeating D - Parental smoking E - Swallowing excessive air Potential causes of colic include too rapid feeding, overeating, swallowing excessive air, improper feeding technique (especially in positioning and burping), emotional stress or tension between the parent and child, parental smoking, and overstimulation

The parents tell a nurse "our child is having some short-term negative outcomes since the hospitalization." The nurse recognizes that what can negatively affect short-term negative outcomes? (Select all that apply.) a. Parents' anxiety b. Consistent nurses c. Number of visitors d. Length of hospitalization e. Multiple invasive procedures

ANS: A - Parents' anxiety D - Length of hospitalization E - Multiple invasive procedures The stressors of hospitalization may cause young children to experience short- and long-term negative outcomes. Adverse outcomes may be related to the length and number of admissions, multiple invasive procedures, and the parents' anxiety. Consistent nurses would have a positive effect on short-term negative outcomes. The number of visitors does not have an effect on negative outcomes.

The nurse is teaching an adolescent female with primary dysmenorrhea foods that are natural diuretics. What foods should the nurse include in the teaching plan? (Select all that apply.) a. Peaches b. Asparagus c. Watermelon d. Wheat bread e. Dairy products

ANS: A - Peaches B - Asparagus C - Watermelon Natural diuretics such as asparagus, cranberry juice, peaches, parsley, or watermelon may help reduce edema and related discomforts of primary dysmenorrhea. Wheat bread and dairy products are not natural diuretics.

The nurse is teaching parents about foods that are hyperallergenic. Which foods should the nurse include? (Select all that apply.) a. Peanuts b. Bananas c. Potatoes d. Egg noodles e. Tomato juice

ANS: A - Peanuts D - Egg noodles E - Tomato juice Hyperallergenic foods include peanuts, egg noodles, and tomato juice. Bananas and potatoes are not hyperallergenic

What are the goals of organized athletics for preadolescent children? (Select all that apply.) a. Physical fitness b. Basic motor skills c. A positive self-image d. Commitment to winning

ANS: A - Physical fitness B - Basic motor skills C - A positive self-image The goals of organized athletics for preadolescent children include physical fitness, basic motor skills, and a positive self-image. The commitment is to the values of teamwork, fair play, and sportsmanship, not to winning.

The nurse is caring for a child with neurofibromatosis. What local manifestations does the nurse expect to assess in this child? (Select all that apply.) a. Pigmented nevi b. Axillary freckling c. Café-au-lait spots d. Slowly growing cutaneous neurofibromas e. Wheals that spread irregularly and fade within a few hours

ANS: A - Pigmented nevi B - Axillary freckling C - Café-au-lait spots D - Slowly growing cutaneous neurofibromas Local manifestations of neurofibromatosis include pigmented nevi, axillary freckling, café-au-lait spots, and slowly growing cutaneous neurofibromas. Wheals that spread irregularly and fade within a few hours are characteristic of urticaria.

The nurse is teaching parents of a toddler how to handle temper tantrums. What should the nurse include in the teaching? (Select all that apply.) a. Provide realistic expectations. b. Avoid using rewards for good behavior. c. Ensure consistency among all caregivers in expectations. d. During tantrums, ignore the behavior and continue to be present. e. Use time-outs for managing temper tantrums, starting at 12 months.

ANS: A - Provide realistic expectations. C - Ensure consistency among all caregivers in expectations. D - During tantrums, ignore the behavior and continue to be present. The best approach toward tapering temper tantrums requires consistency and developmentally appropriate expectations and rewards. Ensuring consistency among all caregivers in expectations, prioritizing what rules are important, and developing consequences that are reasonable for the child's level of development help manage the behavior. During tantrums, ignore the behavior, provided the behavior is not injurious to the child, such as violently banging the head on the floor. Continue to be present to provide a feeling of control and security to the child after the tantrum has subsided. Starting at 18 months, time-outs work well for managing temper tantrums, but not at 12 months.

The nurse relates to parents that there are some beneficial effects of hospitalization for their child. What are beneficial effects of hospitalization? (Select all that apply.) a. Recovery from illness b. Improve coping abilities c. Opportunity to master stress d. Provide a break from school e. Provide new socialization experiences

ANS: A - Recovery from illness B - Improve coping abilities C - Opportunity to master stress E - Provide new socialization experiences The most obvious benefit is the recovery from illness, but hospitalization also can present an opportunity for children to master stress and feel competent in their coping abilities. The hospital environment can provide children with new socialization experiences that can broaden their interpersonal relationships. Having a break from school is not a benefit of hospitalization.

The nurse is assessing a family's use of complementary medicine practices. What practices are classified as mind-body control therapies? (Select all that apply.) a. Relaxation b. Acupuncture c. Prayer therapy d. Guided imagery e. Herbal medicine

ANS: A - Relaxation C - Prayer therapy D - Herbal medicine Relaxation, prayer therapy, and guided imagery are classified as mind-body control therapies. Acupuncture and herbal medicine are classified as traditional and ethnomedicine therapies.

Parents are concerned about their child riding an all-terrain vehicle. What should the nurse tell the parents about safe use of all-terrain vehicles? (Select all that apply.) a. Restrict riding to familiar terrain. b. Limit street use to the neighborhood. c. Nighttime riding should not be allowed. d. Vehicles should not carry more than two persons. e. Vehicles should include seat belts, roll bars, and automatic headlights.

ANS: A - Restrict riding to familiar terrain. C -Nighttime riding should not be allowed. E - Vehicles should include seat belts, roll bars, and automatic headlights. Safe use of all-terrain vehicles includes restricting riding to familiar terrain; not allowing nighttime riding; and assuring the vehicle has seat belts, roll bars, and automatic headlights. Street use should not be allowed, and the vehicle should not carry more than one person.

The nurse is teaching parents of a 3-year-old child about gross motor developmental milestones. What milestones should the nurse include in the teaching session? (Select all that apply.) a. Rides a tricycle b. Catches a ball reliably c. Jumps off the bottom step d. Stands on one foot for a few seconds e. Walks downstairs using alternate footing

ANS: A - Rides a tricycle C - Jumps off the bottom step D - Stands on one foot for a few seconds The gross motor milestones of a 3-year-old child include riding a tricycle, jumping off the bottom step, and standing on one foot for a few seconds. Catching a ball reliably and walking downstairs using alternate footing are gross motor milestones seen at the age of 4 years.

What is the reason pedestrian motor vehicle injuries increase in the preschool age? (Select all that apply.) a. Riding tricycles b. Running after balls c. Playing in the street d. Crossing streets at the crosswalk e. Crossing streets with an adult

ANS: A - Riding tricycles B - Running after balls C - Playing in the street Pedestrian motor vehicle injuries increase because of activities such as playing in the street, riding tricycles, running after balls, and forgetting safety regulations when crossing streets. Crossing streets at the crosswalk or with an adult are safety measures.

The nurse is planning strategies to assist difficult or easily distracted children when they participate in activities. What strategies should the nurse plan? (Select all that apply.) a. Role-play before the activity. b. Handle behavior with firmness. c. Acquaint them with what to expect. d. Be patient with inappropriate behavior. e. Don't give them much information about the activity.

ANS: A - Role-play before the activity. B - Handle behavior with firmness. C - Acquaint them with what to expect. D - Don't give them much information about the activity. Difficult or easily distracted children may benefit from "practice" sessions in which they are prepared for a given event by role-playing, visiting the site, reading or listening to stories, or using other methods to acquaint them with what to expect. Nurses need to handle children with difficult temperaments with exceptional patience, firmness, and understanding so they can learn appropriate behavior in their interactions with others

What are symptoms of abusive head trauma (AHT) in the more severe form that may be present? (Select all that apply.) a. Seizures b. Posturing c. Tachypnea d. Tachycardia e. Altered level of consciousness

ANS: A - Seizures B - Posturing E - Altered level of consciousness In more severe forms, presenting symptoms of abusive head trauma may include seizures, posturing, alterations in level of consciousness, apnea, bradycardia, or death.

The school nurse recognizes that students who are targeted for repeated harassment and bullying may exhibit what? (Select all that apply.) a. Skip school b. Attempt suicide c. Bring weapons to school d. Attend extracurricular activities e. Report symptoms of depression

ANS: A - Skip school B - Attempt suicide C - Bring weapons to school E - Report symptoms of depression Students targeted for repeated teasing and harassment are more likely to skip school, to report symptoms of depression, and to attempt suicide. Equally troubling, teens who are regularly harassed or bullied are also more likely to bring weapons to school to feel safe. Students who are bullied do not want to attend extracurricular activities.

The nurse is planning play activities for a 2-month-old hospitalized infant to stimulate the auditory sense. Which activities should the nurse implement? (Select all that apply.) a. Talk to the infant. b. Play a music box. c. Place a squeaky doll in the crib. d. Give the infant a small-handled clear rattle.

ANS: A - Talk to the infant. B - Play a music box. D - Give the infant a small-handled clear rattle. Auditory stimulation appropriate for a 2-month-old infant includes talking to the infant, playing a music box, and giving the infant a small-handled clear rattle. Placing a squeaky doll in the crib is appropriate for an infant 6 months of age or older

A parent asks the nurse about the "characteristics of a sleep terror." What response should the nurse give to the parent? (Select all that apply.) a. The child screams during the sleep terror. b. Return to sleep is delayed because of persistent fear. c. The night terror occurs during the second half of night. d. The child has no memory of the dream with a sleep terror. e. The child is not aware of another's presence during a sleep terror.

ANS: A - The child screams during the sleep terror. D - The child has no memory of the dream with a sleep terror. E - The child is not aware of another's presence during a sleep terror. During sleep terrors, the child screams and has no memory of the dream. The child is not aware of another's presence during a sleep terror. Return to sleep is usually rapid with a sleep terror, but it is delayed with a nightmare. The sleep terror occurs usually within 1 to 4 hours of sleep, but nightmares occur during the second half of night

What conditions are physical complications of obesity? (Select all that apply.) a. Type 2 diabetes mellitus b. QT interval prolongation c. Fatty liver disease d. Gastrointestinal dysfunction e. Abnormal growth acceleration f. Dental erosion

ANS: A - Type 2 diabetes mellitus C - Fatty liver disease E - Abnormal growth acceleration Physical complications of obesity include type 2 diabetes mellitus, which is reaching epidemic proportions in children and adolescents; fatty liver disease not related to alcohol consumption; and abnormal growth acceleration in which overweight children tend to be taller and mature earlier than children who are not overweight. Prolonged QT intervals, gastrointestinal dysfunction, and dental erosion are physical complications observed in children or adolescents who have eating disorders such as anorexia nervosa or bulimia.

The school nurse is teaching a group of adolescent females which measures to take to prevent genital tract infections. What should the nurse include in the teaching session? (Select all that apply.) a. Use condoms. b. Douche once a week. c. Avoid tight-fitting clothing. d. Limit exposure to bubble baths. e. Avoid colored and scented toilet tissue.

ANS: A - Use condoms. C - Avoid tight-fitting clothing. D - Limit exposure to bubble baths. E - Avoid colored and scented toilet tissue. Measure to take to prevent genital tract infections include using condoms, avoiding tight-fitting clothing, limiting exposure to bubble baths, and avoiding colored and scented toilet tissue. Douching should be avoided

The nurse is teaching parents of a 4-year-old child about socialization developmental milestones. What milestones should the nurse include in the teaching session? (Select all that apply.) a. Very independent b. Has mood swings c. Has better manners d. Eager to do things right e. Tends to be selfish and impatient

ANS: A - Very independent B - Has mood swings E - Tends to be selfish and impatient The socialization milestones of a 4-year-old child include being very independent, having moods swings, and tending to be selfish and impatient. Having better manners and being eager to do things right are socialization milestones seen at the age of 5 years.

The nurse is preparing to administer some iron drops to a toddler. Which factor can increase iron absorption? (Select all that apply.) a. Vitamin A b. Acidity (low pH) c. Phosphates (milk) d. Malabsorptive disorders e. Ascorbic acid (Vitamin C)

ANS: A - Vitamin A B - Acidity (low pH) E - Ascorbic acid (Vitamin C) Factors that increase iron absorption are vitamin A, acidity (low pH), and ascorbic acid (vitamin C). Phosphates (milk) and malabsorptive disorders decrease absorption of iron.

What are signs and symptoms of the stage of despair in relation to separation anxiety in young children? (Select all that apply.) a. Withdrawn from others b. Uncommunicative c. Clings to parents d. Physically attacks strangers e. Forms new but superficial relationships f. Regresses to early behaviors

ANS: A - Withdrawn from others B -Uncommunicative F - Regresses to early behaviors Manifestations of the stage of despair seen in children during a hospitalization may include withdrawing from others, being uncommunicative, and regressing to earlier behaviors. Clinging to parents and physically attacking a stranger should be seen during the stage of protest, and forming new but superficial relationships is seen during the stage of detachment.

The community health nurse is reviewing risk factors for vitamin D deficiency. Which children are at high risk for vitamin D deficiency? (Select all that apply.) a. Children with fair pigmentation b. Children who are overweight or obese c. Children who are exclusively bottle fed d. Children with diets low in sources of vitamin D e. Children of families who use milk products not supplemented with vitamin D

ANS: B - Children who are overweight or obese D - Children with diets low in sources of vitamin D E - Children of families who use milk products not supplemented with vitamin D Populations at risk for vitamin D deficiency include overweight or obese children, children with diets low in sources of vitamin D, and children of families who use milk products not supplemented with vitamin D. Children with dark, not fair, pigmentation and children who are exclusively breast fed, not bottle fed, are also at risk.

What are sources of stress in preschoolers? (Select all that apply.) a. Shares possessions b. Damages or destroys objects c. May fear dogs or other animals d. Seems to be in perpetual motion e. May stutter or stumble over words

ANS: B - Damages or destroys objects C - May fear dogs or other animals D - Seems to be in perpetual motion E - May stutter or stumble over words Sources of stress in preschoolers include damaging or destroying objects, fearing dogs or other animals, in perpetual motion, and may stutter or stumble over words. Guarding possessions, not sharing, is a source of stress.

The school nurse recognizes that children respond to stress by using which tactics? (Select all that apply.) a. Passivity b. Delinquency c. Daydreaming d. Delaying tactics e. Becoming outgoing

ANS: B - Delinquency C - Daydreaming D - Delaying tactics Children respond to stress by using coping mechanisms that include internalizing symptoms such as withdrawal, delaying tactics, and daydreaming, along with externalizing symptoms such as aggression and delinquency.

Characteristics of bullies include what? (Select all that apply.) a. Female b. Depressed c. Good peer relationships d. Poor academic performance e. Exposed to domestic violence

ANS: B - Depressed D - Poor academic performance E - Exposed to domestic violence Children who are bullies are likely to be male, depressed, have poor academic performance, be exposed to domestic violence, have poor peer relationships, and have poor communication with their parents.

What are core principles of patient- and family-centered care? (Select all that apply.) a. Collaboration b. Empowering families c. Providing formal and informal support d. Maintaining strict policy and procedure routines e. Withholding information that is likely to cause anxiety

ANS: B - Empowering families C - Providing formal and informal support Core principles of patent- and family-centered care include collaboration, empowerment, and providing formal and informal support. There should be flexibility in policy and procedures, and communication should be complete, honest, and unbiased, not withheld.

The nurse is teaching parents strategies to manage their child's refusal to go to sleep. Which should the nurse include in the teaching session? (Select all that apply.) a. Keep bedtime early. b. Enforce consistent limits. c. Use a reward system with the child. d. Have a consistent before bedtime routine.

ANS: B - Enforce consistent limits. C - Use a reward system with the child. D - Have a consistent before bedtime routine. Strategies to manage a child's refusal to go to sleep include enforcement of consistent limits, using a reward system, and having a consistent before bedtime routine. An evaluation of whether the hour of sleep is too early should be considered because an early bedtime could cause the child to resist sleep if not tired

The nurse is caring for a child with erythema multiforme (Stevens-Johnson syndrome). What local manifestations does the nurse expect to assess in this child? (Select all that apply.) a. Papular urticaria b. Erythematous papular rash c. Lesions absent in the scalp d. Lesions enlarge by peripheral expansion e. Firm papules that may be capped by vesicles

ANS: B - Erythematous papular rash C - Lesions absent in the scalp D - Lesions enlarge by peripheral expansion Local manifestations of erythema multiforme include an erythematous popular rash, lesions involving most skin surfaces except the scalp and lesions that enlarge by peripheral expansion. Papular urticaria and firm papules capped by vesicles are characteristics of an insect bite.

The nurse is caring for a child with psoriasis. What local manifestations does the nurse expect to assess in this child? (Select all that apply.) a. Development of wheals b. First lesions appear in the scalp c. Round, thick, dry reddish patches d. Lesions appear in intergluteal folds e. Patches are covered with coarse, silvery scales

ANS: B - First lesions appear in the scalp C - Round, thick, dry reddish patches E - Patches are covered with coarse, silvery scales Local manifestations of psoriasis include lesions that appear in the scalp initially and round, thick dry patches covered with coarse, silvery scales. Development of wheals is seen in urticaria. Lesions in intergluteal folds are characteristic of intertrigo

What are characteristics of dating relationships in early adolescence? (Select all that apply.) a. One-on-one dating b. Follow ritualized "scripts" c. Are psychosocially intimate d. Involve playing stereotypic roles e. Participating in mixed-gender group activities

ANS: B - Follow ritualized "scripts" D - Involve playing stereotypic roles E - Participating in mixed-gender group activities Early dating relationships typically follow highly ritualized "scripts" in which adolescents are more likely to play stereotypic roles than to really be themselves. Participating in mixed-gender group activities, such as going to parties or other events, may have a positive impact on young teenagers' well-being. One-on-one dating during early adolescence, however, with a lot of time spent alone, may lead to sexual intimacy before a teen is ready. Although teenagers may begin dating during early adolescence, these early dating relationships are not usually psychosocially intimate.

What are classified as hydrocarbon poisons? (Select all that apply.) a. Bleach b. Gasoline c. Turpentine d. Lighter fluid e. Oven cleaners

ANS: B - Gasoline C - Turpentine D - Lighter fluid Gasoline, turpentine, and lighter fluid are classified as hydrocarbon poisons. Bleach and oven cleaners are classified as corrosive poisons.

A toddler is in the sensorimotor, tertiary circular reactions stage of cognitive development. What behavior should the nurse expect to assess? (Select all that apply.) a. Refers to self by pronoun b. Gestures "up" and "down" c. Able to insert round object into a hole d. Can find hidden objects but only in the first location e. Uses future-oriented words, such as "tomorrow"

ANS: B - Gestures "up" and "down" C - Able to insert round object into a hole D - Can find hidden objects but only in the first location Children in the sensorimotor, tertiary circular reactions stage of cognitive development show the behaviors of gesturing "up" and "down," have the ability to insert round objects into a hole, and can find hidden objects but only in the first location. The behaviors of referring to oneself by pronoun and using future-oriented words such as "tomorrow" are seen in the preoperational stage of cognitive development

What are risk factors of testicular cancer? (Select all that apply.) a. Hispanic b. Infertility c. Alcohol use d. Tobacco use e. Family history

ANS: B - Infertility D - Tobacco use E - Family history Risk factors of testicular cancer include infertility, tobacco use, and a family history. White, not Hispanic, ethnicity is a high risk, and alcohol use is not a risk.

The nurse is teaching a group of parents at a community education program about introducing solid foods to their infants. Which recommendations should the nurse include? (Select all that apply.) a. Spoon feeding should be introduced after an entire milk feeding. b. It is best to introduce a wide variety of foods during the first year. c. As solid food consumption increases, the quantity of milk should decrease. d. Introduction of low-calorie milk and food should be done by the end of the first year. e. Introduction of citrus fruits, meats, and eggs should be delayed until after 6 months of age. f. Each new food item should be introduced at 5- to 7-day intervals.

ANS: B - It is best to introduce a wide variety of foods during the first year. C - As solid food consumption increases, the quantity of milk should decrease. E - Introduction of citrus fruits, meats, and eggs should be delayed until after 6 months of age. F - Each new food item should be introduced at 5- to 7-day intervals. Teaching related to feeding an infant solid foods should include introducing a wide variety of foods because an infant has not developed a strong food preference as seen with a toddler. As solid food consumption increases, the amount of milk consumed should decrease to less than 1 L/day to prevent overfeeding. Introduction to citrus fruits, meats, and eggs should be delayed until after 6 months of age because of the potential to cause food allergies. New foods should be introduced at 5- to 7-day intervals to evaluate for food allergies. Spoon feedings should be introduced after a small ingestion of milk, not at the end of a milk feeding, to associate the activity with pleasure. In general, low-calorie milk and food should be avoided.

The nurse is assessing a family's use of complementary medicine practices. What practices are classified as nutrition, diet, and lifestyle or behavioral health changes? (Select all that apply.) a. Reflexology b. Macrobiotics c. Megavitamins d. Health risk reduction e. Chiropractic medicine

ANS: B - Macrobiotics C - Megavitamins D - Health risk reduction Macrobiotics, megavitamins, and health risk reduction are classified as nutrition, diet, and lifestyle or behavioral health changes. Reflexology and chiropractic medicine are classified as structural manipulation and energetic therapies.

What nursing interventions should the nurse plan for a hospitalized toddler to minimize fear of bodily injury? (Select all that apply.) a. Perform procedures slowly. b. Maintain parent-child contact. c. Use progressively smaller dressings on surgical incisions. d. Tell the child bleeding will stop after the needle is removed. e. Remove a dressing as quickly as possible from surgical incisions.

ANS: B - Maintain parent-child contact. C - Use progressively smaller dressings on surgical incisions. Whenever procedures are performed on young children, the most supportive intervention to minimize the fear of bodily injury is to do the procedure as quickly as possible while maintaining parent-child contact. Because of toddlers' and preschool children's poorly defined body boundaries, the use of bandages may be particularly helpful. For example, telling children that the bleeding will stop after the needle is removed does little to relieve their fears, but applying a small Band-Aid usually reassures them. The size of bandages is also significant to children in this age group; the larger the bandage, the more importance is attached to the wound. Watching their surgical dressings become successively smaller is one way young children can measure healing and improvement. Prematurely removing a dressing may cause these children considerable concern for their well-being.

The clinic nurse is assessing a 6-month-old infant during a well-child appointment. The nurse should use which approaches to alleviate the infant's stranger anxiety? (Select all that apply.) a. Talk in a loud voice. b. Meet the infant at eye level. c. Avoid sudden intrusive gestures. d. Maintain a safe distance initially. e. Pick up the infant and hold him or her closely.

ANS: B - Meet the infant at eye level. C - Avoid sudden intrusive gestures. D - Maintain a safe distance initially. The best approaches for the nurse to alleviate the infant's stranger anxiety are to talk softly; meet the infant at eye level (to appear smaller); maintain a safe distance from the infant; and avoid sudden, intrusive gestures, such as holding out the arms and smiling broadly. Talking in a loud voice and picking the infant up would increase the infant's anxiety.

A 12-month-old infant has been diagnosed with failure to thrive (FTT). Which assessment findings does the nurse expect to be documented with this infant? (Select all that apply.) a. Fear of strangers b. Minimal smiling c. Avoidance of eye contact d. Meeting developmental milestones e. Wide-eyed gaze and continual scan of the environment

ANS: B - Minimal smiling C - Avoidance of eye contact E - Wide-eyed gaze and continual scan of the environment Signs and symptoms of FTT include minimal smiling, avoidance of eye contact, and a wide-eyed gaze and continual scan of the environment ("radar gaze"). There is no fear of strangers, and there are developmental delays, including social, motor, adaptive, and language.

The nurse is caring for children on an adolescent-only unit. What growth and development milestones should the nurse expect from 11- and 14-year-old adolescents? (Select all that apply.) a. Self-centered with increased narcissism b. No major conflicts with parents c. Established abstract thought process d. Have a rich, idealistic fantasy life e. Highly value conformity to group norms f. Secondary sexual characteristics appear

ANS: B - No major conflicts with parents E - Highly value conformity to group norms F - Secondary sexual characteristics appear Growth and development milestones in the 11- to 14-year-old age group include minimal conflicts with parents (compared with the 15- to 17-year-old age group), a high value placed on conformity to the norm, and the appearance of secondary sexual characteristics. Self-centeredness and narcissism are seen in the 15- to 17-year-old age group along with a rich and idealistic fantasy life. Abstract thought processes are not well established until the 18- to 20-year-old age group.

The nurse is caring for an adolescent male with gynecomastia. What groups of drugs can induce gynecomastia in male adolescents? (Select all that apply.) a. Oral antibiotics b. Oral ketoconazoles c. Calcium channel blockers d. Histamine-2 receptor blockers e. Cancer chemotherapeutic agents

ANS: B - Oral ketoconazoles C - Calcium channel blockers D - Histamine-2 receptor blockers E - Cancer chemotherapeutic agents Gynecomastia may be drug induced; calcium channel blockers, cancer chemotherapeutic agents, histamine-2 receptor blockers, and oral ketoconazoles have all been shown to cause the disorder. Oral antibiotics have not been shown to cause gynecomastia

The school nurse teaches adolescents that the detrimental long-term effects of tanning are what? (Select all that apply.) a. Vitamin D deficiency b. Premature aging of the skin c. Exacerbates acne outbreaks d. Increased risk for skin cancer e. Possible phototoxic reactions

ANS: B - Premature aging of the skin D - Increased risk for skin cancer E - Possible phototoxic reactions Adolescents should be educated regarding the detrimental effects of sunlight on the skin. Long-term effects include premature aging of the skin; increased risk for skin cancer; and, in susceptible individuals, phototoxic reactions. Exposure to levels of sunlight cause an increase in vitamin D production. Tanning can often reduce outbreaks of acne

What influences a child's reaction to the stressors of hospitalization? (Select all that apply.) a. Gender b. Separation c. Support systems d. Developmental age e. Previous experience with illness

ANS: B - Separation C - Support systems D - Developmental age E - Previous experience with illness Major stressors of hospitalization include separation, loss of control, bodily injury, and pain. Children's reactions to these crises are influenced by their developmental age; previous experience with illness, separation, or hospitalization; innate and acquired coping skills; seriousness of the diagnosis; and support systems available. Gender does not have an effect on a child's reaction to stressors of hospitalization.

What identified characteristics occur more frequently in parents who abuse their children? (Select all that apply.) a. Older parents b. Socially isolated c. Middle class parents d. Single-parent families e. Few supportive relationships

ANS: B - Socially isolated D - Single-parent families E - Few supportive relationships Abusive families are often socially isolated and have few supportive relationships. Single-parent families are at higher risk for abuse. Younger parents more often are abusers of their children. Abusive parents have stressors such as low-income circumstances, with little education, and are not middle class parents.

A parent asks the nurse, "When will I know my child is ready for toilet training?" The nurse should include what in the response? (Select all that apply.) a. The child should be able to stay dry for 1 hour. b. The child should be able to sit, walk, and squat. c. The child should have regular bowel movements. d. The child should express a willingness to please.

ANS: B - The child should be able to sit, walk, and squat. C - The child should have regular bowel movements. D - The child should express a willingness to please. Signs of toilet training readiness include physical and psychological readiness. The ability to sit, walk, and squat and having regular bowel movements are physical readiness signs. Expressing a willingness to please is a sign of psychological readiness. The child should be able to stay dry for 2 hours, not 1.

What developmental achievements are demonstrated by a 4-year-old child? (Select all that apply.) a. Cares for self totally b. Throws a ball overhead c. Has a vocabulary of 1500 words d. Can skip and hop on alternate feet e. Tends to be selfish and impatient f. Commonly has an imaginary playmate

ANS: B - Throws a ball overhead C - Has a vocabulary of 1500 words E - Tends to be selfish and impatient F - Commonly has an imaginary playmate Developmental achievements for a 4-year-old child include throwing a ball overhead, having a vocabulary of 1500 words, tending to be selfish and impatient, and perhaps having an imaginary playmate. Caring for oneself totally and skipping and hopping on alternate feet are achievements normally seen in the 5-year-old age group.

The emergency department nurse is admitting a child with a temperature of 35° C (95° F). What physical effects of hypothermia should the nurse expect to observe in this child? (Select all that apply.) a. Bradycardia b. Vigorous shivering c. Decreased respiratory rate d. Decreased intestinal motility e. Task performance is impaired

ANS: B - Vigorous shivering D - Decreased intestinal motility E - Task performance is impaired Hypothermia has varying physical effects depending on the child's core temperature. At 35° C (95° F), a child would experience vigorous shivering, decreased intestinal motility, and task performance impairment. Bradycardia and decreased respiratory rate are physical effects observed as the body temperature continues to decrease

The nurse is administering activated charcoal to a preschool child with acetaminophen (Tylenol) poisoning. What potential complications from the use of activated charcoal should the nurse plan to assess for? (Select all that apply.) a. Diarrhea b. Vomiting c. Fluid retention d. Intestinal obstruction

ANS: B - Vomiting D - Intestinal obstruction Potential complications from the use of activated charcoal include vomiting and possible aspiration, constipation, and intestinal obstruction. Diarrhea and fluid retention are not potential complications of activated charcoal administration.

The school nurse is teaching bicycle safety to a group of school-age children. What should the nurse include in the session? (Select all that apply.) a. Ride double file when possible. b. Watch for and yield to pedestrians. c. Only ride double with someone your own size. d. Ride bicycles with traffic away from parked cars. e. Keep both hands on the handlebars except when signaling.

ANS: B - Watch for and yield to pedestrians. D - Ride bicycles with traffic away from parked cars. E - Keep both hands on the handlebars except when signaling. Bicycle safety includes watching for and yielding to pedestrians, riding bicycles with traffic away from parked cars, and keeping both hands on handlebars except when signaling. It is best to ride single file, not double file, and never to ride double on a bicycle.

The nurse is teaching parents of a 4-year-old child about fine motor developmental milestones. What milestones should the nurse include in the teaching session? (Select all that apply.) a. Can lace shoes b. Uses scissors successfully c. Builds a tower of nine or 10 cubes d. Builds a bridge with three cubes e. Adeptly places small pellets in a narrow-necked bottle

ANS: C - Builds a tower of nine or 10 cubes D - Builds a bridge with three cubes E - Adeptly places small pellets in a narrow-necked bottle The fine motor milestones of a 4-year-old child include building a tower of nine or 10 cubes, building a bridge with three cubes, and adeptly placing small pellets in a narrow-necked bottle. Lacing shoes and using scissors successfully are fine motor milestones seen at the age of 5 years.

The nurse is providing anticipatory guidance to parents of a 6-month-old on preventing an accidental poisoning injury. Which should the nurse include in the teaching? (Select all that apply.) a. Place plants on the floor. b. Place medications in a cupboard. c. Discard used containers of poisonous substances. d. Keep cosmetic and personal products out of the child's reach. e. Make sure that paint for furniture or toys does not contain lead.

ANS: C - Discard used containers of poisonous substances. D - Keep cosmetic and personal products out of the child's reach. E - Make sure that paint for furniture or toys does not contain lead. Anticipatory guidance for a 7-month-old infant to prevent a suffocation injury takes into account that the infant will become more active and eventually crawl, cruise, and walk. Used containers of poisonous substances should be discarded, cosmetic and personal products should be kept out of the child's reach, and paint for furniture or toys should be lead free. Plants should be hung out of reach or placed on a high shelf. Medications should be locked, not just placed in a cupboard.

The nurse is caring for a child who has a temperature of 30° C (86° F). What physical effects of hypothermia should the nurse expect to observe in this child? (Select all that apply.) a. Reduced urinary output b. Injury to peripheral tissue c. Increased blood pressure d. Tachycardia e. Irritability with loss of consciousness f. Rigid extremities

ANS: C - Increased blood pressure D - Tachycardia E - Irritability with loss of consciousness Hypothermia has varying physical effects depending on the child's core temperature. At 30° C (86° F), a child would experience an increase in blood pressure, tachycardia, and irritability followed by a loss of consciousness. Reduced urinary output from a decrease of blood flow to the kidneys, injury to peripheral tissue, and rigid extremities are physical effects observed as the body temperature continues to decrease

What are characteristics of late adolescence (18-20 years) with regard to sexuality? (Select all that apply.) a. Exploration of "self-appeal" b. Limited dating, usually group c. Intimacy involves commitment d. Growing capacity for mutuality and reciprocity e. May publicly identify as gay, lesbian, or bisexual

ANS: C - Intimacy involves commitment D - Growing capacity for mutuality and reciprocity E - May publicly identify as gay, lesbian, or bisexual Characteristics of late adolescence sexuality include intimacy involving commitment; growing capacity for mutuality and reciprocity; and publicly identifying as gay, lesbian, or bisexual. Exploration of "self-appeal" is a characteristic of middle adolescence sexuality. Limited dating, usually group, is a characteristic of early adolescence sexuality.

What are risk factors for sudden infant death syndrome? (Select all that apply.) a. Postterm b. Female gender c. Low Apgar scores d. Recent viral illness e. Native American infants

ANS: C - Low Apgar scores D - Recent viral illness E - Native American infants Infant risk factors for sudden infant death syndrome include those with low Apgar scores and recent viral illness and Native American infants. Preterm, not postterm, birth and male, not female, gender are other risk factors

The nurse understands that traits of gifted children include what? (Select all that apply.) a. Fair memory skills b. Limited sense of humor c. Perfectionism as a focus d. Inquisitive; always asking questions e. Displays intense feelings and emotion

ANS: C - Perfectionism as a focus D - Inquisitive; always asking questions E - Displays intense feelings and emotion Characteristics of gifted children include perfectionism as a focus; inquisitive, always asking questions; and displaying intense feelings and emotion. Memory skills are pronounced, and humor is exceptional.

What factors influence the effects of a child's hospitalization on siblings? (Select all that apply.) a. Older siblings b. Experiencing minimal changes c. Receiving little information about their ill brother or sister d. Being cared for outside the home by care providers who are not relatives e. Perceiving that their parents treat them differently compared with before their sibling's hospitalization

ANS: C - Receiving little information about their ill brother or sister D - Being cared for outside the home by care providers who are not relatives E - Perceiving that their parents treat them differently compared with before their sibling's hospitalization Various factors have been identified that influence the effects of a child's hospitalization on siblings. Factors that are related specifically to the hospital experience and increase the effects on the sibling are being cared for outside the home by care providers who are not relatives, receiving little information about their ill brother or sister, and perceiving that their parents treat them differently compared with before their sibling's hospitalization. Being younger, not older, and experiencing many changes, not minimal changes, are factors that influence the effects of a child's hospitalization on siblings.

What child behavior indicates to the nurse that temper tantrums have become a problem? (Select all that apply.) a. The child is 2 to 3 years old b. Tantrums occur at bedtime c. Tantrums occur past 5 years of age d. Tantrums last longer than 15 minutes e. Tantrums occur more than five times a day

ANS: C - Tantrums occur past 5 years of age D - Tantrums last longer than 15 minutes E - Tantrums occur more than five times a day Temper tantrums are common during the toddler years and essentially represent normal developmental behaviors. However, temper tantrums can be signs of serious problems. Temper tantrums that occur past 5 years of age, last longer than 15 minutes, or occur more than five times a day are considered abnormal and may indicate a serious problem. A popular time for a tantrum is before bedtime.

The nurse is teaching parents about safety for their "latchkey" children. What should the nurse include in the teaching session? (Select all that apply.) a. Teach the child first-aid procedures. b. Keep the key in an easy place to find. c. Teach the child weather-related safety. d. Teach the child to open the door for delivery people. e. Emphasize fire safety rules and conduct practice fire drills.

ANS: C - Teach the child weather-related safety. E - Emphasize fire safety rules and conduct practice fire drills. Safety for "latchkey" children includes teaching the child first-aid procedures, teaching the child weather-related safety, and emphasizing fire safety rules and conducting practice fire drills. Teach the child not to display keys and to always lock doors. The child should be taught to not open the door to anyone, even delivery people.

The nurse is teaching a parent of an 18-month-old about developmental milestones associated with feeding. What should the nurse include in the teaching? (Select all that apply.) a. The child will begin to use a fork. b. The child will be able use a straw and cup. c. The child will be able to hold a cup with both hands. d. The child will be able to drink from a cup with a lid. e. The child will begin to use a spoon but may turn it before reaching the mouth.

ANS: C - The child will be able to hold a cup with both hands. D - The child will be able to drink from a cup with a lid. E - The child will begin to use a spoon but may turn it before reaching the mouth. An 18-month-old child can hold a cup with both hands, is able to drink from a cup with a lid, and begins to use a spoon but may turn it before reaching the mouth. Using a fork is a developmental milestone of a 36-month-old child. Using a straw and cup is a milestone seen at 24 months.

What are characteristics of early adolescence (11-14 years) with regard to identity? (Select all that apply.) a. Mature sexual identity b. Increase in self-esteem c. Trying out of various roles d. Conformity to group norms e. Preoccupied with rapid body changes

ANS: C - Trying out of various roles D - Conformity to group norms E - Preoccupied with rapid body changes Characteristics of early adolescence identity include trying out of various roles, conformity to group norms, and preoccupation with rapid body changes. Mature sexual identity and increase in self-esteem are characteristics of late adolescent identity

A health care provider prescribes adrenaline (epinephrine), intramuscularly (IM) 0.15 mg, times one, stat. The medication label states: "Epinephrine 1:1000 1 mg/1 ml." The nurse prepares to administer the stat dose. How many milliliters will the nurse prepare to administer one dose? Fill in the blank. Record your answer using two decimal places.

ANS: 0.15

A health care provider prescribes iron supplements (Fer-In-Sol), 1 mg/kg/day orally (PO). The infant weighs 5 kg. The medication label states: "Fer-In-Sol 25 mg/1 ml." The nurse prepares to administer one dose. How many milliliters will the nurse prepare to administer one dose? Fill in the blank. Record your answer to one decimal place

ANS: 0.2

A health care provider prescribes vitamin D supplements, 300 IU orally, daily. The medication label states: "Vitamin D 1000 IU/10 ml." The nurse prepares to administer one dose. How many milliliters will the nurse prepare to administer one dose? Fill in the blank. Record your answer in a whole number

ANS: 3

The parent of 16-month-old child asks, "What is the best way to keep my child from getting into our medicines at home?" What should the nurse advise? a. "All medicines should be locked securely away." b. "The medicines should be placed in high cabinets." c. "Your child just needs to be taught not to touch medicines." d. "Medicines should not be kept in the homes of small children."

ANS: A - "All medicines should be locked securely away." The major reason for poisoning in the home is improper storage. Toddlers can climb, unlatch cabinets, and obtain access to high-security places. For medications, only a locked cabinet is safe. Toddlers can climb using furniture. High places are not a deterrent to an exploring toddler. Toddlers are not able to generalize that all the different forms of medications in the home may be dangerous. Keeping medicines out of the homes of small children is not feasible because many parents require medications for chronic or acute illnesses. Parents must be taught safe storage for their home and when they visit other homes.

The nurse is providing guidance strategies to a group of parents with toddlers at a community outreach program. Which statement by a parent indicates a correct understanding of the teaching? a. "I should expect my 24-month-old child to express some signs of readiness for toilet training." b. "I should be firm and structured when disciplining my 18-month-old child." c. "I should expect my 12-month-old child to start to develop a fear of darkness and to need a security blanket." d. "I should expect my 36-month-old child to understand time and proximity of events."

ANS: A - "I should expect my 24-month-old child to express some signs of readiness for toilet training." A 24-month-old toddler starts to show readiness for toilet training; it is important for the parent to be aware of this and be ready to start the process. At 18 months of age, a child needs consistent but gentle discipline because the child cannot yet understand firmness and structure with discipline. Development of fears and need for security items usually occurs at the end of the 18- to 24-month stage. A 36-month-old child does not yet understand time and proximity of events, so the parent needs to understand that the toddler cannot "hurry up or we will be late."

The nurse is teaching a parent with a 2-month-old infant who has been diagnosed with colic about ways to relieve colic. Which statement by the parent indicates the need for additional teaching? a. "I should let my infant cry for at least 30 minutes before I respond." b. "I will swaddle my infant tightly with a soft blanket." c. "I should massage my infant's abdomen whenever possible." d. "I will place my infant in an upright seat after feeding."

ANS: A - "I should let my infant cry for at least 30 minutes before I respond." Because the infant has been diagnosed with colic, the parent should respond to the infant immediately or any type of interventions to relieve colic may not be effective. Also, the infant may develop a mistrust of the world if his or her needs are not met. The parent should swaddle the baby tightly with a soft blanket, massage the baby's abdomen, and place the infant in an upright seat after a feeding to help relieve colic.

An infant, age 5 months, is brought to the clinic by his parents for a well-baby checkup. What is the best advice that the nurse should include at this time about injury prevention? a. "Keep buttons, beads, and other small objects out of his reach." b. "Do not permit him to chew paint from window ledges because he might absorb too much lead." c. "When he learns to roll over, you must supervise him whenever he is on a surface from which he might fall." d. "Lock the crib sides securely because he may stand and lean against them and fall out of bed."

ANS: A - "Keep buttons, beads, and other small objects out of his reach." Aspiration of foreign objects is a great risk at this age. Parents are instructed to keep small objects out of the infant's reach. At this age, the child is not mobile enough to reach window sills. If window sills have cracked or chipped paint, it needs to be removed before he is a toddler. This child should already be rolling over. This information is reinforced but should have been taught earlier. Pulling to a stand occurs between 8 and 12 months of age

The nurse is teaching parents about instilling a positive body image for the preschool age. What statement made by the parents indicates the teaching is understood? a. "We will make sure our child is praised about his or her looks." b. "We will help our child compare his or her size with other children." c. "We understand our child will have well-defined body boundaries." d. "We will be sure our child understands about being little for his or her age."

ANS: A - "We will make sure our child is praised about his or her looks." Because these are formative years for both boys and girls, parents should make efforts to instill positive principles regarding body image. Children at this age are aware of the meaning of words such as "pretty" or "ugly," and they reflect the opinions of others regarding their own appearance. Despite the advances in body image development, preschoolers have poorly defined body boundaries. By 5 years of age, children compare their size with that of their peers and can become conscious of being large or short, especially if others refer to them as "so big" or "so little" for their age. Parents should not suggest their child compare him- or herself with other children in regard to size, and parents should not focus on their child's size as being little.

The nurse is teaching parents about avoiding accidental burns with their toddler. What water heater setting should the nurse recommend to the parents? a. 120° F b. 130° F c. 140° F d. 150° F

ANS: A - 120° F The water heater should be set to limit household water temperatures to less than 49° C (120° F). At this temperature, it takes 10 minutes for exposure to the water to cause a full-thickness burn. Conversely, water temperatures of 54° C (130° F), the usual setting of most water heaters, expose household members to the risk of full-thickness burns within 30 seconds

At a well-child check-up, the nurse notes that an infant with a previous diagnosis of failure to thrive (FTT) is now steadily gaining weight. The nurse should recommend that fruit juice intake be limited to no more than how much? a. 4 oz/day b. 6 oz/day c. 8 oz/day d. 12 oz/day

ANS: A - 4 oz/day Restrict juice intake in children with FTT until adequate weight gain has been achieved with appropriate milk sources; thereafter, give no more than 4 oz/day of juice.

The nurse is collecting a stool sample from an infant with lactose intolerance. Which fecal pH should the nurse expect as the result? a. 5.5 b. 7.0 c. 7.5 d. 8

ANS: A - 5.5 An acidic pH (5-5.5) indicates malabsorption, which occurs with lactose intolerance. The normal pH of the stool is 7.0 to 7.5. A finding of 8 would be alkaline.

Deficiency of which vitamin or mineral results in an inadequate inflammatory response? a. A b. B1 c. C d. Zinc

ANS: A - A A deficiency of vitamin A results in an inadequate inflammatory response. Deficiencies of vitamins B1 and C result in decreased collagen formation. A deficiency of zinc leads to impaired epithelialization.

An 8-year-old girl tells the nurse that she has cancer because God is punishing her for "being bad." What should the nurse interpret this as? a. A common belief at this age b. Indicative of excessive family pressure c. Faith that forms the basis for most religions d. Suggestive of a failure to develop a conscience

ANS: A - A common belief at this age Children at this age may view illness or injury as a punishment for a real or imagined misbehavior. School-age children expect to be punished and tend to choose a punishment that they think "fits the crime." This is a common belief and not related to excessive family pressure. Many faiths do not include a God that causes cancer in response for "bad" behavior. This statement reflects the child's belief in what is right and wrong.

What is an important consideration in preventing injuries during middle childhood? a. Achieving social acceptance is a primary objective. b. The incidence of injuries in girls is significantly higher than it is in boys. c. Injuries from burns are the highest at this age because of fascination with fire. d. Lack of muscular coordination and control results in an increased incidence of injuries.

ANS: A - Achieving social acceptance is a primary objective. School-age children often participate in dangerous activities in an attempt to prove themselves worthy of acceptance. The incidence of injury during middle childhood is significantly higher in boys compared with girls. Motor vehicle collisions are the most common cause of severe injuries in children. Children have increasing muscular coordination. Children who are risk takers may have inadequate self-regulatory behavior

A toddler, age 16 months, falls down a few stairs. He gets up and "scolds" the stairs as if they caused him to fall. What is this an example of? a. Animism b. Ritualism c. Irreversibility d. Delayed cognitive development

ANS: A - Animism Animism is the attribution of lifelike qualities to inanimate objects. By scolding the stairs, the toddler is attributing human characteristics to them. Ritualism is the need to maintain sameness and reliability. It provides a sense of comfort to toddlers. Irreversibility is the inability to reverse or undo actions initiated physically. The toddler is acting in an age-appropriate manner.

A child, age 4 years, tells the nurse that she "needs a Band-Aid" where she had an injection. What nursing action should the nurse implement? a. Apply a Band-Aid. b. Ask her why she wants a Band-Aid. c. Explain why a Band-Aid is not needed. d. Show her that the bleeding has already stopped.

ANS: A - Apply a Band-Aid. Children in this age group still fear that their insides may leak out at the injection site. The nurse should be prepared to apply a small Band-Aid after the injection. No explanation should be required.

The nurse is teaching parents of preschoolers about plants that are poisonous. What plant should the nurse include in the teaching session? a. Azalea b. Begonia c. Boston fern d. Asparagus fern

ANS: A - Azalea All parts of the azalea are poisonous. Begonias, Boston ferns, and asparagus ferns are nonpoisonous plants

What is most descriptive of the spiritual development of older adolescents? a. Beliefs become more abstract. b. Rituals and practices become increasingly important. c. Strict observance of religious customs is common. d. Emphasis is placed on external manifestations, such as whether a person goes to church.

ANS: A - Beliefs become more abstract. Because of their abstract thinking abilities, adolescents are able to interpret analogies and symbols. Rituals, practices, and strict observance of religious customs become less important as adolescents question values and ideals of families. Adolescents question external manifestations when not supported by adherence to supportive behaviors.

The management of a child who has just been stung by a bee or wasp should include applying what? a. Cool compresses b. Antibiotic cream c. Warm compresses d. Corticosteroid cream

ANS: A - Cool compresses Bee or wasp stings are initially treated by carefully removing the stinger, cleansing with soap and water, applying cool compresses, and using common household agents such as lemon juice or a paste made with aspirin and baking soda. Antibiotic cream is unnecessary unless a secondary infection occurs. Warm compresses are avoided. Corticosteroid cream is not part of the initial therapy. If a severe reaction occurs, systemic corticosteroids may be indicated.

The nurse is discussing with a parent group the importance of fluoride for healthy teeth. What should the nurse recommend? a. Determine whether the water supply is fluoridated. b. Use fluoridated mouth rinses in children older than 1 year. c. Give fluoride supplements to infants beginning at age 2 months. d. Brush teeth with fluoridated toothpaste unless the fluoride content of water supply is adequate.

ANS: A - Determine whether the water supply is fluoridated. The decision about fluoride supplementation cannot be made until it is known whether the water supply contains fluoride and the amount. It is difficult to teach toddlers to spit out mouthwash. Swallowing fluoridated mouthwashes can contribute to fluorosis. Fluoride supplementation is not recommended until after age 6 months and then only if the water is not fluoridated. Fluoridated toothpaste is still indicated if the fluoride content of the water supply is adequate, but very small amounts are used.

In teaching parents about appropriate pacifier selection, the nurse should recommend which characteristic? a. Easily grasped handle b. Detachable shield for cleaning c. Soft, pliable material d. Ribbon or string to secure to clothing

ANS: A - Easily grasped handle A good pacifier should be easily grasped by the infant. One-piece construction is necessary to avoid having the nipple and guard separate, posing a risk for aspiration. The material should be sturdy and flexible. If the pacifier is too pliable, it may be aspirated. No ribbon or string should be attached. This poses additional risks.

The nurse is caring for a child with suspected ingestion of some type of poison. What action should the nurse take next after initiating cardiopulmonary resuscitation (CPR)? a. Empty the mouth of pills, plants, or other material. b. Question the victim and witness. c. Place the child in a side-lying position. d. Call poison control.

ANS: A - Empty the mouth of pills, plants, or other material. Emptying the mouth of any leftover pills, plants, or other ingested material is the next step after assessment and initiation of CPR if needed. Questioning the victim and witnesses, calling poison control, and placing the child in a side-lying position are follow-up steps

The parent of an infant with colic tells the nurse, "All this baby does is scream at me; it is a constant worry." What is the nurse's best action? a. Encourage the parent to verbalize feelings. b. Encourage the parent not to worry so much. c. Assess the parent for other signs of inadequate parenting. d. Reassure the parent that colic rarely lasts past age 9 months.

ANS: A - Encourage the parent to verbalize feelings. Colic is multifactorial, and no single treatment is effective for all infants. The parent is verbalizing concern and worry. The nurse should allow the parent to put these feelings into words. An empathetic, gentle, and reassuring attitude, in addition to suggestions about remedies, will help alleviate the parent's anxiety. The nurse should reassure the parent that he or she is not doing anything wrong. The infant with colic is experiencing spasmodic pain that is manifested by loud crying, in some cases up to 3 hours each day. Telling the parent that it will eventually go away does not help him or her through the current situation

The mother of a 7-month-old infant newly diagnosed with cystic fibrosis is rooming in with her infant. She is breastfeeding and provides all the care except for the medication administration. What should the nurse include in the plan of care? a. Ensuring that the mother has time away from the infant b. Making sure the mother is providing all of the infant's care c. Determining whether other family members can provide the necessary care so the mother can rest d. Contacting the social worker because of the mother's interference with the nursing care

ANS: A - Ensuring that the mother has time away from the infant The mother needs sufficient rest and nutrition so she can be effective as a caregiver. While the infant is hospitalized, the care is the responsibility of the nursing staff. The mother should be made comfortable with the care the staff provides in her absence. The mother has a right to provide care for the infant. The nursing staff and the mother should agree on the care division.

An 8-year-old girl is being admitted to the hospital from the emergency department with an injury from falling off her bicycle. What intervention will help her most in her adjustment to the hospital? a. Explain hospital schedules to her, such as mealtimes. b. Use terms such as "honey" and "dear" to show a caring attitude. c. Explain when parents can visit and why siblings cannot come to see her. d. Orient her parents, because she is too young, to her room and hospital facility.

ANS: A - Explain hospital schedules to her, such as mealtimes. School-age children need to have control of their environment. The nurse should offer explanations or prepare the child for what to expect. The nurse should refer to the child by the preferred name. Explaining when parents can visit and why siblings cannot come focuses on the limitations rather than helping her adjust to the hospital. At the age of 8 years, the child should be oriented to the environment along with the parents.

What method is the most commonly used in completed suicides? a. Firearms b. Drug overdose c. Self-inflicted laceration d. Carbon monoxide poisoning

ANS: A - Firearms Firearms are the most commonly used instruments in completed suicides among both males and females. For completed suicides in adolescent boys, firearms are followed by hanging and overdose. For adolescent girls, overdose and strangulation are the next most common means of completed suicide. The most common method of suicide attempt is overdose or ingestion of potentially toxic substances such as drugs. The second most common method of suicide attempt is self-inflicted laceration. Carbon monoxide poisoning is not one of the more frequent forms of suicide completion.

The mother of a 3-month-old breastfed infant asks about giving her baby water because it is summer and very warm. What should the nurse tell her? a. Fluids in addition to breast milk are not needed. b. Water should be given if the infant seems to nurse longer than usual. c. Clear juices are better than water to promote adequate fluid intake. d. Water once or twice a day will make up for losses resulting from environmental temperature.

ANS: A - Fluids in addition to breast milk are not needed. Infants who are breastfed or bottle fed do not need additional water during the first 4 months of life. Excessive intake of water can create problems such as water intoxication, hyponatremia, or failure to thrive. Juices provide empty calories for infants.

A 9-year-old child has just been diagnosed with recurrent abdominal pain (RAP). In preparing for discharge, the nurse should include what in the home care instructions to the parents? a. Following a high-fiber diet b. Using stimulant laxatives c. Using ice packs on the abdomen when pain occurs d. Sitting on the toilet for 30 minutes after each meal

ANS: A - Following a high-fiber diet A high-fiber diet with possible addition of bulk laxatives is beneficial for children with RAP. Bulk-forming laxatives such as psyllium are recommended. Stimulant laxatives may produce painful cramping for the child. Warm packs, such as a heating pad, may help ease the discomfort. Bowel training is recommended to assist the child in establishing regular bowel habits. Thirty minutes is too long for the child to sit on the toilet. The time should be limited to 15 minutes

Parents tell the nurse that their toddler eats little at mealtime, only sits at the table with the family briefly, and wants snacks "all the time." What should the nurse recommend? a. Give her nutritious snacks. b. Offer rewards for eating at mealtimes. c. Avoid snacks so she is hungry at mealtimes. d. Explain to her in a firm manner what is expected of her.

ANS: A - Give her nutritious snacks. Most toddlers exhibit a physiologic anorexia in response to the decreased nutritional requirements associated with the slower growth rate. Parents should assist the child in developing healthy eating habits. Toddlers are often unable to sit through a meal. Frequent nutritious snacks are a good way to ensure proper nutrition. To help with developing healthy eating habits, food should be not be used as positive or negative reinforcement for behavior. The child may develop habits of overeating or eat non-nutritious foods in response. A toddler is not able to understand explanations of what is expected of her and comply with the expectations

What information should be given to the parents of a 12-month-old child regarding appropriate play activities for this age? a. Give large push-pull toys for kinetic stimulation. b. Place a cradle gym across the crib to help develop fine motor skills. c. Provide the child with finger paints to enhance fine motor skills. d. Provide a stick horse to develop gross motor coordination.

ANS: A - Give large push-pull toys for kinetic stimulation. A 12-month-old child is able to pull to a stand and walk holding on or independently. Appropriate toys for this age child include large push-pull toys for kinetic stimulation. A cradle gym should not be placed across the crib. Finger paints are appropriate for older children. A 12-month-old child does not have the stability to use a stick horse.

Girls experience an increase in weight and fat deposition during puberty. What do nursing considerations related to this include? a. Give reassurance that these changes are normal. b. Suggest dietary measures to control weight gain. c. Encourage a low-fat diet to prevent fat deposition. d. Recommend increased exercise to control weight gain.

ANS: A - Give reassurance that these changes are normal. A certain amount of fat is increased along with lean body mass to fill the characteristic contours of the adolescent's gender. A healthy balance must be achieved between expected healthy weight gain and obesity. Suggesting dietary measures or increased exercise to control weight gain would not be recommended unless weight gain was excessive because eating disorders can develop in this group. Some fat deposition is essential for normal hormonal regulation. Menarche is delayed in girls with body fat contents that are too low.

According to Erikson, the psychosocial task of adolescence is developing what? a. Identity b. Intimacy c. Initiative d. Independence

ANS: A - Identity Traditional psychosocial theory holds that the developmental crises of adolescence lead to the formation of a sense of identity. Intimacy is the developmental stage for early adulthood. Independence is not one of Erikson's developmental stages.

The parents of a 2-year-old child tell the nurse they are concerned because the toddler has started to use "baby talk" since the arrival of their new baby. What should the nurse recommend? a. Ignore the baby talk. b. Tell the toddler frequently, "You are a big kid now." c. Explain to the toddler that baby talk is for babies. d. Encourage the toddler to practice more advanced patterns of speech.

ANS: A - Ignore the baby talk. Baby talk is a sign of regression in the toddler. Often toddlers attempt to cope with a stressful situation by reverting to patterns of behavior that were successful in earlier stages of development. It should be ignored while the parents praise the child for developmentally appropriate behaviors. Regression is children's way of expressing stress. The parents should not introduce new expectations and allow the child to master the developmental tasks without criticism.

How does the onset of the pubertal growth spurt compare in girls and boys? a. In girls, it occurs about 1 year before it appears in boys. b. In girls, it occurs about 3 years before it appears in boys. c. In boys. it occurs about 1 year before it appears in girls. d. It is about the same in both boys and girls.

ANS: A - In girls, it occurs about 1 year before it appears in boys The average age of onset is 9 1/2 years for girls and 10 1/2 years for boys. Although pubertal growth spurts may occur in girls 3 years before it appears in boys on an individual basis, the average difference is 1 year. Usually girls begin their pubertal growth spurt earlier than boys.

What is the leading cause of death during the toddler period? a. Injuries b. Infectious diseases c. Childhood diseases d. Congenital disorders

ANS: A - Injuries Injuries are the most common cause of death in children ages 1 through 4 years. It is the highest rate of death from injuries of any childhood age group except adolescence. Congenital disorders are the second leading cause of death in this age group. Infectious and childhood diseases are less common causes of death in this age group.

What may a clinical manifestations of failure to thrive (FTT) in a 13-month-old include? a. Irregularity in activities of daily living b. Preferring solid food to milk or formula c. Weight that is at or below the 10th percentile d. Appropriate achievement of developmental landmarks

ANS: A - Irregularity in activities of daily living One of the clinical manifestations of children with FTT is irregularity or low rhythmicity in activities of daily living. Children with FTT often refuse to switch from liquids to solid foods. Weight below the fifth percentile is indicative of FTT. Developmental delays, including social, motor, adaptive, and language, exist.

The nurse is checking reflexes on a 7-month-old infant. When the infant is suspended in a horizontal prone position, the head is raised and the legs and spine are extended. Which reflex is this? a. Landau b. Parachute c. Body righting d. Labyrinth righting

ANS: A - Landau When the infant is suspended in a horizontal prone position, the head is raised and the legs and spine are extended; this describes the Landau reflex. It appears at 6 to 8 months and persists until 12 to 24 months. The parachute reflex occurs when the infant is suspended in a horizontal prone position and suddenly thrust downward; the infant extends the hands and fingers forward as if to protect against falling. This appears at age 7 to 9 months and lasts indefinitely. Body righting occurs when turning the hips and shoulders to one side causes all other body parts to follow. It appears at 6 months of age and persists until 24 to 36 months. The labyrinth-righting reflex appears at 2 months and is strongest at 10 months. This reflex involves holding infants in the prone or supine position. They are able to raise their heads.

The nurse is admitting a child with frostbite. What health care prescription should the nurse question and verify? a. Massage the injured tissue. b. Apply a loose dressing after rewarming. c. Avoid any application of dry heat to the area. d. Administer acetaminophen (Tylenol) for discomfort.

ANS: A - Massage the injured tissue. A frostbite victim should not have injured tissue rubbed. It is contraindicated because it can cause damage by rupture of crystallized cells. After rewarming, a loose dressing is applied to the affected skin, and analgesia is administered if indicated. Dry heat is not applied.

Which factors will decrease iron absorption and should not be given at the same time as an iron supplement? a. Milk b. Fruit juice c. Multivitamin d. Meat, fish, poultry

ANS: A - Milk Many foods interfere with iron absorption and should be avoided when iron is consumed. These foods include phosphates found in milk, phytates found in cereals, and oxalates found in many vegetables. Vitamin C-containing juices enhance the absorption of iron. Multivitamins may contain iron; no contraindication exists to taking the two together. Meat, fish, and poultry do not affect absorption.

A child is admitted with a suspected diagnosis of Munchausen syndrome by proxy (MSBP). What is an important consideration in the care of this child? a. Monitoring the parents whenever they are with the child b. Reassuring the parents that the cause of the disorder will be found c. Teaching the parents how to obtain necessary specimens d. Supporting the parents as they cope with diagnosis of a chronic illness

ANS: A - Monitoring the parents whenever they are with the child MSBP refers to an illness that one person fabricates or induces in another. The child must be continuously observed for development of symptoms to determine the cause. MSBP is caused by an individual harming the child for the purpose of gaining attention. Nursing staff should obtain all specimens for analyzing. This minimizes the possibility of the abuser contaminating the sample. The child must be supported through the diagnosis of MSBP. The abuser must be identified and the child protected from that individual.

An infant has been diagnosed with an allergy to milk. In teaching the parent how to meet the infant's nutritional needs, the nurse states that a. Most children will grow out of the allergy. b. All dairy products must be eliminated from the child's diet. c. It is important to have the entire family follow the special diet. d. Antihistamines can be used so the child can have milk products.

ANS: A - Most children will grow out of the allergy. Approximately 80% of children with cow's milk allergy develop tolerance by the fifth birthday. The child can have eggs. Any food that has milk as a component or filler is eliminated. These foods include processed meats, salad dressings, soups, and milk chocolate. Having the entire family follow the special diet would provide support for the child, but the nutritional needs of other family members must be addressed. Antihistamines are not used for food allergies.

Parents of a hospitalized child often question the skill of staff. The nurse interprets this behavior by the parents as what? a. Normal b. Paranoid c. Indifferent d. Wanting attention

ANS: A - Normal Recent research has identified common themes among parents whose children were hospitalized, including feeling an overall sense of helplessness, questioning the skills of staff, accepting the reality of hospitalization, needing to have information explained in simple language, dealing with fear, coping with uncertainty, and seeking reassurance from the health care team. The behavior does not indicate the parents are paranoid, indifferent, or wanting attention.

The nurse is providing support to parents adapting to the hospitalization of their child to the pediatric intensive care unit. The nurse notices that the parents keep asking the same questions. What should the nurse do? a. Patiently continue to answer questions, trying different approaches. b. Kindly refer them to someone else for answering their questions. c. Recognize that some parents cannot understand explanations. d. Suggest that they ask their questions when they are not upset.

ANS: A - Patiently continue to answer questions, trying different approaches. In addition to a general pediatric unit, children may be admitted to special facilities such as an ambulatory or outpatient setting, an isolation room, or intensive care. Wherever the location, the core principles of patient and family-centered care provide a foundation for all communication and interventions with the patient, family, and health care team. The nurse should do the therapeutic action and patiently continue to answer questions, trying different approaches.

The nurse has just given a subcutaneous injection to a preschool child, and the child asks for a Band-Aid over the site. Which action should the nurse implement? a. Place a Band-Aid over the site. b. Massage the injection site with an alcohol swab. c. Show the child there is no bleeding from the site. d. Explain that a Band-Aid is not needed after a subcutaneous injection.

ANS: A - Place a Band-Aid over the site. Despite the advances in body image development, preschoolers have poorly defined body boundaries and little knowledge of their internal anatomy. Intrusive experiences are frightening, especially those that disrupt the integrity of the skin (e.g., injections and surgery). They fear that all their blood and "insides" can leak out if the skin is "broken." Therefore, preschoolers may believe it is critical to use bandages after an injury. The nurse should place a Band-Aid over the site.

What is an appropriate play activity for a 7-month-old infant to encourage visual stimulation? a. Playing peek-a-boo b. Playing pat-a-cake c. Imitating animal sounds d. Showing how to clap hands

ANS: A - Playing peek-a-boo Because object permanence is a new achievement, peek-a-boo is an excellent activity to practice this new skill for visual stimulation. Playing pat-a-cake and showing how to clap hands help with kinetic stimulation. Imitating animal sounds helps with auditory stimulation.

What is a priority goal in the postpartum care of an adolescent mother? a. Prevention of subsequent pregnancies b. Ensuring that the father of the baby cares for the child c. Returning the mother to a prepregnancy lifestyle d. Facilitating formula feeding to minimize interruptions

ANS: A - Prevention of subsequent pregnancies Postpartum care of the adolescent is directed at preventing subsequent pregnancies and enhancing life outcomes for the teen parents and child. Health care programs should provide comprehensive contraceptive services at the same time the child is seen for appointments. Ensuring the father of the baby cares for the child is not part of the postpartum care of the mother. The adolescent mother cannot return to a prepregnancy lifestyle. She now has an infant to care for. Breastfeeding is recommended for the infant. The nurse and mother should explore the best nutrition for both the mother's needs and those of the infant

When a preschool-age child is hospitalized without adequate preparation, the nurse should recognize that the child may likely see hospitalization as what? a. Punishment b. Loss of parental love c. Threat to the child's self-image d. Loss of companionship with friends

ANS: A - Punishment The rationale for preparing children for the hospital experience and related procedures is based on the principle that a fear of the unknown (fantasy) exceeds fear of the known. Preschool-age children see hospitalization as a punishment. Loss of parental love would be a toddler's reaction. Threat to the child's self-image would be a school-age child's reaction. Loss of companionship with friends would be an adolescent's reaction.

A breastfed infant is being seen in the clinic for a 6-month checkup. The mother tells the nurse that the infant recently began to suck her thumb. Which is the best nursing intervention? a. Reassure the mother that this is normal at this age. b. Recommend the mother substitute a pacifier for her thumb. c. Assess the infant for other signs of sensory deprivation. d. Suggest the mother breastfeed the infant more often to satisfy her sucking needs.

ANS: A - Reassure the mother that this is normal at this age. Sucking is an infant's chief pleasure, and the infant may not be satisfied by bottle-feeding or breastfeeding alone. During infancy and early childhood, there is no need to restrict nonnutritive sucking. The nurse should explore with the mother her feelings about a pacifier versus the thumb. No data support that the child has sensory deprivation.

A 13-year-old boy comes to the school nurse complaining of sudden and severe scrotal pain. He denies any trauma to the scrotum. What is the most appropriate nursing action? a. Refer him for immediate medical evaluation. b. Administer analgesics and recommend scrotal support. c. Apply an ice bag and observe for increasing pain. d. Reassure the adolescent that occasional pain is common with the changes of puberty.

ANS: A - Refer him for immediate medical evaluation. Any adolescent boy with redness, swelling, or pain in the scrotum is referred for immediate evaluation. These are signs of testicular torsion, which is a medical emergency. If the possibility of testicular torsion is eliminated, appropriate interventions include administering analgesics and recommending scrotal support. applying an ice bag and observing for increasing pain. and reassuring the adolescent that occasional pain is common with the changes of puberty.

A boy age 4 1/2 years has been having increasingly frequent angry outbursts in preschool. He is aggressive toward the other children and the teachers. This behavior has been a problem for approximately 8 to 10 weeks. His parent asks the nurse for advice. What is the most appropriate intervention? a. Refer the child for a professional psychosocial assessment. b. Explain that this is normal in preschoolers, especially boys. c. Encourage the parent to try more consistent and firm discipline. d. Talk to the preschool teacher to obtain validation for behavior parent reports.

ANS: A - Refer the child for a professional psychosocial assessment. The preschool years are a time when children learn socially acceptable behavior. The difference between normal and problematic behavior is not the behavior but the severity, frequency, and duration. This child's behavior meets the definition requiring professional evaluation. Some aggressive behavior is within normal limits, but at 8 to 10 weeks, this behavior has persisted too long. There is no indication that the parent is using inconsistent discipline. A part of the evaluation is to obtain validation for behavior parent reports

The nurse should expect a toddler to cope with the stress of a short period of separation from parents by displaying what? a. Regression b. Happiness c. Detachment d. Indifference

ANS: A - Regression Children in the toddler stage demonstrate goal-directed behaviors when separated from parents for short periods. They may demonstrate displeasure on the parents' return or departure by having temper tantrums; refusing to comply with the usual routines of mealtime, bedtime, or toileting; or regressing to more primitive levels of development. Detachment would be seen with a prolonged absence of parents, not a short one. Toddlers would not be indifferent or happy when experiencing short separations from parents.

A 9-year-old girl often comes to the school nurse complaining of stomach pains. Her teacher says she is completing her school work satisfactorily but lately has been somewhat aggressive and stubborn in the classroom. The school nurse should recognize this as which? a. Signs of stress b. Developmental delay c. Lack of adjustment to school environment d. Physical problem that needs medical intervention

ANS: A - Signs of stress Signs of stress include stomach pains or headache, sleep problems, bedwetting, changes in eating habits, aggressive or stubborn behavior, reluctance to participate, or regression to earlier behaviors. The child is completing school work satisfactorily; any developmental delay would have been diagnosed earlier. The teacher reports that this is a departure from the child's normal behavior. Adjustment issues would most likely be evident soon after a change. Medical intervention is not immediately required. Recognizing that this constellation of symptoms can indicate stress, the nurse should help the child identify sources of stress and how to use stress reduction techniques. The parents are involved in the evaluation process.

During a well-child visit, the nurse practitioner provides guidance about promoting healthy eating in a child who is overweight. What does the nurse advise? a. Slow down eating meals. b. Avoid between-meal snacks. c. Include low-fat foods in meals. d. Use foods that child likes as special treats.

ANS: A - Slow down eating meals. When a child slows down the eating process, it is easier to recognize signs of fullness. If food is consumed rapidly, this feedback is lost. Regular meals and snacks are encouraged to prevent the child from becoming too hungry and overeating. Low-fat foods are usually higher in calories than the regular versions. Nutritional labels should be checked and foods high in sugar and calories avoided. Food should not be used as a special treat or reward; this encourages the child to use food as comfort measures in response to boredom and stress.

The school nurse needs to obtain authorization for a child who requires medications while at school. From whom does the nurse obtain the authorization? a. The parents b. The pharmacist c. The school administrator d. The prescribing practitioner

ANS: A - The parents A child who requires medication during the school day requires written authorization from the parent or guardian. Most schools also require that the medication be in the original container appropriately labeled by the pharmacist or physician. Some schools allow children to receive over-the-counter medications with parental permission. The pharmacist may be asked to appropriately label the medication for use at the school, but authorization is not required. The school administration should have a policy in place that facilitates the administration of medications for children who need them. The prescribing practitioner is responsible for ensuring that the medication is appropriate for the child. Because the child is a minor, parental consent is required.

What is descriptive of the nutritional requirements of preschool children? a. The quality of the food consumed is more important than the quantity. b. The average daily intake of preschoolers should be about 3000 calories. c. Nutritional requirements for preschoolers are very different from requirements for toddlers. d. Requirements for calories per unit of body weight increase slightly during the preschool period.

ANS: A - The quality of the food consumed is more important than the quantity. Parents need to be reassured that the quality of food eaten is more important than the quantity. Children are able to self-regulate their intake when offered foods high in nutritional value. The average daily caloric intake should be approximately 1800 calories. Toddlers and preschoolers have similar nutritional requirements. There is an overall slight decrease in needed calories and fluids during the preschool period.

What statement is correct about young children who report sexual abuse? a. They may exhibit various behavioral manifestations. b. In more than half the cases, the child has fabricated the story. c. Their stories should not be believed unless other evidence is apparent. d. They should be able to retell the story the same way to another person.

ANS: A - They may exhibit various behavioral manifestations. Victims of sexual abuse have no typical profile. The child may exhibit various behavioral manifestations, none of which is diagnostic for sexual abuse. When children report potentially sexually abusive experiences, their reports need to be taken seriously. Other children in the household also need to be evaluated. In children who are sexually abused, it is often difficult to identify other evidence. In one study, approximately 96% of children who were sexually abused had normal genital and anal findings. The ability to retell the story is partly dependent on the child's cognitive level. Children who repeatedly tell identical stories may have been coached.

A pregnant 15-year-old adolescent tells the nurse that she did not use any form of contraception because she was afraid her parents would find out. The nurse should recognize what? a. This is a frequent reason given by adolescents. b. This suggests a poor parent-child relationship. c. This is not a good reason to not get contraception. d. This indicates that the adolescent is unaware of her legal rights.

ANS: A - This is a frequent reason given by adolescents. This is one of the most common reasons given by teenagers for not using contraception. Although it is optimum for the parents to be involved in the health care of adolescents, some adolescents require confidential care. Privacy is important as they develop their personal identity and establish relationships. The adolescent may be concerned about parental judgment. The adolescent should discuss with the health care provider contraception that meets her needs; some of the longer acting birth control methods may be preferable. The adolescent did not tell the nurse that she was unaware that she could legally obtain contraceptive materials; she was concerned about her parents.

The parents of a 3-month-old infant report that their infant sleeps supine (face up) but is often prone (face down) while awake. The nurse's response should be based on remembering what? a. This is acceptable to encourage head control and turning over. b. This is acceptable to encourage fine motor development. c. This is unacceptable because of the risk of sudden infant death syndrome (SIDS). d. This is unacceptable because it does not encourage achievement of developmental milestones.

ANS: A - This is acceptable to encourage head control and turning over. These parents are implementing the guidelines to reduce the risk of SIDS. Infants should sleep on their backs to reduce the risk of SIDS and then be placed on their abdomens when awake to enhance achievement of milestones such as head control. These position changes encourage gross motor, not fine motor, development.

Parents of an 18-month-old boy tells the nurse that he says "no" to everything and has rapid mood swings. If he is scolded, he shows anger and then immediately wants to be held. What is the nurse's best interpretation of this behavior? a. This is normal behavior for his age. b. This is unusual behavior for his age. c. He is not effectively coping with stress. d. He is showing he needs more attention.

ANS: A - This is normal behavior for his age. Toddlers use distinct behaviors in the quest for autonomy. They express their will with continued negativity and use of the word "no." Children at this age also have rapid mood swings. The nurse should reassure the parents that their child is engaged in expected behavior for an 18-month-old

According to Piaget, magical thinking is the belief of which? a. Thoughts are all powerful. b. God is an imaginary friend. c. Events have cause and effect. d. If the skin is broken, the insides will come out.

ANS: A - Thoughts are all powerful. Because of their egocentrism and transductive reasoning, preschoolers believe that thoughts are all powerful. Believing God is an imaginary friend is an example of concrete thinking in a preschooler's spiritual development. Cause-and-effect implies logical thought, not magical thinking. Believing that if the skin is broken, the insides will come out is an example of concrete thinking in development of body image.

The nurse is planning care for a hospitalized toddler. What is the rationale for planning to continue the toddler's rituals while hospitalized? a. To provide security b. To prevent regression c. To prevent dependency d. To decrease negativism

ANS: A - To provide security Ritualism, the need to maintain sameness and reliability, provides a sense of security and comfort. It will not prevent regression or dependency or decrease negativism.

The nurse is teaching parents of a preschool child strategies to implement when the child delays going to bed. What strategy should the nurse recommend? a. Use consistent bedtime rituals. b. Give in to attention-seeking behavior. c. Take the child into the parent's bed for an hour. d. Allow the child to stay up past the decided bedtime.

ANS: A - Use consistent bedtime rituals. For children who delay going to bed, a recommended approach involves a consistent bedtime ritual and emphasizing the normalcy of this type of behavior in young children. Parents should ignore attention-seeking behavior, and the child should not be taken into the parents' bed or allowed to stay up past a reasonable hour.

A sexually active adolescent asks the school nurse about prevention of sexually transmitted infections (STIs). What should the nurse recommend? a. Use of condoms b. Prophylactic antibiotics c. Any type of contraception method d. Withdrawal method of contraception

ANS: A - Use of condoms When used appropriately, condoms provide a barrier to the organisms that cause STIs. Prophylactic antibiotics are not recommended; they are effective only against bacteria, not viruses. Only condoms create a physical barrier that prevents contact with the organisms.

An important distinction in understanding substance abuse is that drug misuse, abuse, and addiction are considered what? a. Voluntary behaviors based on psychosocial needs b. Problems that occur in conjunction with addiction c. Involuntary physiologic responses to the pharmacologic characteristics of drugs d. Legal use of substances for purposes other than medicinal.

ANS: A - Voluntary behaviors based on psychosocial needs Drug misuse, abuse, and addiction are considered voluntary behaviors. Cultural norms define what is abuse and misuse. Addiction is a psychologic dependence on a substance with or without physical dependence. Physical dependence is an involuntary response to the pharmacologic characteristics of the drug such as an opiate or alcohol. Legality is not always a factor in substance abuse. Legal substances such as alcohol and tobacco can also be misused or abused and can cause addiction

Lactose intolerance is diagnosed in an 11-month-old infant. Which should the nurse recommend as a milk substitute? a. Yogurt b. Ice cream c. Fortified cereal d. Cow's milk-based formula

ANS: A - Yogurt Yogurt contains the inactive lactase enzyme, which is activated by the temperature and pH of the duodenum. This lactase activity substitutes for the lack of endogenous lactase. Ice cream and cow's milk-based formula contain lactose, which will probably not be tolerated by the child. Fortified cereal does not have the nutritional equivalents of milk

The parents of a 2-month-old boy are concerned about spoiling their son by picking him up when he cries. What is the nurse's best response? a. "Allow him to cry for no longer than 15 minutes and then pick him up." b. "Babies need comforting and cuddling. Meeting these needs will not spoil him." c. "Babies this young cry when they are hungry. Try feeding him when he cries." d. "If he isn't soiled or wet, leave him, and he'll cry himself to sleep."

ANS: B - "Babies need comforting and cuddling. Meeting these needs will not spoil him." Parents need to learn that a "spoiled child" is a response to inconsistent discipline and limit setting. It is important to meet the infant's developmental needs, including comforting and cuddling. The data suggest that responding to a child's crying can actually decrease the overall crying time. Allowing him to cry for no longer than 15 minutes and then picking him up will reinforce prolonged crying. Infants at this age have other needs besides feeding. The parents should be taught to identify their infant's cues. Counseling parents on letting the baby cry himself to sleep when not soiled or wet refers to sleep issues, not general infant behavior.

The nurse is discussing development and play activities with the parent of a 2-month-old boy. Which statement by the parent would indicate a correct understanding of the teaching? a. "I can give my baby a ball of yarn to pull apart or different textured fabrics to feel." b. "I can use a music box and soft mobiles as appropriate play activities for my baby." c. "I should introduce a cup and spoon or push-pull toys for my baby at this age." d. "I do not have to worry about appropriate play activities at this age."

ANS: B - "I can use a music box and soft mobiles as appropriate play activities for my baby." Music boxes and soft mobiles are appropriate play activities for a 2-month-old infant. A ball of yarn to pull apart or different textured fabrics are appropriate for an infant at 6 to 9 months. A cup and spoon or push-pull toys are appropriate for an older infant. Infants of all ages should be exposed to appropriate types of stimulation.

The nurse is caring for a hospitalized 4-year-old boy. His parents tell the nurse they will be back to visit at 6 PM. When he asks the nurse when his parents are coming, what would the nurse's best response be? a. "They will be here soon." b. "They will come after dinner." c. "Let me show you on the clock when 6 PM is." d. "I will tell you every time I see you how much longer it will be."

ANS: B - "They will come after dinner." A 4-year-old child understands time in relation to events such as meals. Children perceive "soon" as a very short time. The nurse may lose the child's trust if his parents do not return in the time he perceives as "soon." Children cannot read or use a clock for practical purposes until age 7 years. "I will tell you every time I see you how much longer it will be" assumes the child understands the concepts of hours and minutes, which does not occur until age 5 or 6 years.

A 12-year-old girl asks the nurse about an increase in clear white odorless vaginal discharge. What response should the nurse give? a. "This may mean a yeast infection." b. "This is normal before menstruation starts." c. "This is caused by an increase in progesterone." d. "This is possibly a sign of a sexually transmitted infection."

ANS: B - "This is normal before menstruation starts." Early in puberty, there is often an increase in normal vaginal discharge (physiologic leukorrhea) associated with uterine development. Girls or their parents may be concerned that this vaginal discharge is a sign of infection. The nurse can reassure them that the discharge is normal and a sign that the uterus is preparing for menstruation. It is caused by an increase in estrogen, not progesterone.

A male school-age student asks the school nurse, "How much with my height increase in a year?" The nurse should give which response? a. "Your height will increase on average 1 inch a year." b. "Your height will increase on average 2 inches a year." c. "Your height will increase on average 3 inches a year." d. "Your height will increase on average 4 inches a year."

ANS: B - "Your height will increase on average 2 inches a year." Between the ages of 6 and 12 years, children grow an average of 5 cm (2 inches) per year.

A parent brings a 12-month-old infant into the emergency department and tells the nurse that the infant is allergic to peanuts and was accidentally given a cookie with peanuts in it. The infant is dyspneic, wheezing, and cyanotic. The health care provider has prescribed a dose of epinephrine to be administered. The infant weighs 24 lb. How many milligrams of epinephrine should be administered? a. 0.11 to 0.33 mg b. 0.011 to 0.3 mg c. 1.1 to 3.3 mg d. 11 to 33 mg

ANS: B - 0.011 to 0.3 mg The correct dose of epinephrine to use in the emergency management of an anaphylactic reaction is 0.001 mg/kg up to a maximum of 0.3 mg, giving a range of 0.011 to 0.3 mg using a weight of 11 kg (24 lb)

The nurse understands that which guideline should be followed to determine serving sizes for toddlers? a. 1/2 tbsp of solid food per year of age b. 1 tbsp of solid food per year of age c. 2 tbsp of solid food per year of age d. 2 1/2 tbsp of solid food per year of age

ANS: B - 1 tbsp of solid food per year of age To determine serving sizes for young children, the guideline to follow is 1 tbsp of solid food per year of age. One-half tbsp per year of age would not be adequate. Two or 2 1/2 tbsp per year of age would be excessive.

A child has had contact with some poison ivy. The school nurse understands that the full-blown reaction should be evident after how many days? a. 1 day b. 2 days c. 3 days d. 4 days

ANS: B - 2 days The full-blown reaction to poison ivy is evident after about 2 days, with linear patches or streaks of erythemic, raised, fluid-filled vesicles; swelling; and persistent itching at the site of contact.

At which age should the nurse expect an infant to begin smiling in response to pleasurable stimuli? a. 1 month b. 2 months c. 3 months d. 4 months

ANS: B - 2 months At age 2 months, the infant has a social, responsive smile. A reflex smile is usually present at age 1 month. A 3-month-old infant can recognize familiar faces. At age 4 months, infants can enjoy social interactions.

The nurse is explaining average weight gain during the preschool years to a group of parents. Which average weight gain should the nurse suggest to the parents? a. 1 to 2 kg b. 2 to 3 kg c. 3 to 4 kg d. 4 to 5 kg

ANS: B - 2 to 3 kg The average weight gain remains approximately 2 to 3 kg (4.5-6.5 lb) per year during the preschool period.

The nurse is assessing a toddler's visual acuity. Which visual acuity is considered acceptable during the toddler years? a. 20/20 b. 20/40 c. 20/50 d. 20/60

ANS: B - 20/40 Visual acuity of 20/40 is considered acceptable during the toddler years.

To avoid a fall from a crib, the nurse recommends to parents that their toddler should sleep in a bed rather than a crib when reaching what height? a. 30 in b. 35 in c. 40 in d. 45 in

ANS: B - 35 in When children reach a height of 89 cm (35 in), they should sleep in a bed rather than a crib

What is the best age to introduce solid food into an infant's diet? a. 2 to 3 months b. 4 to 6 months c. When birth weight has tripled d. When tooth eruption has started

ANS: B - 4 to 6 months Physiologically and developmentally, 4- to 6-month-old infants are in a transition period. The extrusion reflex has disappeared, and swallowing is a more coordinated process. In addition, the gastrointestinal tract has matured sufficiently to handle more complex nutrients and is less sensitive to potentially allergenic food. Infants of this age will try to help during feeding. Two to 3 months is too young. The extrusion reflex is strong, and the child will push food out with the tongue. No research indicates that the addition of solid food to a bottle has any benefit. Infant birth weight doubles at 1 year. Solid foods can be started earlier. Tooth eruption can facilitate biting and chewing; most infant foods do not require this ability

The clinic nurse is evaluating a patient with a vaginal infection. The nurse knows that the normal vaginal pH is in which range? a. 3.0 to 4.0 b. 4.0 to 5.0 c. 5.0 to 6.0 d. 6.0 to 7.0

ANS: B - 4.0 to 5.0 Normal vaginal secretions are acidic, with a pH range of 4.0 to 5.0.

The development of sexual orientation during adolescence is what? a. Inflexible b. A developmental process c. Differs for boys and girls d. Proceeds in a defined sequence

ANS: B - A developmental process The development of sexual orientation as a part of sexual identity includes several developmental milestones during late childhood and throughout adolescence. The sequence and time spent in phases are different for each individual. Boys and girls pass through the same developmental milestones.

The nurse is notified that a 9-year-old boy with nephrotic syndrome is being admitted. Only semiprivate rooms are available. What roommate should be best to select? a. A 10-year-old girl with pneumonia b. An 8-year-old boy with a fractured femur c. A 10-year-old boy with a ruptured appendix d. A 9-year-old girl with congenital heart disease

ANS: B - An 8-year-old boy with a fractured femur An 8-year-old boy with a fractured femur would be the best choice for a roommate. The boys are similar in age. The child with nephrotic syndrome most likely will be on immunosuppressive agents and susceptible to infection. The child with a fractured femur is not infectious. A girl should not be a good roommate for a school-age boy. In addition, the 10-year-old girl with pneumonia and the 10-year-old boy with a ruptured appendix have infections and could pose a risk for the child with nephrotic syndrome.

An adolescent girl tells the nurse that she is very suicidal. The nurse asks her if she has a specific plan. How should asking about a specific plan be viewed? a. Not a critical part of the assessment b. An appropriate part of the assessment c. Suggesting that adolescent needs a plan d. Encouraging adolescent to devise a plan

ANS: B - An appropriate part of the assessment Routine health assessments of adolescents should include questions that assess the presence of suicidal ideation or intent. Questions such as "Have you ever developed a plan to hurt yourself or kill yourself?" should be part of that assessment. Adolescents who express suicidal feelings and have a specific plan are at particular risk and require further assessment and constant monitoring. The information about having a plan is an essential part of the assessment and greatly affects the treatment plan.

The parents of a 4-month-old infant cannot visit except on weekends. What action by the nurse indicates an understanding of the emotional needs of a young infant? a. Place her in a room away from other children. b. Assign her to the same nurse as much as possible. c. Tell the parents that frequent visiting is unnecessary. d. Assign her to different nurses so she will have varied contacts.

ANS: B - Assign her to the same nurse as much as possible. The infant is developing a sense of trust. This is accomplished by the consistent, loving care of a nurturing person. If the parents are unable to visit, then the same staff nurses should be used as much as possible. Placing her in a room away from other children would isolate the child. The parents should be encouraged to visit. The nurse should describe how the staff will care for the infant in their absence.

Which one of the following strategies might be recommended for an infant with failure to thrive (FTT) to increase caloric intake? a. Vary the schedule for routine activities on a daily basis. b. Be persistent through 10 to 15 minutes of food refusal. c. Avoid solids until after the bottle is well accepted. d. Use developmental stimulation by a specialist during feedings.

ANS: B - Be persistent through 10 to 15 minutes of food refusal. Calm perseverance through 10 to 15 minutes of food refusal will eventually diminish negative behavior. Children with FTT need a structured routine to help establish rhythmicity in their activities of daily living. Many children with FTT are fed exclusively from a bottle. Solids should be fed first. Stimulation is reduced during mealtimes to maintain the focus on eating

What aspects of cognition develop during adolescence? a. Ability to see things from the point of view of another b. Capability of using a future time perspective c. Capability of placing things in a sensible and logical order d. Progress from making judgments based on what they see to making judgments based on what they reason

ANS: B - Capability of using a future time perspective Adolescents are no longer restricted to the real and actual. They also are concerned with the possible; they think beyond the present. During concrete operations (between ages 7 and 11 years), children exhibit thought processes that enable them to see things from the point of view of another, place things in a sensible and logical order, and progress from making judgments based on what they see to making judgments based on what they reason.

It is important that women with anogenital warts caused by the human papillomavirus (HPV) receive adequate treatment because this sexually transmitted infection increases the risk of what? a. Gonorrhea b. Cervical cancer c. Chlamydial infection d. Urinary tract infection

ANS: B - Cervical cancer Infection with HPV is associated with cervical dysplasia and cervical cancer. A vaccine has been developed and is recommended for young women.

What is the most common form of child maltreatment? a. Sexual abuse b. Child neglect c. Physical abuse d. Emotional abuse

ANS: B - Child neglect Child neglect, which is characterized by the failure to provide for the child's basic needs, is the most common form of child maltreatment. Sexual abuse, physical abuse, and emotional abuse are individually not as common as neglect.

What is descriptive of the social development of school-age children? a. Identification with peers is minimum. b. Children frequently have "best friends." c. Boys and girls play equally with each other. d. Peer approval is not yet an influence for the child to conform.

ANS: B - Children frequently have "best friends." Identification with peers is a strong influence in children's gaining independence from parents. Interaction among peers leads to the formation of close friendships with same-sex peers—"best friends." Daily relationships with age mates in the school setting provide important social interactions for school-age children. During the later school years, groups are composed predominantly of children of the same sex. Conforming to the rules of the peer group provides children with a sense of security and relieves them of the responsibility of making decisions.

A 4-year-old boy is hospitalized with a serious bacterial infection. He tells the nurse that he is sick because he was "bad." What is the nurse's best interpretation of this comment? a. Sign of stress b. Common at this age c. Suggestive of maladaptation d. Suggestive of excessive discipline at home

ANS: B - Common at this age Preschoolers cannot understand the cause and effect of illness. Their egocentrism makes them think they are directly responsible for events, making them feel guilt for things outside of their control. Children of this age react to stress by regressing developmentally or acting out. Maladaptation is unlikely. This comment does not imply excessive discipline at home.

In terms of fine motor development, what should the 3-year-old child be expected to do? a. Tie shoelaces. b. Copy (draw) a circle. c. Use scissors or a pencil very well. d. Draw a person with seven to nine parts.

ANS: B - Copy (draw) a circle. Three-year-old children are able to accomplish the fine motor skill of copying (drawing) a circle. The ability to tie shoelaces, to use scissors or a pencil very well, and to draw a person with seven to nine parts are fine motor skills of 5-year-old children.

The nurse is caring for a 10-year-old child during a long hospitalization. What intervention should the nurse include in the care plan to minimize loss of control and autonomy during the hospitalization? a. Allow the child to skip morning self-care activities to watch a favorite television program. b. Create a calendar with special events such as a visit from a friend to maintain a routine. c. Allow the child to sleep later in the morning and go to bed later at night to promote control. d. Create a restrictive environment so the child feels in control of sensory stimulation.

ANS: B - Create a calendar with special events such as a visit from a friend to maintain a routine. School-age children may feel an overwhelming loss of control and autonomy during a longer hospitalization. One intervention to minimize this loss of control is to create a calendar with planned special events such as a visit from a friend. Maintaining the child's daily routine is another intervention to minimize the sense of loss of control; allowing the child to skip morning self-care activities, sleep later, or stay up later would work against this goal. Environments should be as nonrestrictive as possible to allow the child freedom to move about, thus allowing a sense of autonomy.

A father calls the clinic because he found his young daughter squirting Visine eyedrops into her mouth. What is the most appropriate nursing action? a. Reassure the father that Visine is harmless. b. Direct him to seek immediate medical treatment. c. Recommend inducing vomiting with ipecac. d. Advise him to dilute Visine by giving his daughter several glasses of water to drink.

ANS: B - Direct him to seek immediate medical treatment. Visine is a sympathomimetic and if ingested may cause serious consequences. Medical treatment is necessary. Inducing vomiting is no longer recommended for ingestions. Dilution will not decrease risk.

The mother of a 6-month-old infant has returned to work and is expressing breast milk to be frozen. She asks for directions on how to safely thaw the breast milk in the microwave. What should the nurse recommend? a. Heat only 10 oz or more. b. Do not thaw or heat breast milk in a microwave oven. c. Always leave the bottle top uncovered to allow heat to escape. d. Shake the bottle vigorously for at least 30 seconds after heating.

ANS: B - Do not thaw or heat breast milk in a microwave oven. Using a microwave oven to thaw or heat breast milk decreases the anti-infective properties of the breast milk, lowers the vitamin C content, and changes the fat content. Breast milk should be thawed overnight in a refrigerator or in a warm water bath. A microwave should not be used. If steam is created, the milk is too hot. The bottle should be inverted several times after defrosting or warming.

The school nurse is providing guidance to families of children who are entering elementary school. What is essential information to include? a. Meet with teachers only at scheduled conferences. b. Encourage growth of a sense of responsibility in children. c. Provide tutoring for children to ensure mastery of material. d. Homework should be done as soon as child comes home from school.

ANS: B - Encourage growth of a sense of responsibility in children By being responsible for school work, children learn to keep promises, meet deadlines, and succeed in their jobs as adults. Parents should meet with the teachers at the beginning of the school year, for scheduled conferences, and whenever information about the child or parental concerns needs to be shared. Tutoring should be provided only in special circumstances in elementary school, such as in response to prolonged absence. The parent should not dictate the study time but should establish guidelines to ensure that homework is done.

Parents are concerned that their 6-year-old son continues to occasionally wet the bed. What does the nurse explain? a. This is likely because of increased stress at home. b. Enuresis usually ceases between 6 and 8 years of age. c. Drug therapy will be prescribed to treat the enuresis. d. Testing will be necessary to determine what type of kidney problem exists.

ANS: B - Enuresis usually ceases between 6 and 8 years of age Further data must be gathered before the diagnosis of enuresis is made. Enuresis is the inappropriate voiding of urine at least twice a week. This child does meet the age criterion, but the parents need to be questioned about and keep a diary on the frequency of events. If the bedwetting is infrequent, parents can be encouraged that the child may grow out of this behavior. Drug therapy will not be prescribed until a more complete evaluation is done. Additional assessment information must be gathered, but at this time, there is no indication of renal disease

The clinic nurse is teaching parents about physiologic anemia that occurs in infants. What statement should the nurse include about the cause of physiologic anemia? a. Maternally derived iron stores are depleted in the first 2 months. b. Fetal hemoglobin results in a shortened survival of red blood cells. c. The production of adult hemoglobin decreases in the first year of life. d. Low levels of fetal hemoglobin depress the production of erythropoietin.

ANS: B - Fetal hemoglobin results in a shortened survival of red blood cells. Fetal hemoglobin results in a shortened survival of red blood cells (RBCs) and thus a decreased number of RBCs. Maternally derived iron stores are present for the first 5 to 6 months results in a shortened survival of RBCs and thus a decreased number of RBCs. High levels of fetal hemoglobin depress the production of erythropoietin, a hormone released by the kidney that stimulates RBC production.

What is a major physical risk for young adolescents during pregnancy? a. Osteoporosis frequently develops. b. Fetopelvic disproportion is a common problem. c. Delivery is usually precipitous in this age group. d. Pregnancy will adversely affect the adolescent's development.

ANS: B - Fetopelvic disproportion is a common problem. Teenagers younger than 15 years of age have increased obstetric risks. Fetopelvic disproportion is one of the most common complications. Osteoporosis occurs later in life and is not related to adolescent pregnancy. Prolonged, not precipitous, labor is common in this age group. Teenage mothers are socially, educationally, psychologically, and economically disadvantaged. Support is necessary because the tasks of motherhood are superimposed on adolescent development tasks.

What statement is true concerning adolescent suicide? a. A sense of hopelessness and despair is a normal part of adolescence. b. Gay and lesbian adolescents are at a particularly high risk for suicide. c. Problem-solving skills are of limited value to the suicidal adolescent d. Previous suicide attempts are not an indication for completed suicides.

ANS: B - Gay and lesbian adolescents are at a particularly high risk for suicide. A significant number of teenage suicides occur among homosexual youths. Gay and lesbian adolescents who live in families or communities that do not accept homosexuality are likely to experience low self-esteem, self-loathing, depression, and hopelessness. Most adolescents do not experience this stage of life as a time of despair. Depressive symptoms, acting-out behaviors, and talk of suicide need to be taken seriously. At-risk teenagers include those who are depressed, have poor problem-solving skills, or use drugs and alcohol. A history of a previous suicide attempt is a serious indicator for possible suicide completion in the future

What is the usual presenting symptom for testicular cancer? a. Solid, painful mass b. Hard, painless mass c. Scrotal swelling and pain d. Epididymis easily palpated

ANS: B - Hard, painless mass The usual presenting symptom for testicular cancer is a heavy, hard, painless mass that is either smooth or nodular and palpated on the testes. Pain is not usually associated with a testicular tumor. Scrotal swelling needs to be evaluated. The epididymis is easily palpated in a normal scrotum.

What is an important consideration for the school nurse who is planning a class on bicycle safety? a. Most bicycle injuries involve collision with an automobile. b. Head injuries are the major causes of bicycle-related fatalities. c. Children should wear a bicycle helmet if they ride on paved streets. d. Children should not ride double unless the bicycle has an extra large seat.

ANS: B - Head injuries are the major causes of bicycle-related fatalities. The most important aspect of bicycle safety is to encourage the rider to use a protective helmet. Head injuries are the major cause of bicycle-related fatalities. Although motor vehicle collisions do cause injuries to bicyclists, most injuries result from falls. The child should always wear a properly fitted helmet approved by the U.S. Consumer Product Safety Commission. Children should not ride double unless it is a tandem bike (built for two).

What is a clinical manifestation of acetaminophen poisoning? a. Hyperpyrexia b. Hepatic involvement c. Severe burning pain in stomach d. Drooling and inability to clear secretions

ANS: B - Hepatic involvement Hepatic involvement is the third stage of acetaminophen poisoning. Hyperpyrexia is a severe elevation in body temperature and is not related to acetaminophen poisoning. Acetaminophen does not cause burning pain in stomach and does not pose an airway threat.

Parents phone the nurse and say that their child just knocked out a permanent tooth. What should the nurse's instructions to the parents include? a. Place the tooth in dry container for transport. b. Hold the tooth by the crown and not by the root area. c. Transport the child and tooth to a dentist within 18 hours. d. Take the child to hospital emergency department if his or her mouth is bleeding.

ANS: B - Hold the tooth by the crown and not by the root area. It is important to avoid touching the root area of the tooth. The tooth should be held by the crown area; rinsed in milk, saline, or running water; and reimplanted as soon as possible. The tooth is kept moist during transport to maintain viability. Cold milk is the most desirable medium for transport. The child needs to be seen by a dentist as soon as possible. Tooth evulsion causes a large amount of bleeding. The child will need to be seen by a dentist because of the loss of a tooth, not the bleeding.

A school-age child has been bitten on the leg by a large snake that may be poisonous. During transport to an emergency facility, what should the care include? a. Apply ice to the snakebite. b. Immobilize the leg with a splint. c. Place a loose tourniquet distal to the bite. d. Apply warm compresses to the snakebite.

ANS: B - Immobilize the leg with a splint. The leg should be immobilized. Ice decreases blood flow to the area, which allows the venom to work more destruction and decreases the effect of antivenin on the natural immune mechanisms. A loose tourniquet is placed proximal, not distal, to the area of the bite to delay the flow of lymph. This can delay movement of the venom into the peripheral circulation. The tourniquet should be applied so that a pulse can be felt distal to the bite. Warmth increases circulation to the area and helps the toxin into the peripheral circulation.

Parents are switching their toddler, who has met the weight requirement, from a rear-facing car seat to a forward-facing seat. The nurse should recommend the parents place the seat where in the car? a. In the front passenger seat b. In the middle of the rear seat c. In the rear seat behind the driver d. In the rear seat behind the passenger

ANS: B - In the middle of the rear seat Children 0 to 3 years of age riding properly restrained in the middle of the backseat have a 43% lower risk of injury than children riding in the outboard (window) seat during a crash.

A child has been admitted to the hospital with a blood lead level of 72 mcg/dL. What treatment should the nurse anticipate? a. Referral to social services b. Initiation of chelation therapy c. Follow-up testing within 1 month d. Aggressive environmental intervention

ANS: B - Initiation of chelation therapy Severe lead toxicity (lead level ?5=70 mcg/dL) requires immediate inpatient chelation treatment. Referral to social service and follow-up in 1 month are prescribed for lead levels of 15 to 19 mcg/dL. Aggressive environmental intervention would be initiated after chelation treatments.

The parent of a 4-year-old boy tells the nurse that the child believes "monsters and bogeymen" are in his bedroom at night. What is the nurse's best suggestion for coping with this problem? a. Let the child sleep with his parents. b. Keep a night light on in the child's bedroom. c. Help the child understand that these fears are illogical. d. Tell the child that monsters and bogeymen do not exist.

ANS: B - Keep a night light on in the child's bedroom. Involve the child in problem solving. A night light shows a child that imaginary creatures do not lurk in the darkness. Letting the child sleep with his parents will not get rid of the fears. A 4-year-old child is in the preconceptual stage and cannot understand logical thought.

The nurse is discussing the management of atopic dermatitis (eczema) with a parent. What should be included? a. Dress infant warmly to prevent chilling. b. Keep the infant's fingernails and toenails cut short and clean. c. Give bubble baths instead of washing lesions with soap. d. Launder clothes in mild detergent; use fabric softener in the rinse.

ANS: B - Keep the infant's fingernails and toenails cut short and clean. The infant's nails should be kept short and clean and have no sharp edges. Gloves or cotton socks can be placed over the child's hands and pinned to the shirt sleeves. Heat and humidity increase perspiration, which can exacerbate the eczema. The child should be dressed properly for the climate. Synthetic material (not wool) should be used for the child's clothing during cold months. Baths are given as prescribed with tepid water, and emollients such as Aquaphor, Cetaphil, and Eucerin are applied within 3 minutes. Soap (except as indicated), bubble bath oils, and powders are avoided. Fabric softener should be avoided because of the irritant effects of some of its components.

A school-age child has begun to sleepwalk. What does the nurse advise the parents to perform? a. Wake the child and help determine what is wrong. b. Leave the child alone unless he or she is in danger of harming him- or herself or others. c. Arrange for psychologic evaluation to identify the cause of stress. d. Keep the child awake later in the evening to ensure sufficient tiredness for a full night of sleep.

ANS: B - Leave the child alone unless he or she is in danger of harming him- or herself or others. Sleepwalking is usually self-limiting and requires no treatment. The child usually moves about restlessly and then returns to bed. Usually the actions are repetitive and clumsy. The child should not be awakened unless in danger. If there is a need to awaken the child, it should be done by calling the child's name to gradually bring to a state of alertness. Some children, who are usually well behaved and tend to repress feelings, may sleepwalk because of strong emotions. These children usually respond to relaxation techniques before bedtime. If a child is overly fatigued, sleepwalking can increase.

What is an important nursing responsibility when dealing with a family experiencing the loss of an infant from sudden infant death syndrome (SIDS)? a. Discourage the parents from making a last visit with the infant. b. Make a follow-up home visit to the parents as soon as possible after the child's death. c. Explain how SIDS could have been predicted and prevented. d. Interview the parents in depth concerning the circumstances surrounding the child's death.

ANS: B - Make a follow-up home visit to the parents as soon as possible after the child's death. A competent, qualified professional should visit the family at home as soon as possible after the death. Printed information about SIDS should be provided to the family. Parents should be allowed and encouraged to make a last visit with their child. SIDS cannot always be prevented or predicted, but parents can take steps to reduce the risk (e.g., supine sleeping, removing blankets and pillows from the crib, and not smoking). Discussions about the cause only increase parental guilt. The parents should be asked only factual questions to determine the cause of death.

What is true concerning masturbation during adolescence? a. Homosexuality is encouraged by the practice of masturbation. b. Many girls do not begin masturbation until after they have intercourse. c. Masturbation at an early age leads to sexual intercourse at an earlier age. d. Development of intimate relationships is delayed when masturbation is regularly practiced.

ANS: B - Many girls do not begin masturbation until after they have intercourse. The age of first masturbation for girls is variable. Some begin masturbating in early adolescence; many do not begin until after they have had intercourse. Boys typically begin masturbation in early adolescence. Masturbation provides an opportunity for self-exploration. Both heterosexual and homosexual youth use masturbation. It does not affect the development of intimacy

An awake, alert 4-year-old child has just arrived at the emergency department after an ingestion of aspirin at home. The practitioner has ordered activated charcoal. The nurse administers charcoal in which manner? a. Giving half of the solution and then repeating the other half in 1 hour b. Mixing with a flavorful beverage in an opaque container with a straw c. Serving it in a clear plastic cup so the child can see how much has been drunk d. Administering it through a nasogastric tube because the child will not drink it because of the taste

ANS: B - Mixing with a flavorful beverage in an opaque container with a straw Although activated charcoal can be mixed with a flavorful sugar-free beverage, it will be black and resemble mud. When it is served in an opaque container, the child will not have any preconceived ideas about its being distasteful. The ability to see the charcoal solution may affect the child's desire to drink the solution. The child should be encouraged to drink the solution all at once. The nasogastric tube would be traumatic. It should be used only in children who cannot be cooperative or those without a gag reflex.

The nurse is caring for an adolescent brought to the hospital with acute drug toxicity. Cocaine is believed to be the drug involved. Data collection by the nurse should include what information? a. Drug's actual content b. Mode of administration c. Adolescent's level of interest in rehabilitation d. Function the drug plays in the adolescent's life

ANS: B - Mode of administration Cocaine is available in two forms, water soluble and non-water soluble, and can be administered through multiple routes. For treatment purposes, it is essential to know the type of drug and route of administration. Because cocaine is a street drug, the actual content usually cannot be identified. The adolescent's level of interest in rehabilitation and the function that drug plays in the adolescent's life are concerns to be addressed after the initial emergency treatment is instituted.

A 13-year-old child with cystic fibrosis (CF) is a frequent patient on the pediatric unit. This admission, she is sleeping during the daytime and unable to sleep at night. What should be a beneficial strategy for this child? a. Administer prescribed sedative at night to aid in sleep. b. Negotiate a daily schedule that incorporates hospital routine, therapy, and free time. c. Have the practitioner speak with the child about the need for rest when receiving therapy for CF. d. Arrange a consult with the social worker to determine whether issues at home are interfering with her care.

ANS: B - Negotiate a daily schedule that incorporates hospital routine, therapy, and free time. Children's response to the disruption of routine during hospitalization is demonstrated in eating, sleeping, and other activities of daily living. The lack of structure is allowing the child to sleep during the day, rather than at night. Most likely the lack of schedule is the problem. The nurse and child can plan a schedule that incorporates all necessary activities, including medications, mealtimes, homework, and patient care procedures. The schedule can then be posted so the child has a ready reference. Sedatives are not usually used with children. The child has a chronic illness and most likely knows the importance of rest. The parents and child can be questioned about changes at home since the last hospitalization.

Parents tell the nurse they found their 3-year-old daughter and a male cousin of the same age inspecting each other closely as they used the bathroom. What is the most appropriate recommendation for the nurse to make? a. Punish the children so this behavior stops. b. Neither condone nor condemn the curiosity. c. Get counseling for this unusual and dangerous behavior. d. Allow the children unrestricted permission to satisfy this curiosity.

ANS: B - Neither condone nor condemn the curiosity. Three-year-old children become aware of anatomic differences and are concerned about how the other sex "works." Such exploration should not be condoned or condemned. Children should not be punished for this normal exploration. This is age appropriate and not dangerous behavior. Encouraging the children to ask their parents questions and redirecting their activity is more appropriate than giving permission.

A toddler has a deep laceration contaminated with dirt and sand. Before closing the wound, the nurse should irrigate with what solution? a. Alcohol b. Normal saline c. Povidone-iodine d. Hydrogen peroxide

ANS: B - Normal saline Normal saline is the only acceptable fluid for irrigation listed. The nurse should cleanse the wound with a forced stream of normal saline or water. Alcohol is not used for wound irrigation. Povidone-iodine is contraindicated for cleansing fresh, open wounds. Hydrogen peroxide can cause formation of subcutaneous gas when applied under pressure

An adolescent girl is brought to the hospital emergency department by her parents after being raped. The girl is calm and controlled throughout the interview and examination. The nurse should recognize this behavior is what? a. A sign that a rape has not actually occurred b. One of a variety of behaviors normally seen in rape victims c. Indicative of a higher than usual level of maturity in the adolescent d. Suggestive that the adolescent had severe emotional problems before the rape occurred

ANS: B - One of a variety of behaviors normally seen in rape victims Rape victims display a wide range of behaviors. A controlled manner may be an attempt to maintain composure while hiding the inner turmoil. Because the observed behavior is within the range of expected behavior, there are no data to indicate that a rape has not actually occurred, that the adolescent is unusually mature, or that she had severe emotional problems before the rape occurred.

The nurse is teaching a class on obesity prevention to parents in the community. What is a contributing factor to childhood obesity? a. Birth weight b. Parental overweight c. Age at the onset of puberty d. Asian ethnic background

ANS: B - Parental overweight There is a high correlation between parental adiposity and childhood adiposity. Obese children do not have higher birth weights than nonobese children. Early menarche is associated with obesity, but the age of puberty is not a contributing factor. African Americans and Hispanics have disproportionately high percentages of overweight individuals, but Asians do not.

After the introduction of the Back to Sleep campaign in 1992, an increased incidence has been noted of which pediatric issues? a. Sudden infant death syndrome (SIDS) b. Plagiocephaly c. Failure to thrive d. Apnea of infancy

ANS: B - Plagiocephaly Plagiocephaly is a misshapen head caused by the prolonged pressure on one side of the skull. If that side becomes misshapen, facial asymmetry may result. SIDS has decreased by more than 40% with the introduction of the Back to Sleep campaign. Apnea of infancy and failure to thrive are unrelated to the Back to Sleep campaign.

A bottle-fed infant has been diagnosed with cow's milk allergy. Which formula should the nurse expect to be prescribed for the infant? a. Similac b. Pregestimil c. Enfamil with iron d. Gerber Good Start

ANS: B - Pregestimil For infants with cow's milk allergy, the formula will be changed to a casein hydrolysate milk formula (Pregestimil, Nutramigen, or Alimentum) in which the protein has been broken down into its amino acids through enzymatic hydrolysis. Similac, Enfamil with iron, and Gerber Good Start are cow's milk-based formulas.

The parent of an 8.2-kg (18-lb) 9-month-old infant is borrowing a federally approved car seat from the clinic. The nurse should explain that the safest way to put in the car seat is what? a. Front facing in back seat b. Rear facing in back seat c. Front facing in front seat with air bag on passenger side d. Rear facing in front seat if an air bag is on the passenger side

ANS: B - Rear facing in back seat A rear-facing car seat provides the best protection for an infant's disproportionately heavy head and weak neck. The middle of the back seat is the safest position for the child. Severe injuries and deaths in children have occurred from air bags deploying on impact in the front passenger seat.

A 15-year-old girl tells the school nurse that she has not started to menstruate yet. Onset of secondary sexual characteristics was about 2 1/2 years ago. The nurse should take which action? a. Explain that this is not unusual. b. Refer the adolescent for an evaluation. c. Make an assumption that the adolescent is pregnant. d. Suggest that the adolescent stop exercising until menarche occurs.

ANS: B - Refer the adolescent for an evaluation. A referral is indicated. Menarche should follow the onset of secondary sexual development within 2 1/2 years. A careful examination is done to reveal any physical abnormalities, signs of androgen excess, and congenital defects of the genital tract. The lack of the onset of menstruation at this age is a potential indication of a physical problem. Assuming that the adolescent is pregnant is inappropriate. The nurse does not have any indication that the adolescent is sexually active. The amount of exercise should be assessed before suggesting that the adolescent stop exercising until menarche occurs.

What do nursing responsibilities regarding weight gain for an adolescent with anorexia nervosa include? a. Administer tube feedings until target weight is achieved. b. Restore body weight to within 10% of the adolescent's ideal weight. c. Encourage continuation of strenuous exercise as long as adolescent is not losing weight. d. Facilitate as rapid a weight gain as possible with a high-calorie diet.

ANS: B - Restore body weight to within 10% of the adolescent's ideal weight. The restoration of body weight to a target weight or endpoint within 10% of ideal body weight is one of the main goals of therapy. Strenuous exercise is avoided as part of the need to modify behaviors. Tube feedings are intrusive and are avoided. They should only be used when other measures have failed. Weight restoration is accomplished slowly. The goal is 1 kg/wk to avoid the risk of metabolic and cardiac problems. Slow weight gain can minimize anxiety and depression

What is a significant secondary prevention nursing activity for lead poisoning? a. Chelation therapy b. Screening children for blood lead levels c. Removing lead-based paint from older homes d. Questioning parents about ethnic remedies containing lead

ANS: B - Screening children for blood lead levels Screening children for lead poisoning is an important secondary prevention activity. Screening does not prevent the initial exposure of the child to lead. It can lead to identification and treatment of children who are exposed. Chelation therapy is treatment, not prevention. Removing lead-based paints from older homes before children are affected is primary prevention. Questioning parents about ethnic remedies containing lead is part of the assessment to determine the potential source of lead.

A 3-month-old infant dies shortly after arrival to the emergency department. The infant has subdural and retinal hemorrhages but no external signs of trauma. What should the nurse suspect? a. Unintentional injury b. Shaken baby syndrome c. Congenital neurologic problem d. Sudden infant death syndrome (SIDS)

ANS: B - Shaken baby syndrome Shaken baby syndrome causes internal bleeding but may have no external signs. Unintentional injury would not cause these injuries. With unintentional injuries, external signs are usually present. Congenital neurologic problems would usually have signs of abnormal neurologic anatomy. SIDS does not usually have identifiable injuries.

A child age 4 1/2 years sometimes wakes her parents up at night screaming, thrashing, sweating, and apparently frightened, yet she is not aware of her parents' presence when they check on her. She lies down and sleeps without any parental intervention. This is most likely what? a. Nightmare b. Sleep terror c. Sleep apnea d. Seizure activity

ANS: B - Sleep terror This is a description of a sleep terror. The child is observed during the episode and not disturbed unless there is a possibility of injury. A child who awakes from a nightmare is distressed. She is aware of and reassured by the parent's presence. This is not the case with sleep apnea. This behavior is not indicative of seizure activity

The American Academy of Pediatrics (AAP) recommends that children younger than the age of 16 years be prohibited from participating in what? a. Skateboarding b. Snowmobiling c. Trampoline use d. Horseback riding

ANS: B - Snowmobiling The AAP views the use of snowmobiles and all-terrain vehicles as major health hazards for children. This group opposes the use of these vehicles by children younger than 16 years of age. The AAP recommends that children younger than the age of 10 years not use skateboards without parental supervision. Protective gear is always suggested. Trampoline use has increased along with injuries. Adults should supervise use. Horseback riding injuries are also a source of concern. Parents should determine the instructor's safety record with students.

What is the most significant factor in distinguishing those who commit suicide from those who make suicidal attempts or threats? a. Level of stress b. Social isolation c. Degree of depression d. Desire to punish others

ANS: B - Social isolation Social isolation is a significant factor in distinguishing adolescents who will kill themselves from those who will not. It is also more characteristic of those who complete suicide versus those who make attempts or threats. Although the level of stress, the degree of depression, and the desire to punish others are contributing factors in suicide, they are not the most significant factor in distinguishing those who complete suicide from those who attempt suicide.

Which is an appropriate recommendation in preventing tooth decay in young children? a. Substitute raisins for candy. b. Substitute sugarless gum for regular gum. c. Use honey or molasses instead of refined sugar. d. When sweets are to be eaten, select a time not during meals.

ANS: B - Substitute sugarless gum for regular gum. Regular gum has high sugar content. When the child chews gum, the sugar is in prolonged contact with the teeth. Sugarless gum is less cariogenic than regular gum. Raisins, honey, and molasses are highly cariogenic and should be avoided. Sweets should be consumed with meals so that the teeth can be cleaned afterward. This decreases the amount of time that the sugar is in contact with the teeth.

When teaching injury prevention during the school-age years, what should the nurse include? a. Teach children about the need to fear strangers. b. Teach basic rules of water safety. c. Avoid letting children cook in microwave ovens. d. Caution children against engaging in competitive sports.

ANS: B - Teach basic rules of water safety. Water safety instruction is an important component of injury prevention at this age. The child should be taught to swim, select safe and supervised places to swim, swim with a companion, check sufficient water depth for diving, and use an approved flotation device. Teach stranger safety, not fear of strangers. This includes telling the child not to go with strangers, not to wear personalized clothing in public places, to tell parents if anyone makes child feel uncomfortable, and to say no in uncomfortable situations. Teach the child safe cooking. Caution against engaging in dangerous sports such as jumping on trampolines.

The nurse is interviewing the father of a 10-month-old girl. The child is playing on the floor when she notices an electrical outlet and reaches up to touch it. Her father says "no" firmly and moves her away from the outlet. The nurse should use this opportunity to teach the father what? a. That the child should be given a time-out b. That the child is old enough to understand the word "no" c. That the child will learn safety issues better if she is spanked d. That the child should already know that electrical outlets are dangerous

ANS: B - That the child is old enough to understand the word "no" By age 10 months, children are able to associate meaning with words. The father is using both verbal and physical cues to alert the child to dangerous situations. A time-out is not appropriate. The child is just learning about the environment. Physical discipline should be avoided. The 10-month-old child is too young to understand the purpose of an electrical outlet.

The nurse is explaining the purpose of using a vacuum-assisted closure (VAC) device to assist in the healing of a wound. What should the nurse explain as the purpose of using a VAC device? a. The device will decrease capillary flow. b. The device applies gentle continuous suction. c. The device will allow the wound to remain open. d. The device will prevent the formation of granulation tissue.

ANS: B - The device applies gentle continuous suction. A VAC device uses a technique that involves placing a foam dressing into the wound, covering it with an occlusive dressing, and applying gentle continuous suction. The negative pressure of the suction is applied from the foam dressing to the wound surfaces. The mechanical force removes excess fluids from the wound, stimulates formation of granulation tissue, restores capillary flow, and fosters closure of the wound.

The parents of a 7-year-old boy tell the nurse that lately he has been cruel to their family pets and actually caused physical harm. The nurse's recommendation should be based on remembering what? a. This is an expected behavior at this age. b. This is a warning sign of a serious problem. c. This is harmless venting of anger and frustration. d. This is common in children who are physically abused.

ANS: B - This is a warning sign of a serious problem. Cruelty to family pets is not an expected behavior. Hurting animals can be one of the earliest symptoms of a conduct disorder. Abusing animals does not dissipate violent emotions; rather, the acts may fuel the abusive behaviors. Referral for evaluation is essential. This behavior may be seen in emotional abuse or neglect, not physical abuse

A 10-year-old girl needs to have another intravenous (IV) line started. She keeps telling the nurse, "Wait a minute," and, "I'm not ready." How should the nurse interpret this behavior? a. IV insertions are viewed as punishment. b. This is expected behavior for a school-age child. c. Protesting like this is usually not seen past the preschool years. d. The child has successfully manipulated the nurse in the past.

ANS: B - This is expected behavior for a school-age child. This school-age child is attempting to maintain some control over the hospital experience. The nurse should provide the girl with structured choices about when the IV line will be inserted. Preschoolers can view procedures as punishment; this is not typical behavior of a preschool-age child.

What statement is true about gonorrhea? a. It is caused by Treponema pallidum. b. Treatment of all sexual contacts is essential. c. Topical application of medication to the lesions is necessary. d. Therapeutic management includes multidose administration of penicillin.

ANS: B - Treatment of all sexual contacts is essential. The treatment plan should include finding and treating all sexual partners. Gonorrhea is caused by Neisseria gonorrhoeae. Syphilis is caused by T. pallidum. Systemic therapy is necessary to treat this disease. Primary treatment is with different antibiotics because of N. gonorrhoeae's resistance to penicillin.

A child is admitted to the hospital with lesions on his abdomen that appear like cigarette burns. What should accurate documentation by the nurse include? a. Two unhealed lesions are on the child's abdomen. b. Two round 4-mm lesions are on the child's lower abdomen. c. Two round symmetrical lesions are on the child's lower abdomen. d. Two round lesions on the child's abdomen that appear to be cigarette burns.

ANS: B - Two round 4-mm lesions are on the child's lower abdomen. Burn documentation should include the location, pattern, demarcation lines, and presence of eschar or blisters. The option that includes the size of the lesions is the most accurate.

In terms of cognitive development, a 5-year-old child should be expected to do which? a. Think abstractly. b. Use magical thinking. c. Understand conservation of matter. d. Understand another person's perspective.

ANS: B - Use magical thinking. Magical thinking is believing that thoughts can cause events. An example is thinking of the death of a parent might cause it to happen. Abstract thought does not develop until the school-age years. The concept of conservation is the cognitive task of school-age children, ages 5 to 7 years. A 5-year-old child cannot understand another person's perspective.

A 6-year-old boy with very fair skin will be joining his family during a beach vacation. What should the nurse recommend? a. Keep him off the beach during the daytime hours. b. Use sunscreen with an SPF of at least 15 and reapply it every 2 to 3 hours. c. Apply a topical sunscreen product with an SPF of 30 in the morning. d. Dress him in long pants and long-sleeved shirt and keep him under a beach umbrella.

ANS: B - Use sunscreen with an SPF of at least 15 and reapply it every 2 to 3 hours. A sunscreen with an SPF (sun protection factor) of at least 15 is recommended. The sunscreen should be reapplied every 2 to 3 hours and after the child is in the water or sweating excessively. During a beach vacation, avoiding the beach during daytime hours is impractical. The highest risk of sun exposure is from 10 AM to 3 PM. Sunlight exposure should be limited during this time. An SPF of 30 is good, but reapplying it is necessary every 2 to 3 hours and when the child gets wet. Long pants and a shirt are impractical. The beach umbrella can be used with the sunscreen to limit exposure to the sun.

Which statement is correct about toilet training? a. Bladder training is usually accomplished before bowel training. b. Wanting to please the parent helps motivate the child to use the toilet. c. Watching older siblings use the toilet confuses the child. d. Children must be forced to sit on the toilet when first learning.

ANS: B - Wanting to please the parent helps motivate the child to use the toilet. Voluntary control of the anal and urethral sphincters is achieved sometime after the child is walking. The child must be able to recognize the urge to let go and to hold on. The child must want to please the parent by holding on rather than pleasing him- or herself by letting go. Bowel training precedes bladder training. Watching older siblings provides role modeling and facilitates imitation for the toddler. The child should be introduced to the potty chair or toilet in a nonthreatening manner.

The nurse is teaching a class on nutrition to a group of parents of 10- and 11-year-old children. What statement by one of the parents indicates a correct understanding of the teaching? a. "My child does not need to eat a variety of foods, just his favorite food groups." b. "My child can add salt and sugar to foods to make them taste better." c. "I will serve foods that are low in saturated fat and cholesterol." d. "I will continue to serve red meat three times per week for extra iron."

ANS: C - "I will serve foods that are low in saturated fat and cholesterol." School-age children should be eating foods that are low in saturated fat and cholesterol to prevent long-term consequences. The child's diet should include a variety of foods, include moderate amounts of extra salt and sugar, emphasize consumption of lean protein (chicken and pork), and limit red meat

Parents ask the nurse, "How should we deal with our toddler's regression since our new baby has come home?" The nurse should give the parents which response? a. "Introduce new areas of learning." b. "Use time-out as punishment when regression occurs." c. "Ignore the behavior and praise appropriate behavior." d. "Explain to the toddler that the behavior is not acceptable."

ANS: C - "Ignore the behavior and praise appropriate behavior." When regression does occur, the best approach is to ignore it while praising existing patterns of appropriate behavior. It is advisable not to introduce new areas of learning when an additional crisis is present or expected, such as beginning toilet training shortly before a sibling is born or during a brief hospitalization. Time-out should not be used as a punishment, and the toddler does not have the cognitive ability to understand an explanation that the behavior is not acceptable.

A parent taking a preschool child to school on the first day asks the nurse, "What do I do if my child wants me to stay?" What is an appropriate response by the nurse? a. "It is better if you do not stay." b. "It is best to stay and participate in the activities." c. "It is OK to stay part of the first day, but be inconspicuous." d. "It would be better to have a good friend take your child to class the first day."

ANS: C - "It is OK to stay part of the first day, but be inconspicuous." On the first day of preschool, in some instances, it is helpful for parents to remain for at least part of the first day until the child is comfortable. If parents stay, they should be available to the child but inconspicuous. It would not be appropriate not to stay, to have someone else take the child to school, or to stay and participate in activities.

The school nurse is teaching an adolescent about social networking and texting on phones. What statement by the adolescent indicates a need for further teaching? a. "Social networking can help me develop interpersonal skills." b. "I will have an opportunity to interact with people like myself." c. "My text messaging during class time in school will not cause any disruption." d. "I should be cautious, as the online environment can create opportunities for cyberbullying."

ANS: C - "My text messaging during class time in school will not cause any disruption." Internet chatrooms and social networking sites have created a more public arena for trying out identities and developing interpersonal skills with a wider network of people, occasionally with anonymity. This can create opportunities for young people who have a limited access to friends (because of rural location, shyness, or rare chronic conditions) to interact with people like themselves. Both the online and text environment can create opportunities for cyberbullying, in which teens engage in insults, harassment, and publicly humiliating statements online or on cell phones. Text messaging and instant messaging via cell phones has become a common activity and can sometimes be disruptive during school. If the adolescent indicates it will not be disruptive, further teaching is needed.

The parent of 2-week-old infant asks the nurse if fluoride supplements are necessary because the infant is exclusively breastfed. What is the nurse's best response? a. "The infant needs to begin taking them now." b. "Supplements are not needed if you drink fluoridated water." c. "The infant may need to begin taking them at age 6 months." d. "The infant can have infant cereal mixed with fluoridated water instead of supplements."

ANS: C - "The infant may need to begin taking them at age 6 months." Fluoride supplementation is recommended by the American Academy of Pediatrics beginning at age 6 months if the child is not drinking adequate amounts of fluoridated water. Supplementation is not recommended before age 6 months regardless of whether the mother drinks fluoridated water. Infant cereal is not recommended at 2 weeks of age.

The nurse is explaining to an adolescent the rationale for administering a Tdap (tetanus, diphtheria, acellular pertussis) vaccine 3 years after the last Td (tetanus) booster. What should the nurse tell the adolescent? a. "It is time for a booster vaccine." b. "It is past the time for a booster vaccine." c. "This vaccine will provide pertussis immunity." d. "This vaccine will be the last booster you will need."

ANS: C - "This vaccine will provide pertussis immunity." When the Tdap is used as a booster dose, it may be administered earlier than the previous 5-year interval to provide adequate pertussis immunity (regardless of interval from the last Td dose). It is not time or past time for a booster because they are required every 5 years. Another booster will be needed in 5 years, so it is not the last dose.

The nurse is teaching parents guidelines for feeding their 8-month-old infant with failure to thrive (FTT). Which statement by the parents indicates a need for further teaching? a. "We will continue to use the 24-kcal/oz formula." b. "We will be sure to follow the formula preparation instructions." c. "We will be sure to give our infant at least 8 oz of juice every day." d. "We will be sure to feed our infant according to the written schedule."

ANS: C - "We will be sure to give our infant at least 8 oz of juice every day." Juice intake in infants with FTT should be withheld until adequate weight gain has been achieved with appropriate milk sources; thereafter, no more than 4/oz day of juice should be given. Further teaching is needed if the parents indicate 8 oz of juice is allowed. For infants with FTT, 24-kcal/oz formulas may be provided to increase caloric intake. Because maladaptive feeding practices often contribute to growth failure, parents should follow specific step-by-step directions for formula preparation, as well as a written schedule of feeding times. Statements by the parents indicating they will use a 24-kcal/oz formula, follow directions for formula preparation, and feed their infant on schedule are accurate statements.

A female school-age child asks the school nurse, "How many pounds should I expect to gain in a year?" The nurse should give which response? a. "You will gain about 2.4 to 4.6 lb per year" b. "You will gain about 3.4 to 5.6 lb per year." c. "You will gain about 4.4 to 6.6 lb per year." d. "You will gain about 5.5 to 7.6 lb per year."

ANS: C - "You will gain about 4.4 to 6.6 lb per year." Between the ages of 6 and 12 years, children will almost double in weight, increasing 2 to 3 kg (4.4 to 6.6 lb) per year.

At which age should the nurse expect most infants to begin to say "mama" and "dada" with meaning? a. 4 months b. 6 months c. 10 months d. 14 months

ANS: C - 10 months Beginning at about age 10 months, an infant is able to ascribe meaning to the words "mama" and "dada." Four to 6 months is too young for this behavior to develop. At 14 months, the child should be able to attach meaning to these words. By age 1 year, the child can say three to five words with meaning and understand as many as 100 words.

By which age should the nurse expect that an infant will be able to pull to a standing position? a. 5 to 6 months b. 7 to 8 months c. 11 to 12 months d. 14 to 15 months

ANS: C - 11 to 12 months Most infants can pull themselves to a standing position at age 9 months. Infants who are not able to pull themselves to standing by age 11 to 12 months should be further evaluated for developmental dysplasia of the hip. At 6 months, infants have just obtained coordination of arms and legs. By age 8 months, infants can bear full weight on their legs.

At what age is it safe to give infants whole milk instead of commercial infant formula? a. 6 months b. 9 months c. 12 months d. 18 months

ANS: C - 12 months The American Academy of Pediatrics does not recommend the use of cow's milk for children younger than 12 months. At 6 and 9 months, the infant should be receiving breast milk or iron-fortified commercial infant formula. At age 18 months, milk and formula are supplemented with solid foods, water, and some fruit juices.

The school nurse is teaching female school-age children about the average age of puberty. What is the average age of puberty for girls? a. 10 years b. 11 years c. 12 years d. 13 years

ANS: C - 12 years The average age of puberty is 12 years in girls

The school nurse recognizes that pubertal delay in boys is considered if no enlargement of the testes or scrotal changes have occurred by what age? a. 11 1/2 to 12 years b. 12 1/2 to 13 years c. 13 1/2 to 14 years d. 14 1/2 to 15 years

ANS: C - 13 1/2 to 14 years Concerns about pubertal delay should be considered for boys who exhibit no enlargement of the testes or scrotal changes by ages 13 1/2 to 14 years or if genital growth is not complete 4 years after the testicles begin to enlarge.

The school nurse is teaching male school-age children about the average age of puberty. What is the average age of puberty for boys? a. 12 years b. 13 years c. 14 years d. 15 years

ANS: C - 14 years The average age of puberty is 14 years in boys. Boys experience little sexual maturation during preadolescence

An infant weighed 8 lb at birth and was 18 inches in length. What weight and length should the infant be at 5 months of age? a. 12 lb, 20 inches b. 14 lb, 21.5 inches c. 16 lb, 23 inches d. 18 lb, 24.5 inches

ANS: C - 16 lb, 23 inches Infants gain 680 g (1.5 lb) per month until age 5 months, when the birth weight has at least doubled. Height increases by 2.5 cm (1 inch) per month during the first 6 months. Therefore, at 5 months the infant should weigh 16 lb and be 23 inches in length.

At which age does an infant start to recognize familiar faces and objects, such as his or her own hand? a. 1 month b. 2 months c. 3 months d. 4 months

ANS: C - 3 months The child can recognize familiar objects at approximately age 3 months. For the first 2 months of life, infants watch and observe their surroundings. The 4-month-old infant is beginning to develop hand-eye coordination

At which age do most infants begin to fear strangers? a. 2 months b. 4 months c. 6 months d. 12 months

ANS: C - 6 months Between ages 6 and 8 months, fear of strangers and stranger anxiety become prominent and are related to infants' ability to discriminate between familiar and unfamiliar people. At 2 months, infants are just beginning to respond differentially to their mothers. The infant at age 4 months is beginning the process of separation-individuation, which involves recognizing the self and mother as separate beings. Twelve months is too late; the infant requires referral for evaluation if he or she does not fear strangers by this age.

After a treatment plan for acne has been initiated, which time period should the nurse explain to an adolescent before improvement will be seen? a. 2 to 4 weeks b. 4 to 6 weeks c. 6 to 8 weeks d. 8 to 10 weeks

ANS: C - 6 to 8 weeks Inform patients that after a treatment plan for acne has been initiated, it will take 6 to 8 weeks to appreciate improvement in their skin.

At which age can most infants sit steadily unsupported? a. 4 months b. 6 months c. 8 months d. 12 months

ANS: C - 8 months Sitting erect without support is a developmental milestone usually achieved by 8 months. At age 4 months, an infant can sit with support. At age 6 months, the infant will maintain a sitting position if propped. By 10 months, the infant can maneuver from a prone to a sitting position

A parent asks about whether a 7-year-old child is able to care for a dog. Based on the child's age, what does the nurse suggest? a. Caring for an animal requires more maturity than the average 7-year-old possesses. b. This will help the parent identify the child's weaknesses. c. A dog can help the child develop confidence and emotional health. d. Cats are better pets for school-age children.

ANS: C - A dog can help the child develop confidence and emotional health. Pets have been observed to influence a child's self-esteem. They can have a positive effect on physical and emotional health and can teach children the importance of nurturing and nonverbal communication. Most 7-year-old children are capable of caring for a pet with supervision. Caring for a pet should be a positive experience. It should not be used to identify weaknesses. The pet chosen does not matter as much as the child's being responsible for a pet.

What is an important consideration for the school nurse planning a class on injury prevention for adolescents? a. Adolescents generally are not risk takers. b. Adolescents can anticipate the long-term consequences of serious injuries. c. Adolescents need to discharge energy, often at the expense of logical thinking. d. During adolescence, participation in sports should be limited to prevent permanent injuries.

ANS: C - Adolescents need to discharge energy, often at the expense of logical thinking. The physical, sensory, and psychomotor development of adolescents provides a sense of strength and confidence. There is also an increase in energy coupled with risk taking that puts them at risk. Adolescents are risk takers because their feelings of indestructibility interfere with understanding of consequences. Sports can be a useful way for adolescents to discharge energy. Care must be taken to avoid overuse injuries.

A child with cyanide poisoning has been admitted to the emergency department. What antidote does the nurse anticipate being prescribed for the child? a. Atropine b. Glucagon c. Amyl nitrate d. Naloxone (Narcan)

ANS: C - Amyl nitrate Amyl nitrate is the antidote for cyanide poisoning. Atropine is an antidote for organophosphate poisoning, glucagon is an antidote for a beta-blocker poisoning, and naloxone (Narcan) is an antidote for an opioid poisoning.

Tretinoin (Retin-A) is a commonly used topical agent for the treatment of acne. What do nursing considerations with this drug include? a. Sun exposure increases effectiveness. b. Cosmetics with lanolin and petrolatum are preferred in acne. c. Applying of the medication occurs at least 20 to 30 minutes after washing. d. Erythema and peeling are indications of toxicity and need to be reported.

ANS: C - Applying of the medication occurs at least 20 to 30 minutes after washing. The medication should not be applied for at least 20 to 30 minutes after washing to decrease the burning sensation. The avoidance of sun and the use of sunscreen agents must be emphasized because sun exposure can result in severe sunburn. Cosmetics with lanolin, petrolatum, vegetable oil, lauryl alcohol, butyl stearate, and oleic acid can increase comedone production. Erythema and peeling are common local manifestations.

The nurse is admitting a 7-year-old child to the pediatric unit for abdominal pain. To determine what the child understands about the reason for hospitalization, what should the nurse do? a. Find out what the parents have told the child. b. Review the note from the admitting practitioner. c. Ask the child why he came to the hospital today. d. Question the parents about why they brought the child to the hospital.

ANS: C - Ask the child why he came to the hospital today. School-age children are able to answer questions. The only way for the nurse to know about the child's understanding of the reason for hospitalization is to ask the child directly. Finding out what the parents told the child and why they brought the child to the hospital or reading the admitting practitioner's description of the reason for admission will not provide information about what the child has heard and retained.

What is most descriptive of atopic dermatitis (AD) (eczema) in an infant? a. Easily cured b. Worse in humid climates c. Associated with hereditary allergies d. Related to upper respiratory tract infections

ANS: C - Associated with hereditary allergies AD is a type of pruritic eczema that usually begins during infancy and is associated with allergy with a hereditary tendency. Approximately 50% of children with AD develop asthma. AD can be controlled but not cured. Manifestations of the disease are worse when environmental humidity is lower. AD is not associated with respiratory tract infections.

What statement regarding chlamydial infections is correct? a. The treatment of choice is oral penicillin. b. The treatment of choice is nystatin or miconazole. c. Both men and women may have asymptomatic infections. d. Clinical manifestations include small, painful vesicles on the genital areas.

ANS: C - Both men and women may have asymptomatic infections. The incidence of asymptomatic chlamydial infections is as high as 50% of men and 75% of women. Symptoms of chlamydial infection in men include meatal erythema, tenderness, itching, dysuria, and urethral discharge. Oral penicillin, nystatin, and miconazole are not the antibiotics of choice. Small, painful vesicles on genital areas are clinical manifestations of herpetic infections

What statement accurately describes physical development during the school-age years? a. The child's weight almost triples. b. Muscles become functionally mature. c. Boys and girls double strength and physical capabilities. d. Fat gradually increases, which contributes to children's heavier appearance.

ANS: C - Boys and girls double strength and physical capabilities. Boys and girls double both strength and physical capabilities. Their consistent refinement in coordination increases their poise and skill. In middle childhood, growth in height and weight occurs at a slower pace. Between the ages of 6 and 12 years, children grow 5 cm/yr and gain 3 kg/yr. Their weight will almost double. Although the strength increases, muscles are still functionally immature when compared with those of adolescents. This age group is more easily injured by overuse. Children take on a slimmer look with longer legs in middle childhood.

In girls, what is the initial indication of puberty? a. Menarche b. Growth spurt c. Breast development d. Growth of pubic hair

ANS: C - Breast development In most girls, the initial indication of puberty is the appearance of breast buds, an event known as thelarche. The usual sequence of secondary sexual characteristic development in girls is breast changes, a rapid increase in height and weight, growth of pubic hair, appearance of axillary hair, menstruation (menarche), and abrupt deceleration of linear growth.

The nurse is assessing the Tanner stage in an adolescent female. The nurse recognizes that the stages are based on which? a. The stages of vaginal changes b. The progression of menstrual cycles to regularity c. Breast size and the shape and distribution of pubic hair d. The development of fat deposits around the hips and buttocks

ANS: C - Breast size and the shape and distribution of pubic hair In females, the Tanner stages describe pubertal development based on breast size and the shape and distribution of pubic hair. The stages of vaginal changes, progression of menstrual cycles to regularity, and the development of fat deposits occur during puberty but are not used for the Tanner stages.

The nurse is examining an infant, age 10 months, who was brought to the clinic for persistent diaper rash. The nurse finds perianal inflammation with satellite lesions. What is the most likely cause? a. Impetigo b. Urine and feces c. Candida albicans infection d. Infrequent diapering

ANS: C - Candida albicans infection C. albicans infection produces perianal inflammation and a maculopapular rash with satellite lesions that may cross the inguinal folds. Impetigo is a bacterial infection that spreads peripherally in sharply marginated, irregular outlines. Eruptions involving the skin in contact with the diaper but sparing the folds are likely to be caused by chemical irritation, especially urine and feces, and may be related to infrequent diapering.

The nurse should know what about Lyme disease? a. Very difficult to prevent b. Easily treated with oral antibiotics in stages 1, 2, and 3 c. Caused by a spirochete that enters the skin through a tick bite d. Common in geographic areas where the soil contains the mycotic spores that cause the disease

ANS: C - Caused by a spirochete that enters the skin through a tick bite Lyme disease is caused by Borrelia burgdorferi, a spirochete spread by ticks. The early characteristic rash is erythema migrans. Tick bites should be avoided by entering tick-infested areas with caution. Light-colored clothing should be worn to identify ticks easily. Long-sleeve shirts and long pants tucked into socks should be worn. Early treatment of the erythema migrans (stage 1) can prevent the development of Lyme disease. Lyme disease is caused by a spirochete, not mycotic spores

What is a characteristic of children with depression? a. Increased range of affective response b. Tendency to prefer play instead of schoolwork c. Change in appetite resulting in weight loss or gain d. Preoccupation with need to perform well in school

ANS: C - Change in appetite resulting in weight loss or gain Physiologic characteristics of children with depression include changes in appetite resulting in weight loss or gain, nonspecific complaints of not feeling well, alterations in sleeping patterns, insomnia or hypersomnia, and constipation. Children who are depressed have sad facial expressions with absent or diminished range of affective response. These children withdraw from previously enjoyed activities and engage in solitary play or work with a lack of interest in play. They are uninterested in doing homework or achieving in school, resulting in lower grades.

The nurse is caring for a child with a decubiti on the buttocks. The nurse notes that the dressing covering the decubiti is loose. What action should the nurse implement? a. Retape the dressing b. Remove the dressing. c. Change the dressing. d. Reinforce the dressing.

ANS: C - Change the dressing. Dressings should always be changed when they are loose or soiled. They should be changed more frequently in areas where contamination is likely (e.g., sacral area, buttocks, tracheal area). The dressing should not be retaped, removed, or reinforced.

The parents of 9-year-old twin children tell the nurse, "They have filled up their bedroom with collections of rocks, shells, stamps, and bird nests." The nurse should recognize that this is which? a. Indicative of giftedness b. Indicative of typical twin behavior c. Characteristic of cognitive development at this age d. Characteristic of psychosocial development at this age

ANS: C - Characteristic of cognitive development at this age Classification skills involve the ability to group objects according to the attributes they have in common. School-age children can place things in a sensible and logical order, group and sort, and hold a concept in their mind while they make decisions based on that concept. Individuals who are not twins engage in classification at this age. Psychosocial behavior at this age is described according to Erikson's stage of industry versus inferiority.

An infant has been diagnosed with failure to thrive (FTT) classified according to the pathophysiology of defective utilization. The nurse understands that the reason for the FTT is most likely related to what? a. Cystic fibrosis b. Hyperthyroidism c. Congenital infection d. Breastfeeding problems

ANS: C - Congenital infection FTT classified according to the pathophysiology of defective utilization is related to a genetic anomaly, congenital infection of metabolic storage disease. Cystic fibrosis would be related to the pathophysiology of inadequate absorption, hyperthyroidism would be related to the pathophysiology of increased metabolism, and breastfeeding problems are related to inadequate caloric intake

What goal is most important when caring for a child with anorexia nervosa (AN)? a. Limit fluid intake. b. Prevent depression. c. Correct malnutrition. d. Encourage weight gain.

ANS: C - Correct malnutrition. In children diagnosed with AN or bulimia nervosa, the priority consideration is to correct the malnutrition. Severe malnutrition, electrolyte disturbances, vital sign abnormalities, and psychiatric disorders may be present. Careful monitoring is necessary to avoid complications. Often fluid intake is restricted by individuals with AN. Fluid balance must be restored. Preventing depression is important, but the correction of potentially life-threatening malnutrition takes precedence. After the initial malnutrition is corrected, then a plan is established for nutritional therapy

The nurse is talking to the parent of a 5-year-old child who refuses to go to sleep at night. What intervention should the nurse suggest in helping the parent to cope with this sleep disturbance? a. Establish a consistent punishment if the child does not go to bed when told. b. Allow the child to fall asleep in a different room and then gently move the child to his or her bed. c. Establish limited rituals that signal readiness for bedtime. d. Allow the child to watch television until almost asleep.

ANS: C - Establish limited rituals that signal readiness for bedtime. An appropriate intervention for a child who resists going to bed is to establish limited rituals such as a bath or story that signal readiness for bed and consistently follow through with the ritual. Punishing the child will not alleviate the resistance problem and may only add to the frustration. Allowing the child to fall asleep in a different room and to watch television to fall asleep are not recommended approaches to sleep resistance.

A new parent relates to the nurse that the family has many known food allergies. Which is considered a primary strategy for feeding the infant with many family food allergies? a. Using soy formula for feeding b. Maternal avoidance of cow's milk protein c. Exclusive breastfeeding for 4 to 6 months d. Delaying the introduction of highly allergenic foods past 6 months

ANS: C - Exclusive breastfeeding for 4 to 6 months Exclusive breastfeeding for 4 to 6 months is now considered a primary strategy for avoiding atopy in families with known food allergies; however, there is no evidence that maternal avoidance (during pregnancy or lactation) of cow's milk protein or other dietary products known to cause food allergy will prevent food allergy in children. Researchers indicate that delaying the introduction of highly allergenic foods past 4 to 6 months of age may not be as protective for food allergy as previously believed. Likewise, studies have shown that soy formula does not prevent allergic disease in infants.

A child with diazepam (Valium) poisoning has been admitted to the emergency department. What antidote does the nurse anticipate being prescribed? a. Succimer (Chemet) b. EDTA (Versenate) c. Flumazenil (Romazicon) d. Octreotide acetate (Sandostatin)

ANS: C - Flumazenil (Romazicon) The antidote for diazepam (Valium) poisoning is flumazenil (Romazicon). Succimer (Chemet) and EDTA (Versenate) are antidotes for heavy metal poisoning. Octreotide acetate (Sandostatin) is an antidote for sulfonylurea poisoning.

The nurse is performing an assessment on a 12-month-old infant. Which fine or gross motor developmental skill demonstrates the proximodistal acquisition of skills? a. Standing b. Sitting without assistance c. Fully developed pincer grasp d. Taking a few steps holding onto something

ANS: C - Fully developed pincer grasp Acquisition of fine and gross motor skills occurs in an orderly center-to-periphery (proximodistal) or head-to-toe (cephalocaudal) sequence. A fully developed pincer grasp is an example of the proximodistal development because infants use a palmar grasp before developing the finer pincer grasp. Standing, sitting without assistance, and taking a few steps are examples of a cephalocaudal development sequence.

What is an appropriate action when an infant becomes apneic? a. Shake vigorously. b. Roll the infant's head to the side. c. Gently stimulate the trunk by patting or rubbing. d. Hold the infant by the feet upside down with the head supported.

ANS: C - Gently stimulate the trunk by patting or rubbing. If an infant is apneic, the infant's trunk should be gently stimulated by patting or rubbing. If the infant is prone, turn onto the back. Vigorous shaking, rolling of the head, and hanging the child upside down can cause injury and should not be done

Which intervention is the most appropriate recommendation for relief of teething pain? a. Rub gums with aspirin to relieve inflammation. b. Apply hydrogen peroxide to gums to relieve irritation. c. Give the infant a frozen teething ring to relieve inflammation. d. Have the infant chew on a warm teething ring to encourage tooth eruption.

ANS: C - Give the infant a frozen teething ring to relieve inflammation. Teething pain is a result of inflammation, and cold is soothing. A frozen teething ring or ice cube wrapped in a washcloth helps relieve the inflammation. Aspirin is contraindicated secondary to the risks of aspiration. Hydrogen peroxide does not have an anti-inflammatory effect. Warmth increases inflammation.

At a well-child visit, parents ask the nurse how to know if a daycare facility is a good choice for their infant. Which observation should the nurse stress as especially important to consider when making the selection? a. Developmentally appropriate toys b. Nutritious snacks served to the children c. Handwashing by providers after diaper changes d. Certified caregivers for each of the age groups at the facility

ANS: C - Handwashing by providers after diaper changes Health practices should be most important. With the need for diaper changes and assistance with feeding, young children are at increased risk when handwashing and other hygienic measures are not consistently used. Developmentally appropriate toys are important, but hygiene and the prevention of disease transmission take precedence. An infant should not have snacks. This is a concern for an older child. Certified caregivers for each age group may be an indicator of a high-quality facility, but parental observation of good hygiene is a better predictor of care.

What behavior is the nurse most likely to assess in an adolescent with anorexia nervosa (AN)? a. Eats in secrecy b. Uses food as a coping mechanism c. Has a marked preoccupation with food d. Lacks awareness of how eating affects weight loss

ANS: C - Has a marked preoccupation with food Individuals with AN display great interest in food. They prepare meals for others, talk about food, and hoard food. During meals, food play may occur to appear as if the person is eating. Persons with AN consume a small amount of food, so they have no need to eat in secrecy. Individuals with bulimia nervosa (BN) usually binge privately. Food is not used as a coping mechanism in AN, as is common in BN. Individuals with AN know about the relationship between calorie intake and calorie expenditure. They can regulate intake and then exercise to not gain or to lose weight

The parents of a 3-year-old admitted for recurrent diarrhea are upset that the practitioner has not told them what is going on with their child. What is the priority intervention for this family? a. Answer all of the parents' questions about the child's illness. b. Immediately page the practitioner to come to the unit to speak with the family. c. Help the family develop a written list of specific questions to ask the practitioner. d. Inform the family of the time that hospital rounds are made so that they can be present.

ANS: C - Help the family develop a written list of specific questions to ask the practitioner. Often families ask general questions of health care providers and do not receive the information they need. The nurse should determine what information the family does want and then help develop a list of questions. When the questions are written, the family can remember which questions to ask or can hand the sheet to the practitioner for answers. The nurse may have the information the parents want, but they are asking for specific information from the practitioner. Unless it is an emergency, the nurse should not place a stat page for the practitioner. Being present is not necessarily the issue but rather the ability to get answers to specific questions.

The nurse is assessing a child's functional self-care level for feeding, bathing and hygiene, dressing, and grooming and toileting. The child requires assistance or supervision from another person and equipment or device. What code does the nurse assign for this child? a. I b. II c. III d. IV

ANS: C - III A code of III indicates the child requires assistance from another person and equipment or device. A code of I indicates use of equipment or device. A code of II indicates assistance or supervision from another person. A code of IV indicates the child is totally dependent.

What signals the resolution of the Oedipus or Electra complex? a. Learns sex differences b. Learns sexually appropriate behavior c. Identifies with the same-sex parent d. Has guilt over feelings toward the father or mother

ANS: C - Identifies with the same-sex parent The resolution of the Oedipus or Electra complex is identification with the same-sex parent. Learning sex differences and sexually appropriate behavior is a goal in further differentiation of oneself but does not signal the resolution of the Oedipus or Electra complex. Guilt over feelings toward the father or mother is seen as a stage in the complex, not the resolution.

The middle school nurse is planning a behavior modification program for overweight children. What is the most important goal for participants of the program? a. Learn how to cook low-fat meals. b. Improve relationships with peers. c. Identify and eliminate inappropriate eating habits. d. Achieve normal weight during the program.

ANS: C - Identify and eliminate inappropriate eating habits. The goal of behavior modification in weight control is to help the participant identify abnormal eating processes. After the abnormal patterns are identified, then techniques, including problem solving, are taught to eliminate inappropriate eating. Learning how to cook low-fat meals can be a component of the program, but the focus of behavior modification is identifying target behaviors that need to be changed. Improving relationships is not the focus of weight management behavior management programs. Achieving normal weight during the program is an inappropriate goal. As the child incorporates the techniques, weight gain will slow. In childhood obesity, the goal is to stop the increase of weight gain.

What behavior should most likely be manifested in an infant experiencing the protest phase of separation anxiety? a. Inactivity b. Depression and sadness c. Inconsolable and crying d. Regression to earlier behavior

ANS: C - Inconsolable and crying For older infants, being inconsolable and crying is seen during the protest phase of separation anxiety. Inactivity is observed during the stage of despair. The child is much less active and withdraws from others. Depression, sadness, and regression to earlier behaviors are observed during the phase of despair.

What is a characteristic of a toddler's language development at age 18 months? a. Vocabulary of 25 words b. Use of holophrases c. Increasing level of understanding d. Approximately one third of speech understandable

ANS: C - Increasing level of understanding During the second year of life, the understanding and understanding of speech increase to a level far greater than the child's vocabulary. This is also true for bilingual children, who are able to achieve this linguistic milestone in both languages. An 18-month-old child has a vocabulary of approximately 10 words. At this age, the child does not use the one-word sentences that are characteristic of 1-year-old children. The child has a very limited vocabulary of single words that are comprehensible.

The nurse understands that medications delivered by which route are more likely to cause a drug reaction? a. Oral b. Topical c. Intravenous d. Intramuscular

ANS: C - Intravenous Drugs administered by the intravenous route are more likely to cause a reaction than the oral, topical, or intramuscular route.

What is true about pelvic inflammatory disease (PID)? a. It can be prevented by proper personal hygiene. b. It is easily prevented by compliance with any form of contraception. c. It may have devastating effects on the reproductive tract of affected adolescents. d. It can potentially cause life-threatening and serious defects in the future children of affected adolescents.

ANS: C - It may have devastating effects on the reproductive tract of affected adolescents. PID is a major concern because of its devastating effects on the reproductive tract. Short-term complications include abscess formation in the fallopian tubes, and long-term complications include ectopic pregnancy, infertility, and dyspareunia. PID is an infection of the upper female genital tract, most commonly caused by sexually transmitted infections. Personal hygiene, oral contraceptives, and many other forms of contraception do not prevent transmission of the disease. There is a possibility of ectopic pregnancy but not birth defects in children.

What is an important nursing consideration when a child is hospitalized for chelation therapy to treat lead poisoning? a. Maintain bed rest. b. Maintain isolation precautions. c. Keep an accurate record of intake and output. d. Institute measures to prevent skeletal fracture.

ANS: C - Keep an accurate record of intake and output. The iron chelates are excreted though the kidneys. Adequate hydration is essential. Periodic measurement of renal function is done. Bed rest is not necessary. Often the chelation therapy is done on an outpatient basis. Chelation therapy is not infectious or dangerous. Isolation is not indicated. Skeletal weakness does not result from high levels of lead.

What is descriptive of the play of school-age children? a. They like to invent games, making up the rules as they go. b. Individuality in play is better tolerated than at earlier ages. c. Knowing the rules of a game gives an important sense of belonging. d. Team play helps children learn the universal importance of competition and winning.

ANS: C - Knowing the rules of a game gives an important sense of belonging. Play involves increased physical skill, intellectual ability, and fantasy. Children form groups and cliques and develop a sense of belonging to a team or club. At this age, children begin to see the need for rules. Conformity and ritual permeate their play. Their games have fixed and unvarying rules, which may be bizarre and extraordinarily rigid. With team play, children learn about competition and the importance of winning, an attribute highly valued in the United States but not in all cultures.

The parents of a 4-month-old infant tell the nurse that they are getting a microwave oven and will be able to heat the baby's formula faster. What should the nurse recommend? a. Heat only 8 oz or more. b. Do not heat a plastic bottle in a microwave oven. c. Leave the bottle top uncovered to allow heat to escape. d. Shake the bottle vigorously for at least 30 seconds after heating.

ANS: C - Leave the bottle top uncovered to allow heat to escape. If a microwave is being used, the bottle should be left uncovered. This will allow heat to escape. No more than 4 oz should be heated at any one time. Bottles can be heated safely in microwave ovens if safety guidelines are followed. The bottle should be inverted 10 times; vigorous shaking is not necessary.

What is characteristic of dishonest behavior in children ages 8 to 10 years? a. Cheating during games is now more common. b. Stealing can occur because their sense of property rights is limited. c. Lying is used to meet expectations set by others that they have been unable to attain. d. Dishonesty results from the inability to distinguish between fact and fantasy.

ANS: C - Lying is used to meet expectations set by others that they have been unable to attain. Older school-age children may lie to meet expectations set by others to which they have been unable to measure up. Cheating usually becomes less frequent as the child matures. Young children may lack a sense of property rights; older children may steal to supplement an inadequate allowance, or it may be an indication of serious problems. In this age group, children are able to distinguish between fact and fantasy

An occlusive dressing is applied to a large abrasion. This is advantageous because the dressing will accomplish what? a. Deliver vitamin C to the wound. b. Provide an antiseptic for the wound. c. Maintain a moist environment for healing. d. Promote mechanical friction for healing.

ANS: C - Maintain a moist environment for healing. Occlusive dressings, such as Acuderm, are not adherent to the wound site. They provide a moist wound surface and insulate the wound. The dressing does not have vitamin C or antiseptic capabilities. Acuderm protects against friction.

An adolescent patient has been diagnosed with a vulvovaginal candidiasis (yeast infection). The nurse expects the health care provider to recommend which vaginal cream? a. Premarin b. Estradiol (Estrace) c. Miconazole (Monistat) d. Clindamycin phosphate (Cleocin)

ANS: C - Miconazole (Monistat) A number of antifungal preparations are available for the treatment of vulvovaginal candidiasis infections. Many of these medications (e.g., miconazole [Monistat] and clotrimazole [Gyne-Lotrimin]) are available as over-the-counter (OTC) agents. Premarin and Estrace are estrogen vaginal creams and are used to treat vaginal dryness. Cleocin is an antibacterial vaginal cream used to treat bacterial vaginal infections.

A 14-year-old boy is of normal weight, and his parents are concerned about bilateral breast enlargement. The nurse's discussion of this should be based on what? a. The presence of too much body fat b. Symptom that a hormonal imbalance is present c. Most likely part of normal pubertal development d. Indication that he is developing precocious puberty

ANS: C - Most likely part of normal pubertal development Gynecomastia is common during midpuberty in about one third of boys. For most, the breast enlargement disappears within 2 years. Although breast enlargement in overweight children can indicate too much body fat, in children of normal body weight, it is a normal occurrence. If the gynecomastia persists beyond 2 years, then a hormonal cause may need to be investigated. Precocious puberty is the early onset of puberty, before age 9 years in boys.

What statement best describes fear in school-age children? a. Increasing concerns about bodily safety overwhelm them. b. They should be encouraged to hide their fears to prevent ridicule by peers. c. Most of the new fears that trouble them are related to school and family. d. Children with numerous fears need continuous protective behavior by parents to eliminate these fears.

ANS: C - Most of the new fears that trouble them are related to school and family. During the school-age years, children experience a wide variety of fears, but new fears related predominantly to school and family bother children during this time. Parents and other persons involved with children should discuss children's fear with them individually or as a group activity. Sometimes school-age children hide their fears to avoid being teased. Hiding the fears does not end them and may lead to phobias.

What statement best describes the relationship school-age children have with their families? a. Ready to reject parental controls b. Desire to spend equal time with family and peers c. Need and want restrictions placed on their behavior by the family d. Peer group replaces the family as the primary influence in setting standards of behavior and rules

ANS: C - Need and want restrictions placed on their behavior by the family School-age children need and want restrictions placed on their behavior, and they are not prepared to cope with all the problems of their expanding environment. Although increased independence is the goal of middle childhood, they feel more secure knowing that an authority figure can implement controls and restriction. In the middle school years, children prefer peer group activities to family activities and want to spend more time in the company of peers. Family values usually take precedence over peer value systems.

What is marasmus? a. Deficiency of protein with an adequate supply of calories b. Syndrome that results solely from vitamin deficiencies c. Not confined to geographic areas where food supplies are inadequate d. Characterized by thin, wasted extremities and a prominent abdomen resulting from edema (ascites)

ANS: C - Not confined to geographic areas where food supplies are inadequate Marasmus is a syndrome of emotional and physical deprivation. It is not confined to geographic areas were food supplies are inadequate. Marasmus is characterized by gradual wasting and atrophy of body tissues, especially of subcutaneous fat. The child appears old, with flabby and wrinkled skin. Marasmus is a deficiency of both protein and calories.

What parents should have the most difficult time coping with their child's hospitalization? a. Parents of a child hospitalized for juvenile arthritis b. Parents of a child hospitalized with a recent diagnosis of bronchiolitis c. Parents of a child hospitalized for sepsis resulting from an untreated injury d. Parents of a child hospitalized for surgical correction of undescended testicles

ANS: C - Parents of a child hospitalized for sepsis resulting from an untreated injury Factors that affect parents' reactions to their child's illness include the seriousness of the threat to the child. The parents of a child hospitalized for sepsis resulting from an untreated injury would have more difficulty coping because of the seriousness of the illness and because the wound was not treated immediately.

Descriptions of young people with anorexia nervosa (AN) often include which criteria? a. Impulsive b. Extroverted c. Perfectionist d. Low achieving

ANS: C - Perfectionist Individuals with AN are described as striving for perfection, which may manifest in other compulsive disorders. They are also academically high achievers. Impulsive and extroverted personalities are more characteristic of bulimia nervosa.

During a well-child visit, the father of a 4-year-old boy tells the nurse that he is not sure if his son is ready for kindergarten. The boy's birthday is close to the cut-off date, and he has not attended preschool. What is the nurse's best recommendation? a. Start kindergarten. b. Talk to other parents about readiness. c. Perform a developmental screening. d. Postpone kindergarten and go to preschool.

ANS: C - Perform a developmental screening. A developmental assessment with a screening tool that addresses cognitive, social, and physical milestones can help identify children who may need further assessment. A readiness assessment involves an evaluation of skill acquisition. Stating the child should start kindergarten or go to preschool and postpone kindergarten does not address the father's concerns about readiness for school. Talking to other parents about readiness does not ascertain if the child is ready and does not address the father's concerns.

A 4-year-old child tells the nurse that she doesn't want another blood sample drawn because "I need all of my insides and I don't want anyone taking them out." What is the nurse's best interpretation of this? a. The child is being overly dramatic. b. The child has a disturbed body image. c. Preschoolers have poorly defined body boundaries. d. Preschoolers normally have a good understanding of their bodies.

ANS: C - Preschoolers have poorly defined body boundaries. Preschoolers have little understanding of body boundaries, which leads to fears of mutilation. The child is not capable of being dramatic at this age. She truly has fear. Body image is just developing in school-age children. Preschoolers do not have good understanding of their bodies.

A 6-year-old is being discharged home, which is 90 miles from the hospital, after an outpatient hernia repair. In addition to explicit discharge instructions, what should the nurse provide? a. An ambulance for transport home b. Verbal information about follow-up care c. Prescribed pain medication before discharge d. Driving instructions for a route with less traffic

ANS: C - Prescribed pain medication before discharge The nurse should anticipate that the child will begin experiencing pain on the trip home. By providing a dose of oral analgesia, the nurse can ensure the child remains comfortable during the trip. Transport by ambulance is not indicated for a hernia repair. Discharge instructions should be written. The parents will be focusing on their child and returning home, which limits their ability to retain information. The parents should know the most expedient route home.

What developmental characteristic does not occur until a child reaches age 2 1/2 years? a. Birth weight has doubled. b. Anterior fontanel is still open. c. Primary dentition is complete. d. Binocularity may be established.

ANS: C - Primary dentition is complete. Usually by age 30 months, the primary dentition of 20 teeth is complete. Birth weight doubles at approximately ages 5 to 6 months. The anterior fontanel closes at ages 12 to 18 months. Binocularity is established by age 15 months

The nurse's role in facilitating successful childrearing in unmarried teenage mothers includes what? a. Facilitating marriage between the mother and father of the baby b. Teaching the adolescent the long-term needs of the growing child c. Providing information and feedback about positive parenting skills d. Encouraging the infant's grandmother to take responsibility for care

ANS: C - Providing information and feedback about positive parenting skills Competence in a teenage mother is increased when feedback is provided about positive parenting skills and use of community resources. The nurse can identify and refer the mother to programs such as support groups for adolescent mothers, infant stimulation programs, and parenting programs. Facilitating marriage between the mother and the father of the baby may produce additional stress and detract from their ability to care for the infant. Encouraging the infant's grandmother to take responsibility for care would decrease the mother's ability to develop successful childrearing behaviors. Supportive families can provide assistance to enable the teenage mother to complete school. Many adolescents do not have a future perspective for themselves. The nurse includes information on normal infant development to aid the mother in having reasonable expectations

The school nurse suspects a testicular torsion in a young adolescent student. What action should the nurse take? a. Place a warm moist pack on the scrotal area. b. Instruct the adolescent to lie down and elevate the legs. c. Refer the adolescent for immediate medical evaluation. d. Suggest that the adolescent wear a scrotum-protecting guard.

ANS: C - Refer the adolescent for immediate medical evaluation. Because torsion may result from trauma to the scrotum, school nurses are likely to encounter such injuries and should refer the child or adolescent for medical evaluation immediately. It would not be appropriate to apply warmth, elevate the legs, or tell the adolescent to wear a scrotum-protecting guard because these actions could delay treatment

What statement is true about smoking in college students? a. The rate of smoking cigarettes is declining. b. Smokeless tobacco use is rising dramatically. c. Regular cigar use is becoming more common. d. Students in the health professions do not smoke.

ANS: C - Regular cigar use is becoming more common. Approximately 8.5% of college students smoke cigars on a regular basis. Among college students, the rate of cigarette smoking is rising. At last report, 28.5% of this group smoked cigarettes. Use of smokeless tobacco is declining overall. Students in the health professions do smoke

The school nurse is seeing a child who collected some poison ivy leaves during recess. He says only his hands touched it. What is the most appropriate nursing action? a. Soak his hands in warm water. b. Apply Burow's solution compresses. c. Rinse his hands in cold running water. d. Scrub his hands thoroughly with antibacterial soap.

ANS: C - Rinse his hands in cold running water. The first recommended action is to rinse his hands in cold running water within 15 minutes of exposure. This will neutralize the urushiol not yet bonded to the skin. Soaking his hands in warm water is effective for soothing the skin lesions after the dermatitis has begun. Antibacterial soap removes protective skin oils and dilutes the urushiol, allowing it to spread.

According to Piaget, a 6-month-old infant should be in which developmental stage? a. Use of reflexes b. Primary circular reactions c. Secondary circular reactions d. Coordination of secondary schemata

ANS: C - Secondary circular reactions Infants are usually in the secondary circular reaction stage from ages 4 to 8 months. This stage is characterized by a continuation of the primary circular reaction for the response that results. Shaking is performed to hear the noise of the rattle, not just for shaking. The use of reflexes stage is primarily during the first month of life. The primary circular reaction stage marks the replacement of reflexes with voluntary acts. The infant is in this stage from ages 1 to 4 months. The fourth sensorimotor stage is coordination of secondary schemata, which occurs at ages 9 to 12 months. This is a transitional stage in which increasing motor skills enable greater exploration of the environment

A child with corrosive poisoning is being admitted to the emergency department. What clinical manifestation does the nurse expect to assess on this child? a. Nausea and vomiting b. Alterations in sensorium, such as lethargy c. Severe burning pain in the mouth, throat, and stomach d. Respiratory symptoms of acute pulmonary involvement

ANS: C - Severe burning pain in the mouth, throat, and stomach Severe burning pain in the mouth, throat, and stomach is a clinical manifestation of corrosive poisoning. Nausea and vomiting; alterations in sensorium, such as lethargy; and respiratory symptoms of acute pulmonary involvement are clinical manifestations of hydrocarbon poisoning.

The school nurse has been asked to begin teaching sex education in the fifth grade. What should the nurse recognize? a. Questions need to be discouraged in this setting. b. Most children in the fifth grade are too young for sex education. c. Sexuality is presented as a normal part of growth and development. d. Correct terminology should be reserved for children who are older.

ANS: C - Sexuality is presented as a normal part of growth and development. When sexual information is presented to school-age children, sex should be treated as a normal part of growth and development. They should be encouraged to ask questions. At 10 to 11 years old, fifth graders are not too young to speak about physiologic changes in their bodies. Preadolescents need precise and concrete information.

What dysfunctional speech pattern is a normal characteristic of the language development of a preschool child? a. Lisp b. Echolalia c. Stammering d. Repetition without meaning

ANS: C - Stammering Stammering and stuttering are normal dysfluency in preschool-age children. Lisps are not a normal characteristic of language development. Echolalia and repetition are traits of toddlers' language.

A parent calls the clinic nurse because his 7-year-old child was bitten by a black widow spider. What action should the nurse advise the parent to take? a. Apply warm compresses. b. Carefully scrape off the stinger. c. Take the child to the emergency department. d. Apply a thin layer of corticosteroid cream.

ANS: C - Take the child to the emergency department. The venom of the black widow spider has a neurotoxic effect. The parent should take the child to the emergency department for treatment with antivenin and muscle relaxants as needed. Warm compresses increase the circulation to the area and facilitate the spread of the venom. The black widow spider does not have a stinger. Corticosteroid cream has no effect on the venom.

A 17-month-old child should be expected to be in which stage, according to Piaget? a. Preoperations b. Concrete operations c. Tertiary circular reactions d. Secondary circular reactions

ANS: C - Tertiary circular reactions A 17-month-old is in the fifth stage of the sensorimotor phase, tertiary circular reactions. The child uses active experimentation to achieve previously unattainable goals. Preoperations is the stage of cognitive development usually present in older toddlers and preschoolers. Concrete operations is the cognitive stage associated with school-age children. The secondary circular reaction stage lasts from about ages 4 to 8 months

In boys, what is the initial indication of puberty? a. Voice changes b. Growth of pubic hair c. Testicular enlargement d. Increased size of penis

ANS: C - Testicular enlargement Testicular enlargement is the first change that signals puberty in boys; it usually occurs between the ages of 9 1/2 and 14 years during Tanner stage 2. Voice change occurs between Tanner stages 3 and 4. Fine pubic hair may occur at the base of the penis; darker hair occurs during Tanner stage 3. The penis enlarges during Tanner stage 3.

The nurse has determined that an adolescent's body mass index (BMI) is in the 90th percentile. What information should the nurse convey to the adolescent? a. The adolescent is overweight. b. The adolescent has maintained weight within the normal range. c. The adolescent is at risk for becoming overweight. d. Nutritional supplementation should occur at least three times per week

ANS: C - The adolescent is at risk for becoming overweight. Adolescents with BMIs between the 85th and 94th percentile for age and gender are at risk for becoming overweight. Adolescents with BMIs greater than the 95th percentile are classified as overweight. Nutritional guidance, not supplementation, is needed.

Two hospitalized adolescents are playing pool in the activity room. Neither of them seems enthusiastic about the game. How should the nurse interpret this situation? a. Playing pool requires too much concentration for this age group. b. Pool is an activity better suited for younger children. c. The adolescents may be enjoying themselves but have lower energy levels than healthy children. d. The adolescents' lack of enthusiasm is one of the signs of depression.

ANS: C - The adolescents may be enjoying themselves but have lower energy levels than healthy children. Children who are ill and hospitalized typically have lower energy levels than healthy children. Therefore, children may not appear enthusiastic about an activity even when they are enjoying it. Pool is an appropriate activity for adolescents. They have the cognitive and psychomotor skills that are necessary. If the adolescents were significantly depressed, they would be unable to engage in the game.

What does the nurse understand about caloric needs for school-age children? a. The caloric needs for the school-age children are the same as for other age groups. b. The caloric needs for school-age children are more than they were in the preschool years. c. The caloric needs for school-age children are lower than they were in the preschool years. d. The caloric needs for school-age children are greater than they will be in the adolescent years.

ANS: C - The caloric needs for school-age children are lower than they were in the preschool years. School-age children do not need to be fed as carefully, as promptly, or as frequently as before. Caloric needs are lower than they were in the preschool years and lower than they will be during the coming adolescent growth spurt.

The nurse is performing an assessment on a 10-week-old infant. The nurse understands that the developmental characteristic of hearing at this age is which? a. The infant responds to his own name. b. The infant localizes sounds by turning his head directly to the sound. c. The infant turns his head to the side when sound is made at the level of the ear. d. The infant locates sound by turning his head to the side and then looking up or down.

ANS: C - The infant turns his head to the side when sound is made at the level of the ear. At 8 to 12 weeks of age, the infant turns the head to the side when sound is made at the level of the ear. At 16 to 24 weeks, the infant locates sound by turning the head to the side and then looking up or down. At 24 to 32 weeks, infants respond to their own name. At 32 to 40 weeks, the infant localizes sounds by turning the head directly toward the sound.

Although a 14-month-old girl received a shock from an electrical outlet recently, her parent finds her about to place a paper clip in another outlet. Which is the best interpretation of this behavior? a. Her cognitive development is delayed. b. This is typical behavior because toddlers are not very developed. c. This is typical behavior because of toddlers' inability to transfer remembering to new situations. d. This is not typical behavior because toddlers should know better than to repeat an act that caused pain.

ANS: C - This is typical behavior because of toddlers' inability to transfer remembering to new situations. During the tertiary circular reactions stage, children have only a rudimentary sense of the classification of objects. The appearance of an object denotes its function for these children. The slot of an outlet is for putting things into. This is typical behavior for a toddler, who is only somewhat aware of a causal relation between events. Her cognitive development is appropriate for her age

The school nurse is presenting sexual information to a group of school-age girls. What approach should the nurse take when presenting the information? a. Put off answering questions. b. Give technical terms when giving the presentation. c. Treat sex as a normal part of growth and development. d. Plan to give the presentation with boys and girls together.

ANS: C - Treat sex as a normal part of growth and development. When nurses present sexual information to children, they should treat sex as a normal part of growth and development. Nurses should answer questions honestly, matter-of-factly, and at the children's level of understanding. School-age children may be more comfortable when boys and girls are segregated for discussions.

In terms of gross motor development, what should the nurse expect an infant age 5 months to do? a. Sit erect without support. b. Roll from the back to the abdomen. c. Turn from the abdomen to the back. d. Move from a prone to a sitting position.

ANS: C - Turn from the abdomen to the back. Rolling from the abdomen to the back is developmentally appropriate for a 5-month-old infant. The ability to roll from the back to the abdomen is developmentally appropriate for an infant at age 6 months. Sitting erect without support is a developmental milestone usually achieved by 8 months. A 10-month-old infant can usually move from a prone to a sitting position.

During the 2-month well-child checkup, the nurse expects the infant to respond to sound in which manner? a. Respond to name. b. React to loud noise with Moro reflex. c. Turn his or her head to side when sound is at ear level. d. Locate sound by turning his or her head in a curving arc.

ANS: C - Turn his or her head to side when sound is at ear level. At 2 months of age, an infant should turn his or her head to the side when a noise is made at ear level. At birth, infants respond to sound with a startle or Moro reflex. An infant responds to his or her name and locates sounds by turning his or her head in a curving arc at age 6 to 9 months.

The nurse is facilitating a conference between the teachers and parents of a 7-year-old child newly diagnosed with attention deficit hyperactivity disorder (ADHD). What does the nurse stress? a. Academic subjects should be taught in the afternoon. b. Low-interest activities in the classroom should be minimized. c. Visual references should accompany verbal instruction. d. The child's environment should be visually stimulating.

ANS: C - Visual references should accompany verbal instruction. Verbal instructions should always be accompanied by visual or written instructions. This provides the child with reinforcement and a reference to expectations. Academic subjects should be taught in the morning when the child is experiencing the effects of the morning dose of medication. Low-interest activities should be mixed with high-interest activities to maintain the child's attention. Environmental stimulation should be minimized to help eliminate distractions that can overexcite the child.

A 1-year-old child is on a pure vegetarian (vegan) diet. This diet requires supplementation with what? a. Niacin b. Folic acid c. Vitamins D and B12 d. Vitamins C and E

ANS: C - Vitamins D and B12 Pure vegetarian (vegan) diets eliminate any food of animal origin, including milk and eggs. These diets require supplementation with many vitamins, especially vitamin B6, vitamin B12, riboflavin, vitamin D, iron, and zinc. Niacin, folic acid, and vitamins C and E are readily obtainable from foods of vegetable origin.

The nurse is explaining about the developmental sequence in children's capacity to conserve matter to a group of parents. What type of matter is last in the sequence for a child to develop? a. Mass b. Length c. Volume d. Numbers

ANS: C - Volume There is a developmental sequence in children's capacity to conserve matter. Children usually grasp conservation of numbers (ages 5 to 6 years) before conservation of substance. Conservation of liquids, mass, and length usually is accomplished at about ages 6 to 7 years, conservation of weight sometime later (ages 9 to 10 years), and conservation of volume or displacement last (ages 9 to 12 years).

A new parent asks the nurse, "How can diaper rash be prevented?" What should the nurse recommend? a. Wash the infant with soap before applying a thin layer of oil. b. Clean the infant with soap and water every time diaper is changed. c. Wipe stool from the skin using water and a mild cleanser. d. When changing the diaper, wipe the buttocks with oil and powder the creases.

ANS: C - Wipe stool from the skin using water and a mild cleanser. Change the diaper as soon as it becomes soiled. Gently wipe stool from the skin with water and mild soap. The skin should be thoroughly dried after washing. Applying oil does not create an effective barrier. Over washing the skin should be avoided, especially with perfumed soaps or commercial wipes, which may be irritating. Baby powder should not be used because of the danger of aspiration

The parents of a 5-year-old child ask the nurse, "How many hours of sleep a night does our child need?" The nurse should give which response? a. "A 5-year-old child requires 8 hours of sleep." b. "A 5-year-old child requires 9.5 hours of sleep." c. "A 5-year-old child requires 10 hours of sleep." d. "A 5-year-old child requires 11.5 hours of sleep."

ANS: D - "A 5-year-old child requires 11.5 hours of sleep." Sleep requirements decrease during school-age years; 5-year-old children generally require 11.5 hours of sleep.

The nurse is presenting an educational program to a group of parents about differences between anorexia nervosa (AN) and bulimia nervosa (BN) at a community outreach program. What statement by a parent would indicate a need for additional teaching? a. "A child with AN will turn away from food to cope, but a child with BN turns to food to cope." b. "A child with AN maintains rigid control and is introverted, but a child with BN is an extrovert and frequently loses control." c. "A child with AN denies the illness, but a child with BN recognizes the illness." d. "A child with AN is usually sexually active and seeks intimacy, but a child with BN avoids intimacy and is usually not sexually active."

ANS: D - "A child with AN is usually sexually active and seeks intimacy, but a child with BN avoids intimacy and is usually not sexually active." A child with AN is usually the one who avoids intimacy and is not sexually active, but a child BN often seeks intimacy and is sexually active. A child with AN turns away from food to cope with life, maintains rigid control, is introverted, and denies the illness. A child with BN turns to food to cope, is an extrovert who loses control, and recognizes that he or she has an illness.

What choice of words or phrases would be inappropriate to use with a child? a. "Rolling bed" for "stretcher" b. "Special medicine" for "dye" c. "Make sleepy" for "deaden" d. "Catheter" for "intravenous"

ANS: D - "Catheter" for "intravenous" Children can grasp information only if it is presented on or close to their level of cognitive development. This necessitates an awareness of the words used to describe events or processes, and exploring family traditions or approaches to information sharing and creating patient specific language or context. Therefore, to prevent or alleviate fears, nurses must be aware of the medical terminology and vocabulary that they use every day and be sensitive to the use of slang or confusing terminology. "Catheter" is a medical term and would be confusing.

At a seminar for parents with preschool-age children, the nurse has discussed anticipatory tasks during the preschool years. Which statement by a parent should indicate a correct understanding of the teaching? a. "I should be worried if my 4-year-old child has an increase in sexual curiosity because this is a sign of sexual abuse." b. "I should expect my 5-year-old to change from a tranquil child to an aggressive child when school starts." c. "I should be concerned if my 4-year-old child starts telling exaggerated stories and has an imaginary playmate, since these could be signs of stress." d. "I should expect my 3-year-old child to have a more stable appetite and an increase in food selections."

ANS: D - "I should expect my 3-year-old child to have a more stable appetite and an increase in food selections." A 3-year-old child exhibits a more stable appetite than during the toddler years and is more willing to try different foods. A 4-year-old child is imaginative and indulges in telling "tall tales" and may have an imaginary playmate; these are normal findings, not signs of stress. Also a 4-year-old child has an increasing curiosity in sexuality, which is not a sign of child abuse. A 5-year-old child is usually tranquil, not aggressive like a 4-year-old child

The nurse is teaching an adolescent about acne care. What statement by the adolescent indicates a need for further teaching? a. "I will cleanse my face twice a day." b. "I will frequently shampoo my hair." c. "I will brush my hair away from my forehead." d. "I will use my antibacterial soap to cleanse my face."

ANS: D - "I will use my antibacterial soap to cleanse my face." Antibacterial soaps are ineffective and may be drying when used in combination with topical acne medications. Further teaching is needed if the adolescent indicates using antibacterial soap. Gentle cleansing with a mild cleanser once or twice daily is usually sufficient. For some adolescents, hygiene of the hair and scalp appears to be related to the clinical activity of acne. Acne on the forehead may improve with brushing the hair away from the forehead and more frequent shampooing.

The nurse is teaching parents about expected language development for their 6-month-old infant. The nurse recognizes the parents understand the teaching if they make which statement? a. "Our baby should comprehend the word 'no.'" b. "Our baby knows the meaning of saying 'mama.'" c. "Our baby should be able to say three to five words." d. "Our baby should begin to combine syllables, such as 'dada.'"

ANS: D - "Our baby should begin to combine syllables, such as 'dada.'" By 6 months, infants imitate sounds; add the consonants t, d, and w; and combine syllables (e.g., "dada"), but they do not ascribe meaning to the word until 10 to 11 months of age. By 9 to 10 months, they comprehend the meaning of the word "no" and obey simple commands accompanied by gestures. By age 1 year, they can say three to five words with meaning and may understand as many as 100 words.

The parents of a newborn say that their toddler "hates the baby. . . . He suggested that we put him in the trash can so the trash truck could take him away." What is the nurse's best reply? a. "Let's see if we can figure out why he hates the new baby." b. "That's a strong statement to come from such a small boy." c. "Let's refer him to counseling to work this hatred out. It's not a normal response." d. "That is a normal response to the birth of a sibling. Let's look at ways to deal with this."

ANS: D - "That is a normal response to the birth of a sibling. Let's look at ways to deal with this." The arrival of a new infant represents a crisis for even the best prepared toddler. Toddlers have their entire schedules and routines disrupted because of the new family member. The nurse should work with the parents on ways to involve the toddler in the newborn's care and to help focus attention on the toddler. The toddler does not hate the infant. This is an expected, normal response to the changes in routines and attention that affect the toddler. The toddler can be provided with a doll to imitate parents' behaviors. The child can care for the doll's needs at the same time the parent is performing similar care for the newborn.

The nurse is teaching parents about caring for their infant with seborrheic dermatitis (cradle cap). Which statement by the parents indicates understanding of the teaching? a. "We will rinse off the shampoo quickly and dry the scalp thoroughly." b. "We will shampoo the hair every other day with antiseborrheic shampoo." c. "We will be sure to shampoo the hair without removing any of the crusts." d. "We will use a fine-tooth comb to help remove the loosened crusts from the strands of hair."

ANS: D - "We will use a fine-tooth comb to help remove the loosened crusts from the strands of hair." A fine-tooth comb or a soft facial brush helps remove the loosened crusts from the strands of hair after shampooing. This is an accurate statement. Shampoo should applied to the scalp and allowed to remain on the scalp until the crusts soften. Shampoo should not be rinsed off quickly. The crusts should be removed, and shampooing with antiseborrheic shampoo should be done daily, not every other day.

The nurse is caring for a 3-year-old child during a long hospitalization. The parent is concerned about how to support the child's siblings during the hospitalization. What statement is appropriate for the nurse to make? a. "You should choose one parent to spend every night in the hospital while the other parent stays at home with the other children." b. "You could leave your hospitalized child for periods at night to be at home with the other children." c. "You should discourage the siblings from visiting because this could upset everyone in the family." d. "You could encourage a nightly phone call between the siblings as part of the bedtime routine."

ANS: D - "You could encourage a nightly phone call between the siblings as part of the bedtime routine." A supportive measure for siblings of a hospitalized child is to have a routine of a phone call at some point during the day or evening so the parent at the hospital can stay in touch and the children at home are involved and can hear that their sibling is doing well. Parents should alternate who stays at the hospital overnight to prevent burnout and to allow each parent time at home with the siblings. Encourage siblings to visit if appropriate to keep the family unit intact. Leaving the hospitalized child alone at night will not support the siblings at home and may cause problems with the hospitalized child.

At an 8-month-old well-baby visit, the parent tells the nurse that her infant falls asleep at night during the last bottle feeding but wakes up when moved to the infant's crib. What is the most appropriate response for the nurse to make? a. "You should put your baby to sleep 1 hour earlier without the nighttime feeding but with a pacifier for soothing." b. "You could place rice cereal in the last bottle feeding of the day to ensure a longer sleep pattern." c. "You should have your partner give the last bottle of the day and observe whether your infant stays awake for your partner." d. "You could increase daytime feeding intervals to every 4 hours and put your baby in the crib while the baby is still awake."

ANS: D - "You could increase daytime feeding intervals to every 4 hours and put your baby in the crib while the baby is still awake." Increasing the daytime intervals to 4 hours and placing the baby in the crib while still awake are interventions for nighttime sleeping problems. Putting the baby to bed 1 hour earlier with a pacifier will not stop the need for the bedtime bottle; there is no research that rice cereal in the bottle helps to satisfy the baby longer at night, and switching partners does not guarantee that the baby will go to sleep better.

The school nurse recognizes that pubertal delay in girls is considered if breast development has not occurred by which age? a. 10 years b. 11 years c. 12 years d. 13 years

ANS: D - 13 years Girls may be considered to have pubertal delay if breast development has not occurred by age 13 years or if menarche has not occurred within 2 to 2 1/2 years of the onset of breast development.

The clinic nurse is evaluating an adolescent with menses that have stopped occurring. The nurse understands that which minimum amount of time should the menses be absent after a period of menstruation to be diagnosed as secondary amenorrhea? a. 3 months b. 4 months c. 5 months d. 6 months

ANS: D - 6 months A 6-month or more cessation of menses after a period of menstruation is secondary amenorrhea.

The school nurse recognizes that adolescents should get how many hours of sleep each night? a. 6 hours b. 7 hours c. 8 hours d. 9 hours

ANS: D - 9 hours Adolescents should generally get around 9 hours of sleep each night

A nurse is observing children playing in the playroom. What describes parallel play? a. A child playing a video game b. Two children playing a card game c. Two children watching a movie on a television d. A child playing with blocks next to a child playing with trucks

ANS: D - A child playing with blocks next to a child playing with trucks Parallel play is when a toddler plays alongside, not with, other children. A child playing with blocks next to a child playing with trucks is descriptive of parallel play. The child playing a video game is descriptive of solitary play. Two children playing cards is descriptive of cooperative play. Two children watching a television is descriptive of associative play.

Which statement best describes colic? a. Periods of abdominal pain resulting in weight loss b. Usually the result of poor or inadequate mothering c. Periods of abdominal pain and crying occurring in infants older than age 6 months d. A paroxysmal abdominal pain or cramping manifested by episodes of loud crying

ANS: D - A paroxysmal abdominal pain or cramping manifested by episodes of loud crying Colic is described as paroxysmal abdominal pain or cramping that is manifested by loud crying and drawing up the legs to the abdomen. Weight loss is not part of the clinical picture. There are many theories about the cause of colic. Emotional stress or tension between the parent and child is one component. This is not consistent throughout all cases. Colic is most common in infants younger than 3 months of age.

What do the psychosocial developmental tasks of toddlerhood include? a. Development of a conscience b. Recognition of sex differences c. Ability to get along with age mates d. Ability to delay gratification

ANS: D - Ability to delay gratification If the need for basic trust has been satisfied, then toddlers can give up dependence for control, independence, and autonomy. One of the tasks that toddlers are concerned with is the ability to delay gratification. Development of a conscience and recognition of sex differences occur during the preschool years. The ability to get along with age mates develops during the preschool and school-age years.

Which characteristic best describes the fine motor skills of an infant at age 5 months? a. Neat pincer grasp b. Strong grasp reflex c. Builds a tower of two cubes d. Able to grasp object voluntarily

ANS: D - Able to grasp object voluntarily At age 5 months, the infant should be able to voluntarily grasp an object. The grasp reflex is present in the first 2 to 3 months of life. Gradually, the reflex becomes voluntary. The neat pincer grasp is not achieved until age 11 months. At age 12 months, an infant will attempt to build a tower of two cubes but will most likely be unsuccessful.

The school nurse is teaching a class on injury prevention. What should be included when discussing firearms? a. Adolescents are too young to use guns properly for hunting. b. Gun carrying among adolescents is on the rise, primarily among inner-city youth. c. Nonpowder guns (air rifles, BB guns) are a relatively safe alternative to powder guns. d. Adolescence is the peak age for being a victim or offender in the case of injury involving a firearm.

ANS: D - Adolescence is the peak age for being a victim or offender in the case of injury involving a firearm. The increase in gun availability in the general population is linked to increased gun deaths among children, especially adolescents. Gun carrying among adolescents is on the rise and not limited to the stereotypic inner-city youth. Adolescents can be taught to safely use guns for hunting, but they must be stored properly and used only with supervision. Nonpowder guns (air rifles, BB guns) cause almost as many injuries as powder guns

Cognitive development influences response to pain. What age group is most concerned with the fear of losing control during a painful experience? a. Toddlers b. Preschoolers c. School-age children d. Adolescents

ANS: D - Adolescents Adolescents view illness as physiologic (an organ malfunction) and psychophysiologic (psychologic factors that affect health). Adolescents usually approach pain with self-control. They are concerned with remaining composed and feel embarrassed and ashamed of losing control. Toddlers and preschoolers react to pain primarily as a physical, concrete experience. Preschoolers may try to escape a procedure with verbal statements such as "go away." Young school-age children may view pain as punishment for wrongdoing. This age group fears bodily harm.

Parents ask the nurse for strategies to help their toddler adjust to a new baby. What should the nurse suggest? a. Start talking about the baby very early in the pregnancy. b. Move the toddler to a new bed after the baby comes home. c. Tell the toddler that a new playmate will be coming home soon. d. Alert visitors to the new baby to include the toddler in the visit.

ANS: D - Alert visitors to the new baby to include the toddler in the visit Parents can minimize sibling rivalry by alerting visitors to the toddler's needs, having small presents on hand for the toddler, and including the child in the visits as much as possible. Time is a vague concept for toddlers. A good time to start talking about the new baby is when the toddler becomes aware of the pregnancy and the changes occurring in the home in anticipation of the new member. To avoid additional stresses when the newborn arrives, parents should perform anticipated changes, such as moving the toddler to a different room or bed, well in advance of the birth. Telling the toddler that a new playmate will come home soon sets up unrealistic expectations.

The nurse is doing a prehospitalization orientation for a girl, age 7 years, who is scheduled for cardiac surgery. As part of the preparation, the nurse explains that after the surgery, the child will be in the intensive care unit. How might the explanation by the nurse be viewed? a. Unnecessary b. The surgeon's responsibility c. Too stressful for a young child d. An appropriate part of the child's preparation

ANS: D - An appropriate part of the child's preparation The explanation is a necessary part of preoperative preparation and will help reduce the anxiety associated with surgery. If the child wakes in the intensive care unit and is not prepared for the environment, she will be even more anxious. This is a joint responsibility of nursing, medical staff, and child life personnel.

Which type of play is most typical of the preschool period? a. Team b. Parallel c. Solitary d. Associative

ANS: D - Associative Associative play is group play in similar or identical activities but without rigid organization or rules. School-age children play in teams. Parallel play is that of toddlers. Solitary play is that of infants.

The nurse is developing a teaching pamphlet for parents of school-age children. What anticipatory guidelines should the nurse include in the pamphlet? a. At age 6 years, parents should be certain that the child is reading independently with books provided by school. b. At age 8 years, parents should expect a decrease in involvement with peers and outside activities. c. At age 10 years, parents should expect a decrease in admiration of the parents with little interest in parent-child activities. d. At age 12 years, parents should be certain that the child's sex education is adequate with accurate information.

ANS: D - At age 12 years, parents should be certain that the child's sex education is adequate with accurate information A 12-year-old child should have been introduced to sex education, and parents should be certain that the information is adequate and accurate and that the child is not embarrassed to talk about sexual feelings or other aspects of sex education. At age 6 years, a child does not need to be reading independently and usually still needs help with reading and enjoys being read to. At 8 years of age, parents should expect their child to show increased involvement with peers and outside activities and should encourage this behavior. A 10-year-old child exhibits increased feelings of admiration of parents, especially fathers, and parent-child activities should be encouraged

The developmental task with which the child of 15 to 30 months is likely to be struggling is a sense of which? a. Trust b. Initiative c. Intimacy d. Autonomy

ANS: D - Autonomy Autonomy versus shame and doubt is the developmental task of toddlers. Trust versus mistrust is the developmental stage of infancy. Initiative versus guilt is the developmental stage of early childhood. Intimacy and solidarity versus isolation is the developmental stage of early adulthood

The nurse needs to assess a 15-month-old child who is sitting quietly on his father's lap. What initial action by the nurse would be most appropriate? a. Ask the father to place the child on the exam table. b. Undress the child while he is still sitting on his father's lap. c. Talk softly to the child while taking him from his father. d. Begin the assessment while the child is in his father's lap.

ANS: D - Begin the assessment while the child is in his father's lap. For young children, particularly infants and toddlers, preserving parent-child contact is a good way of decreasing stress or the need for physical restraint during an assessment. For example, much of a patient's physical examination can be done with the patient in a parent's lap with the parent providing reassuring and comforting contact. The initial action would be to begin the assessment while the child is in his father's lap.

The psychosexual conflicts of preschool children make them extremely vulnerable to which threat? a. Loss of control b. Loss of identity c. Separation anxiety d. Bodily injury and pain

ANS: D - Bodily injury and pain The psychosexual conflicts of children in this age group make them vulnerable to threats of bodily injury. Intrusive procedures, whether painful or painless, are threatening to preschoolers, whose concept of body integrity is still poorly developed. Loss of control, loss of identity, and separation anxiety are not related to psychosexual conflicts.

A young boy is found squirting lighter fluid into his mouth. His father calls the emergency department. The nurse taking the call should know that the primary danger is what? a. Hepatic dysfunction b. Dehydration secondary to vomiting c. Esophageal stricture and shock d. Bronchitis and chemical pneumonia

ANS: D - Bronchitis and chemical pneumonia Lighter fluid is a hydrocarbon. The immediate danger is aspiration. Acetaminophen overdose, not hydrocarbons, causes hepatic dysfunction. Dehydration is not the primary danger. Esophageal stricture is a late or chronic consequence of hydrocarbon ingestion.

The nurse is conducting an assessment of fine motor development in a 3-year-old child. Which is the expected drawing skill for this age? a. Can draw a complete stick figure b. Holds the instrument with the fist c. Can copy a triangle and diamond d. Can copy a circle and imitate a cross

ANS: D - Can copy a circle and imitate a cross A 3-year-old child copies a circle and imitates a cross and vertical and horizontal lines. He or she holds the writing instrument with the fingers rather than the fist. A 3-year-old is not able to draw a complete stick figure but draws a circle, later adds facial features, and by age 5 or 6 years can draw several parts (head, arms, legs, body, and facial features). Copying a triangle and diamond are mastered sometime between ages 5 and 6 years.

Where do eczematous lesions most commonly occur in an infant? a. Abdomen, cheeks, and scalp b. Buttocks, abdomen, and scalp c. Back and flexor surfaces of the arms and legs d. Cheeks and extensor surfaces of the arms and legs

ANS: D - Cheeks and extensor surfaces of the arms and legs The lesions of atopic dermatitis are generalized in infants. They are most common on the cheeks, scalp, trunk, and extensor surfaces of the extremities. The abdomen and buttocks are not common sites of lesions. The back and flexor surfaces are not usually involved.

A spinal tap must be done on a 9-year-old boy. While he is waiting in the treatment room, the nurse observes that he seems composed. When the nurse asks him if he wants his mother to stay with him, he says, "I am fine." How should the nurse interpret this situation? a. This child is unusually brave. b. He has learned that support does not help. c. Nine-year-old boys do not usually want a parent present during the procedure. d. Children in this age group often do not request support even though they need and want it.

ANS: D - Children in this age group often do not request support even though they need and want it. The school-age child's visible composure, calmness, and acceptance often mask an inner longing for support. Children of this age have a more passive approach to pain and an indirect request for support. It is especially important to be aware of nonverbal cues such as facial expression, silence, and lack of activity. Usually when someone identifies the unspoken messages, the child will readily accept support.

The nurse is explaining the preconventional stage of moral development to a group of nursing students. What characterizes this stage? a. Children in this stage focus on following the rules. b. Children in this stage live up to social expectations and roles. c. Children in this stage have a concrete sense of justice and fairness. d. Children in this stage have little, if any, concern for why something is wrong.

ANS: D - Children in this stage have little, if any, concern for why something is wrong. Young children's development of moral judgment is at the most basic level in the preconventional stage. They have little, if any, concern for why something is wrong. Following the rules, living up to social expectations, and having a concrete sense of justice and fairness are characteristics in the conventional stage.

What is characteristic of children with posttraumatic stress disorder (PTSD)? a. Denial as a defense mechanism is unusual. b. Traumatic effects cannot remain indefinitely. c. Previous coping strategies and defense mechanisms are not useful. d. Children often play out the situation over and over again.

ANS: D - Children often play out the situation over and over again. The third phase of adjustment to PTSD involves the children playing out the situation over and over to come to terms with their fears. Denial is frequently used as a defense mechanism during the second phase. For some children, traumatic effects can remain indefinitely. Coping is a learned response. During the third stage, the children can be helped to use their coping strategies to deal with their fears

What is true concerning the development of autonomy during adolescence? a. Development of autonomy typically involves rebellion. b. Development of autonomy typically involves parent-child conflicts. c. Parent and peer influences are opposing forces in the development of autonomy. d. Conformity to both parents and peers gradually declines toward the end of adolescence.

ANS: D - Conformity to both parents and peers gradually declines toward the end of adolescence. During middle and late adolescence, the conformity to parents and peers declines. Subjective feelings of self-reliance increase steadily over the adolescent years. Adolescents have genuine behavioral autonomy. Rebellion is not typically part of adolescence. It can occur in response to excessively controlling circumstances or to growing up in the absence of clear standards. Parent and peer relationships can play complementary roles in the development of a healthy degree of individual independence.

Which term refers to the relative lactase deficiency observed in preterm infants of less than 34 weeks of gestation? a. Congenital lactase deficiency b. Primary lactase deficiency c. Secondary lactase deficiency d. Developmental lactase deficiency

ANS: D - Developmental lactase deficiency Developmental lactase deficiency refers to the relative lactase deficiency observed in preterm infants of less than 34 weeks of gestation. Congenital lactase deficiency occurs soon after birth after the newborn has consumed lactose-containing milk. Primary lactase deficiency, sometimes referred to as late-onset lactase deficiency, is the most common type of lactose intolerance and is manifested usually after 4 or 5 years of age. Secondary lactase deficiency may occur secondary to damage of the intestinal lumen, which decreases or destroys the enzyme lactase.

The nurse is administering an oral antihistamine at bedtime to a child with atopic dermatitis (eczema). Which antihistamine should the nurse expect to be prescribed at bedtime? a. Cetirizine (Zyrtec) b. Loratadine (Claritin) c. Fexofenadine (Allegra) d. Diphenhydramine (Benadryl)

ANS: D - Diphenhydramine (Benadryl) Oral antihistamine drugs such as hydroxyzine or diphenhydramine usually relieve moderate or severe pruritus. Nonsedating antihistamines such as cetirizine (Zyrtec), loratadine (Claritin), or fexofenadine (Allegra) may be preferred for daytime pruritus relief. Because pruritus increases at night, a mildly sedating antihistamine such as Benadryl is prescribed

An infant, age 6 months, has six teeth. The nurse should recognize that this is what? a. Normal tooth eruption b. Delayed tooth eruption c. Unusual and dangerous d. Earlier than expected tooth eruption

ANS: D - Earlier than expected tooth eruption Six months is earlier than expected to have six teeth. At age 6 months, most infants have two teeth. Although unusual, having six teeth at 6 months is not dangerous.

The nurse suspects that a child has ingested some type of poison. What clinical manifestation would be most suggestive that the poison was a corrosive product? a. Tinnitus b. Disorientation c. Stupor, lethargy, and coma d. Edema of the lips, tongue, and pharynx

ANS: D - Edema of the lips, tongue, and pharynx Edema of the lips, tongue, and pharynx indicates a corrosive ingestion. Tinnitus is indicative of aspirin ingestion. Corrosives do not act on the central nervous system.

What do nursing interventions to promote health during middle childhood include? a. Stress the need for increased calorie intake to meet increased demands. b. Instruct parents to defer questions about sex until the child reaches adolescence. c. Advise parents that the child will need increasing amounts of rest toward the end of this period. d. Educate parents about the need for good dental hygiene because these are the years in which permanent teeth erupt.

ANS: D - Educate parents about the need for good dental hygiene because these are the years in which permanent teeth erupt. The permanent teeth erupt during the school-age years. Good dental hygiene and regular attention to dental caries are vital parts of health supervision during this period. Caloric needs are decreased in relation to body size for this age group. Balanced nutrition is essential to promote growth. Questions about sex should be addressed honestly as the child asks questions. The child usually no longer needs a nap, but most require approximately 11 hours of sleep each night at age 5 years and 9 hours at age 12 years.

During the physical examination of an adolescent with significant weight loss, what finding may indicate an eating disorder? a. Diarrhea b. Amenorrhea c. Appetite suppression d. Erosion of tooth enamel

ANS: D - Erosion of tooth enamel Some of the signs of bulimia include erosion of tooth enamel and increased dental caries. Check the back of the hands for abrasions caused by rubbing against the maxillary incisors during self-induced vomiting. Diarrhea is not a result of vomiting. Rather, it may occur in patients with inflammatory bowel disease and other gastrointestinal diseases. Amenorrhea can occur with anorexia nervosa, but it can also be a result of the weight loss from other causes. It can also indicate pregnancy in adolescent females. Appetite suppression can occur from central nervous system lesions or from oncologic and metabolic disorders.

The nurse is planning to bring a preschool child a toy from the playroom. What toy is appropriate for this age group? a. Building blocks b. A 500-piece puzzle c. Paint by number picture d. Farm animals and equipment

ANS: D - Farm animals and equipment The most characteristic and pervasive preschooler activity is imitative, imaginative, and dramatic play. Farm animals and equipment would provide hours of self-expression. Building blocks are appropriate for older infants and toddlers. A 500-piece puzzle or a paint by number picture would be appropriate for a school-age child.

What should the nurse suggest to parents of preschoolers about sensitive questions regarding sex? a. Distract your child from the topic. b. Offer complete factual information. c. Dismiss the topic until the child is older. d. Find out what your child knows or thinks.

ANS: D - Find out what your child knows or thinks. Two rules govern answering sensitive questions about topics such as sex. The first is to find out what children know and think. By investigating the theories children have produced as a reasonable explanation, parents can not only give correct information but also help children understand why their explanation is inaccurate. Another reason for ascertaining what the child thinks before offering any information is to avoid giving an "unasked for" answer. The child should not be distracted from the topic. If parents offer too much information, the child will simply become bored or end the conversation with an irrelevant question. What matters is that parents are approachable and do not dismiss their child's inquiries

According to Piaget, adolescents tend to be in what stage of cognitive development? a. Concrete operations b. Conventional thought c. Postconventional thought d. Formal operational thought

ANS: D - Formal operational thought Cognitive thinking culminates in the capacity for abstract thinking. This stage, the period of formal operations, is Piaget's fourth and last stage. Concrete operations usually occur between ages 7 and 11 years. Conventional and postconventional thought refers to Kohlberg's stages of moral development.

The nurse is assessing a 20-month-old toddler during a well-child visit and notices tooth decay. The nurse should understand that early childhood caries are caused by what? a. Allowing the child to eat citrus foods at bedtime b. A hereditary factor that cannot be prevented c. Poor fluoride supply in the drinking water d. Giving the child a bottle of juice or milk at naptime

ANS: D - Giving the child a bottle of juice or milk at naptime One cause of early childhood caries is allowing the child to go to sleep with a bottle of milk or juice; as the sweet liquid pools in the mouth, the teeth are bathed for several hours in this cariogenic environment. Eating citrus fruit at bedtime and poor fluoride supply in drinking water do not cause early childhood caries. The problem is not hereditary and can be prevented with proper education.

Parents of a preschool child tell the nurse, "Our child seems to have many imaginary fears." What suggestion should the nurse give to the parents to help their child resolve the fears? a. Ignore the fears; they will go away. b. Explain to your child the fears are not real. c. Give your child some new toys to allay the fears. d. Help your child to resolve the fears through play activities.

ANS: D - Help your child to resolve the fears through play activities. Preschoolers are able to work through many of their unresolved fears, fantasies, and anxieties through play, especially if guided with appropriate play objects (e.g., dolls or puppets) that represent family members, health professionals, and other children. The fears should not be ignored because they may escalate. Preschoolers are not cognitively prepared for explanations about the fears. They gain security and comfort from familiar objects such as toys, dolls, or photographs of family members, so new toys should not be introduced.

The parents of a 12-month-old child ask the nurse if the child can eat hot dogs as do their other children. The nurse's reply should be based on what? a. The child is too young to digest hot dogs. b. The child is too young to eat hot dogs safely. c. Hot dogs must be sliced into sections to prevent aspiration. d. Hot dogs must be cut into small, irregular pieces to prevent aspiration.

ANS: D - Hot dogs must be cut into small, irregular pieces to prevent aspiration. To eat a hot dog safely, the child should be sitting down, and the hot dog should be cut into small, irregular pieces rather than served whole or in slices. The child's digestive system is mature enough to digest hot dogs. Hot dogs are of a consistency, diameter, and shape that may cause complete obstruction of the child's airway if not cut into irregular, small pieces

A child with acetylsalicylic acid (aspirin) poisoning is being admitted to the emergency department. What early clinical manifestation does the nurse expect to assess on this child? a. Hematemesis b. Hematochezia c. Hyperglycemia d. Hyperventilation

ANS: D - Hyperventilation An early clinical manifestation of acetylsalicylic acid (aspirin) poisoning is hyperventilation. Hematemesis, hematochezia, and hyperglycemia are clinical manifestations of iron poisoning.

What is probably the most important criterion on which to base the decision to report suspected child abuse? a. Inappropriate response of child b. Inappropriate parental concern for the degree of injury c. Absence of parents for questioning about child's injuries d. Incompatibility between the history and injury observed

ANS: D - Incompatibility between the history and injury observed Conflicting stories about the "accident" are the most indicative red flags of abuse. The child or caregiver may have an inappropriate response, but this is subjective. Parents should be questioned at some point during the investigation

An infant has been diagnosed with failure to thrive (FTT) classified according to the pathophysiology of inadequate caloric intake. The nurse understands that the reason for the FTT is most likely related to what? a. Cow's milk allergy b. Congenital heart disease c. Metabolic storage disease d. Incorrect formula preparation

ANS: D - Incorrect formula preparation FTT classified according to the pathophysiology of inadequate caloric intake is related to incorrect formula preparation, neglect, food fads, excessive juice poverty, breastfeeding problems, behavioral problems affecting eating, parental restriction of caloric intake, or central nervous system problems affecting intake consumption. Cow's milk allergy would be related to the pathophysiology of inadequate absorption, congenital heart disease would be related to the pathophysiology of increased metabolism, and metabolic storage disease is related to defective utilization

The school nurse is reviewing the process of wound healing. What is the initial response at the site of injury? a. Contraction b. Maturation c. Fibroplasia d. Inflammation

ANS: D - Inflammation The initial response at the site of injury is inflammation, a vascular and cellular response that prepares the tissues for the subsequent repair process. Fibroplasia (granulation or proliferation), the second phase of healing, lasts from 5 days to 4 weeks. During contraction and maturation, the third and fourth phases of wound healing, collagen continues to be deposited and organized into layers, compressing the new blood vessels and gradually stopping blood flow across the wound.

A 9-year-old boy has an unplanned admission to the intensive care unit (ICU) after abdominal surgery. The nursing staff has completed the admission process, and his condition is beginning to stabilize. When speaking with the parents, the nurse should expect what additional stressor to be evident? a. Usual day-night routine b. Calming influence of staff c. Adequate privacy and support d. Insufficient remembering of his condition and routine

ANS: D - Insufficient remembering of his condition and routine ICUs, especially when the family is unprepared for the admission, are strange and unfamiliar. There are many pieces of unfamiliar equipment, and the sights and sounds are much different from those of a general hospital unit. Also, with the child's condition being more precarious, it may be difficult to keep the parents updated on what is happening. Lights are usually on around the clock, seriously disrupting the diurnal rhythm. In most ICUs, the staff works with a sense of urgency. It is difficult for parents to ask questions about their child when staff is with other patients. Usually little privacy is available for families in ICUs.

The nurse is helping parents achieve a more nutritionally adequate vegetarian diet for their children. Which is most likely lacking in their particular diet? a. Fat b. Protein c. Vitamins C and A d. Iron and calcium

ANS: D - Iron and calcium Deficiencies can occur when various substances in the diet interact with minerals. For example, iron, zinc, and calcium can form insoluble complexes with phytates or oxalates (substances found in plant proteins), which impair the bioavailability of the mineral. This type of interaction is important in vegetarian diets because plant foods such as soy are high in phytates. Fat and vitamins C and A are readily available from vegetable sources. Plant proteins are available

A 16-year-old adolescent boy tells the school nurse that he is gay. The nurse's response should be based on what? a. He is too young to have had enough sexual activity to determine this. b. The nurse should feel open to discussing his or her own beliefs about homosexuality. c. Homosexual adolescents do not have concerns that differ from those of heterosexual adolescents. d. It is important to provide a nonthreatening environment in which he can discuss this.

ANS: D - It is important to provide a nonthreatening environment in which he can discuss this. The nurse needs to be open and nonjudgmental in interactions with adolescents. This will provide a safe environment in which to provide appropriate health care. Adolescence is when sexual identity develops. The nurse's own beliefs should not bias the interaction with this student. Homosexual adolescents face very different challenges as they grow up because of society's response to homosexuality

What is an important consideration in the diagnosis of attention deficit hyperactivity disorder (ADHD)? a. Learning disabilities are apparent at an early age. b. The child will always be distracted by external stimuli. c. Parental observations of the child's behavior are most relevant. d. It must be determined whether the child's behavior is age appropriate or problematic.

ANS: D - It must be determined whether the child's behavior is age appropriate or problematic. The diagnosis of ADHD is complex. A multidisciplinary evaluation should be done to determine whether the child's behavior is appropriate for the developmental age or whether it is problematic. Learning disabilities are usually not evident until the child enters school. Each child with ADHD responds differently to stimuli. Some children are distracted by internal stimuli and others by external stimuli. Parents can only provide one viewpoint of the child's behavior. Many observers should be asked to provide input with structured tools to facilitate the diagnosis.

A parent needs to leave a hospitalized toddler for a short period of time. What action should the nurse suggest to the parent to ease the separation for the toddler? a. Bring a new toy when returning. b. Leave when the child is distracted. c. Tell the child when they will return. d. Leave a favorite article from home with the child.

ANS: D - Leave a favorite article from home with the child. If the parents cannot stay with the child, they should leave favorite articles from home with the child, such as a blanket, toy, bottle, feeding utensil, or article of clothing. Because young children associate such inanimate objects with significant people, they gain comfort and reassurance from these possessions. They make the association that if the parents left this, the parents will surely return. Bringing a new toy would not help with the separation. The parent should not leave when the child is distracted, and toddlers would not understand when the parent should return because time is not a concept they understand.

A 6-year-old child is admitted to the pediatric unit and requires bed rest. Having art supplies available meets which purpose? a. Allows the child to create gifts for parents b. Provides developmentally appropriate activities c. Is essential for play therapy so the child can work on past problems d. Lets the child express thoughts and feelings through pictures rather than words

ANS: D - Lets the child express thoughts and feelings through pictures rather than words The art supplies allow the child to draw images that come into the mind. This can help the child develop symbols and then verbalize reactions to illness and hospitalization. The child can make gifts and drawings for parents, but the goal is to allow expression of feelings. Although art is developmentally and situationally appropriate, the child benefits by being able to express feelings nonverbally. The art supplies are not therapeutic play but a mechanism for expressive play. The child will not work on past problems.

A child with acetaminophen (Tylenol) poisoning has been admitted to the emergency department. What antidote does the nurse anticipate being prescribed? a. Carnitine (Carnitor) b. Fomepizole (Antizol) c. Deferoxamine (Desferal) d. N-acetylcysteine (Mucomyst)

ANS: D - N-acetylcysteine (Mucomyst) The antidote for acetaminophen (Tylenol) poisoning is N-acetylcysteine (Mucomyst). Carnitine (Carnitor) is an antidote for valproic acid (Depakote), fomepizole (Antizol) is the antidote for methanol poisoning, and deferoxamine (Desferal) is the antidote for iron poisoning.

An adolescent girl asks the school nurse for advice because she has dysmenorrhea. She says that a friend recommended she try an over-the-counter nonsteroidal anti-inflammatory drug (NSAID). The nurse's response should be based on what? a. Hormone therapy is necessary for the treatment of dysmenorrhea. b. Acetaminophen is the drug of choice for the treatment of dysmenorrhea. c. Over-the-counter NSAIDs are rarely strong enough to provide adequate pain relief. d. NSAIDs are effective because they inhibit prostaglandins, leading to reduction in uterine activity.

ANS: D - NSAIDs are effective because they inhibit prostaglandins, leading to reduction in uterine activity. First-line therapy for adolescents with dysmenorrhea is NSAIDs. NSAIDs are potent anti-inflammatory agents that block the formation of prostaglandins, resulting in decreased uterine activity. Hormone therapy may be indicated if there is no physical abnormality and NSAIDs are ineffective. Acetaminophen does not have an antiprostaglandin action. It can help with pain control but will not be as effective as NSAIDs.

What strategy is considered one of the best for preventing smoking in teenagers? a. Large-scale printed information campaigns b. Emphasis on the long-term effects of smoking on health c. Threatening the social norms of groups most likely to smoke d. Peer-led programs emphasizing the social consequences of smoking

ANS: D - Peer-led programs emphasizing the social consequences of smoking Peer-led programs emphasizing the social consequences of smoking have proved most successful. Short-term effects such as an unpleasant odor and stains on the teeth and hands are stressed. If a significant number of peers convince their classmates that smoking is not popular, others will follow. Large-scale printed information campaigns are not effective. A specified curriculum and teaching can increase benefit. Long-term effects do not dissuade adolescents because they do not have a future perspective. Threatening the norms of the social group is one of the least effective means of prevention.

An adolescent girl calls the nurse at the clinic because she had unprotected sex the night before and does not want to be pregnant. What should the nurse explain? a. It is too late to prevent an unwanted pregnancy. b. An abortion may be the best option if she is pregnant. c. The risk of pregnancy is minimal, so no action is necessary. d. Postcoital contraception is available to prevent implantation and therefore pregnancy.

ANS: D - Postcoital contraception is available to prevent implantation and therefore pregnancy. Several emergency methods of contraception (ECP) are available and appropriate for use after unprotected sexual intercourse. A progestin-only ECP (levonorgestrel [Plan B]) is approved by the U.S. Food and Drug Administration and has high effectiveness and low rates of side effects. Plan B is effective if given within 72 hours of unprotected intercourse. An abortion is not indicated. Although the risk of pregnancy depends on the time during her menstrual cycle, a low risk of pregnancy exists. ECP is indicated

The nurse is preparing a pamphlet for parents of adolescents about guidance during the adolescent years. What suggestion should the nurse include in the pamphlet? a. Provide criticism when mistakes are made or when views are different. b. Use comparisons with older siblings or extended family to promote good outcomes. c. Begin to disengage from school functions to allow the adolescent to gain independence. d. Provide clear, reasonable limits and define consequences when rules are broken.

ANS: D - Provide clear, reasonable limits and define consequences when rules are broken. An anticipatory guideline to include when teaching parents of adolescents is to provide clear, reasonable limits and have clear consequences when rules are broken. Parents should avoid criticism when mistakes are made and should allow opportunities for the teen to voice different views and opinions. Parents should try to avoid comparing the teen with a sibling or extended family member. Parents should try to be more engaged in the teen's school functions to show support and unconditional love.

Many adolescents use alcohol for self-medication. How does an adolescent view the benefit of alcohol? a. Believes it has a stimulant effect b. Believes it increases alertness c. Provides a sense of euphoria d. Provides a defense against depression

ANS: D - Provides a defense against depression Adolescents who abuse alcohol often rely on it as a defense against depression, anxiety, fear, and anger. Alcohol is a depressant and has a sedative effect. Alcohol does not provide a sense of euphoria. It does reduce inhibitions against aggressive behaviors.

What is the role of the peer group in the life of school-age children? a. Decreases their need to learn appropriate sex roles b. Gives them an opportunity to learn dominance and hostility c. Allows them to remain dependent on their parents for a longer time d. Provides them with security as they gain independence from their parents

ANS: D - Provides them with security as they gain independence from their parents Peer group identification is an important factor in gaining independence from parents. Through peer relationships, children learn ways to deal with dominance and hostility. They also learn how to relate to people in positions of leadership and authority and how to explore ideas and the physical environment. A child's concept of appropriate sex roles is influenced by relationships with peers.

The nurse is assisting the family of a child with a history of encopresis. What should be included in the nurse's discussion with the family? a. Instruct the parents to sit the child on the toilet at twice-daily routine intervals. b. Instruct the parents that the child will probably need to have daily enemas. c. Suggest the use of stimulant cathartics weekly. d. Reassure the family that most problems are resolved successfully, with some relapses during periods of stress.

ANS: D - Reassure the family that most problems are resolved successfully, with some relapses during periods of stress. Children may be unaware of a prior sensation and be unable to control the urge after it begins. They may be so accustomed to bowel accidents that they may be unable to smell or feel them. Family counseling is directed toward reassurance that most problems resolve successfully, although relapses during periods of stress are possible. Sitting the child on the toilet is not recommended because it may intensify the parent-child conflict. Enemas may be needed for impactions, but long-term use prevents the child from assuming responsibility for defecation. Stimulant cathartics may cause cramping that can frighten children.

A parent asks the nurse about negativism in toddlers. What is the most appropriate recommendation? a. Punish the child. b. Provide more attention. c. Ask child not to always say "no." d. Reduce the opportunities for a "no" answer.

ANS: D - Reduce the opportunities for a "no" answer. The nurse should suggest to the parent that questions should be phrased with realistic choices rather than yes or no answers. This provides a sense of control for the toddler and reduces the opportunity for negativism. Negativism is not an indication of stubbornness or insolence and should not be punished. The negativism is not a function of attention; the child is testing limits to gain an understanding of the world. The toddler is too young to comply with requests not to say "no."

A toddler's parent asks the nurse for suggestions on dealing with temper tantrums. What is the most appropriate recommendation? a. Punish the child. b. Explain to child that this is wrong. c. Leave the child alone until the tantrum is over. d. Remain close by the child but without eye contact.

ANS: D - Remain close by the child but without eye contact. The best way to deal with temper tantrums is to ignore the behaviors, provided that the actions are not dangerous to the child. Tantrums are common during this age group as the child becomes more independent and overwhelmed by increasingly complex tasks. The parents and caregivers need to have consistent and developmentally appropriate expectations. Punishment and explanations will not be beneficial. The presence of the parent is necessary both for safety and to provide a feeling of control and security to the child when the tantrum is over.

A father tells the nurse that his daughter wants the same plate and cup used at every meal, even if they go to a restaurant. The nurse should explain that this is what? a. A sign the child is spoiled b. An attempt to exert unhealthy control c. Regression, which is common at this age d. Ritualism, an expected behavior at this age

ANS: D - Ritualism, an expected behavior at this age The child is exhibiting the ritualism, which is characteristic at this age. Ritualism is the need to maintain sameness and reliability. It provides a sense of structure and comfort to the toddler. It will dictate certain principles in feeding practices, including rejecting a favorite food because it is served in a different container. This does not indicate the child has unreasonable expectations but rather is part of normal development. Ritualism is not regression, which is a retreat from a present pattern of functioning.

What is the most common type of burn in the toddler age group? a. Electric burn from electrical outlets b. Flame burn from playing with matches c. Hot object burn from cigarettes or irons d. Scald burn from high-temperature tap water

ANS: D - Scald burn from high-temperature tap water Scald burns are the most common type of thermal injury in children, especially 1- and 2-year-old children. Temperature should be reduced on the hot water in the house and hot liquids placed out of the child's reach. Electric burns from electrical outlets and hot object burns from cigarettes or irons are both significant causes of burn injury. The child should be protected by reducing the temperature on the hot water heater in the home, keeping objects such as cigarettes and irons away from children, and placing protective guards over electrical outlets when not in use. Flame burns from matches and lighters represent one of the most fatal types of burns in the toddler age group but not one of the most common types of burn

Because of their striving for independence and productivity, which age group of children is particularly vulnerable to events that may lessen their feeling of control and power? a. Infants b. Toddlers c. Preschoolers d. School-age children

ANS: D - School-age children When a child is hospitalized, the altered family role, physical disability, loss of peer acceptance, lack of productivity, and inability to cope with stress usurp individual power and identity. This is especially detrimental to school-age children, who are striving for independence and productivity and are now experiencing events that decrease their control and power. Infants, toddlers, and preschoolers, although affected by loss of power, are not as significantly affected as school-age children.

The nurse is instructing student nurses about the stress of hospitalization for children from middle infancy throughout the preschool years. What major stress should the nurse relate to the students? a. Pain b. Bodily injury c. Loss of control d. Separation anxiety

ANS: D - Separation anxiety The major stress from middle infancy throughout the preschool years, especially for children ages 6 to 30 months, is separation anxiety.

What do inflicted immersion burns often appear as? a. Partial-thickness, asymmetrical burns b. Splash pattern burns on hands or feet c. Any splash burn with dry linear marks d. Sharply demarcated, symmetrical burns

ANS: D - Sharply demarcated, symmetrical burns Immersion burns are sharply demarcated symmetrical burns. Asymmetrical burns and splash burns are often accidental

The nurse is assessing the Tanner stage in an adolescent male. The nurse recognizes that the stages are based on what? a. Hair growth on the face and chest b. Changes in the voice to a deeper timbre c. Muscle growth in the arms, legs, and shoulders d. Size and shape of the penis and scrotum and distribution of pubic hair

ANS: D - Size and shape of the penis and scrotum and distribution of pubic hair In males, the Tanner stages describe pubertal development based on the size and shape of the penis and scrotum and the shape and distribution of pubic hair. During puberty, hair begins to grow on the face and chest; the voice becomes deeper; and muscles grow in the arms, legs, and shoulders, but these are not used for the Tanner stages.

The school nurse is discussing after-school sports participation with parents of children age 10 years. The nurse's presentation includes which important consideration? a. Teams should be gender specific. b. Organized sports are not appropriate at this age. c. Competition is detrimental to the establishment of a positive self-image. d. Sports participation is encouraged if the type of sport is appropriate to the child's abilities.

ANS: D - Sports participation is encouraged if the type of sport is appropriate to the child's abilities. Virtually every child is suited for some type of sport. The child should be matched to the type of sport appropriate to his or her abilities and physical and emotional makeup. At this age, girls and boys have the same basic structure and similar responses to exercise and training. After puberty, teams should be gender specific because of the increased muscle mass in boys. Organized sports help children learn teamwork and skill acquisition. The emphasis should be on playing and learning. Children do enjoy appropriate levels of competition.

The nurse is interviewing the parents of a 4-month-old boy brought to the hospital emergency department. The infant is dead, and no attempt at resuscitation is made. The parents state that the baby was found in his crib with a blanket over his head, lying face down in bloody fluid from his nose and mouth. The nurse might initially suspect his death was caused by what? a. Suffocation b. Child abuse c. Infantile apnea d. Sudden infant death syndrome (SIDS)

ANS: D - Sudden infant death syndrome (SIDS) The description of how the child was found in the crib is suggestive of SIDS. The nurse is careful to tell the parents that a diagnosis cannot be confirmed until an autopsy is performed.

What characteristic best describes the language skills of a 3-year-old child? a. Asks meanings of words b. Follows directional commands c. Can describe an object according to its composition d. Talks incessantly regardless of whether anyone is listening

ANS: D - Talks incessantly regardless of whether anyone is listening Because of the dramatic vocabulary increase at this age, 3-year-old children are known to talk incessantly regardless of whether anyone is listening. A 4- to 5-year-old child asks lots of questions and can follow simple directional commands. A 6-year-old child can describe an object according to its composition.

During the preschool period, the emphasis of injury prevention should be placed on what? a. Limitation of physical activities b. Punishment for unsafe behaviors c. Constant vigilance and protection d. Teaching about safety and potential hazards

ANS: D - Teaching about safety and potential hazards Education about safety and potential hazards is appropriate for preschoolers because they can begin to understand dangers. Limitation of physical activities is not appropriate. Punishment may make children scared of trying new things. Constant vigilance and protection are not practical at this age because preschoolers are becoming more independent.

What statement characterizes moral development in the older school-age child? a. Rule violations are viewed in an isolated context. b. Judgments and rules become more absolute and authoritarian. c. The child remembers the rules but cannot understand the reasons behind them. d. The child is able to judge an act by the intentions that prompted it rather than just by the consequences.

ANS: D - The child is able to judge an act by the intentions that prompted it rather than just by the consequences. Older school-age children are able to judge an act by the intentions that prompted the behavior rather than just by the consequences. Rule violation is likely to be viewed in relation to the total context in which it appears. Rules and judgments become less absolute and authoritarian. The situation and the morality of the rule itself influence reactions.

An older school-age child asks the nurse, "What is the reason for this topical corticosteroid cream?" What rationale should the nurse give? a. The cream is used for an antifungal effect. b. The cream is used for an analgesic effect. c. The cream is used for an antibacterial effect. d. The cream is used for an anti-inflammatory effect.

ANS: D - The cream is used for an anti-inflammatory effect. The glucocorticoids are the therapeutic agents used most widely for skin disorders. Their local anti-inflammatory effects are merely palliative, so the medication must be applied until the disease state undergoes a remission or the causative agent is eliminated. It does not have an antifungal, analgesic, or antibacterial effect.

When only one child is abused in a family, the abuse is usually a result of what? a. The child is the firstborn. b. The child is the same gender as the abusing parent. c. The parent abuses the child to avoid showing favoritism. d. The parent is unable to deal with the child's behavioral style.

ANS: D - The parent is unable to deal with the child's behavioral style. The child unintentionally contributes to the abuse. The "fit" or compatibility between the child's temperament and the parent's ability to deal with that behavior style is an important predictor. Birth order and gender can contribute to abuse, but there is not a specific birth order or gender relationship that is indicative of abuse. Being the firstborn or the same gender as the abuser is not linked to child abuse. Avoidance of favoritism is not usually a cause of abuse

Adolescents often do not use reasoned decision making when issues such as substance abuse and sexual behavior are involved. What is this because of? a. They tend to be immature. b. They do not need to use reasoned decision making. c. They lack cognitive skills to use reasoned decision making. d. They are dealing with issues that are stressful and emotionally laden.

ANS: D - They are dealing with issues that are stressful and emotionally laden. In the face of time pressures, personal stress, or overwhelming peer pressure, young people are more likely to abandon rational thought processes. Many of the health-related decisions adolescents confront are emotionally laden or new. Under such conditions, many people do not use their capacity for formal decision making. The majority of adolescents have cognitive skills and are capable of reasoned decision making. Stress affects their ability to process information. Reasoned decision making should be used in issues that are crucial such as substance abuse and sexual behavior.

What best describes central nervous system (CNS) stimulants? a. Acute intoxication can lead to coma. b. They produce strong physical dependence. c. Withdrawal symptoms are life threatening. d. They can result in strong psychologic dependence.

ANS: D - They can result in strong psychologic dependence. CNS stimulants such as amphetamines and cocaine produce a strong psychologic dependence. Acute intoxication leads to violent aggressive behavior or psychotic episodes characterized by paranoia, uncontrollable agitation, and restlessness. This class of drugs does not produce strong physical dependence and can be withdrawn without much danger.

During a well-child visit, the nurse plots the child's BMI on the health record. What is the purpose of the BMI? a. To determine medication dosages b. To predict adult height and weight c. To identify coping strategies used by the child d. To provide a consistent measure of obesity

ANS: D - To provide a consistent measure of obesity A consistent measure of the degree of obesity is important to determine whether modification of the body fat component is indicated. Body surface area (BSA), not BMI, is used for medication dosage calculation. The BMI is not a predictor of adult height. A child with a high BMI may use food as a coping mechanism, but the BMI is not correlated with coping strategy use.

What factor is most important in predisposing toddlers to frequent infections? a. Respirations are abdominal. b. Pulse and respiratory rates in toddlers are slower than those in infants. c. Defense mechanisms are less efficient than those during infancy. d. Toddlers have short, straight internal ear canals and large lymph tissue.

ANS: D - Toddlers have short, straight internal ear canals and large lymph tissue. Toddlers continue to have the short, straight internal ear canals of infants. The lymphoid tissue of the tonsils and adenoids continues to be relatively large. These two anatomic conditions combine to predispose toddlers to frequent infections. The abdominal respirations and lowered pulse and respiratory rate of toddlers do not affect their susceptibility to infection. The defense mechanisms are more efficient compared with those of infancy.

What factor is most likely to increase the likelihood that an adolescent will misuse alcohol? a. Female gender b. Regular school attendance c. Rural environment d. Unconventional behavior

ANS: D - Unconventional behavior Adolescents who are connected and engage in conventional behavior are less likely to misuse alcohol. Those who are disconnected from school, family, and other social supports have fewer assets and are more likely to abuse alcohol. School attendance is a sign of connectedness. Girls and boys report a similar onset and course of experimentation with alcohol. Urban youths have a higher likelihood of alcohol abuse than rural adolescents.

Rickets is caused by a deficiency in what? a. Vitamin A b. Vitamin C c. Folic acid and iron d. Vitamin D and calcium

ANS: D - Vitamin D and calcium Fat-soluble vitamin D and calcium are necessary in adequate amounts to prevent rickets. No correlation exists between rickets and folic acid, iron, or vitamins A and C.

Which characteristic best describes the gross motor skills of a 24-month-old child? a. Skips b. Broad jumps c. Rides tricycle d. Walks up and down stairs

ANS: D - Walks up and down stairs A 24-month-old child can go up and down stairs alone with two feet on each step. Skipping and broad jumping are skills acquired at age 3 years. Tricycle riding is achieved at age 4 years

Match the following terms related to food sensitivities to the accurate descriptions. Allergy with a hereditary tendency

Atopy

Match each neurologic reflex that appears in infancy to its description. Turning the hips and shoulders to one side causes all the other body parts to follow

Body righting

Match the following terms related to food sensitivities to the accurate descriptions. Specific components of food or ingredients in food that are recognized by allergen-specific immune cells eliciting an immune reaction

Food allergen

Match the following terms related to food sensitivities to the accurate descriptions. An adverse health effect arising from a specific immune response that occurs reproducibly on exposure to a given food

Food allergy

Match the following terms related to food sensitivities to the accurate descriptions. A food elicits a reproducible adverse reaction but does not have an established immunologic mechanism

Food intolerance

Match each neurologic reflex that appears in infancy to its description. An infant in the prone or supine position is able to raise his or her head.

Labyrinth righting

Match each neurologic reflex that appears in infancy to its description. When the infant is suspended in a horizontal prone position, the head is raised and the legs and spine are extended.

Landau

Match each neurologic reflex that appears in infancy to its description When the body of an erect infant is tilted, the head is returned to an upright, erect position

Otolith righting

Match each neurologic reflex that appears in infancy to its description. When the infant is suspended in a horizontal prone position and suddenly thrust downward, the hands and fingers extend forward as if to protect against falling.

Parachute

Match the following terms related to food sensitivities to the accurate descriptions. Initial exposure to an allergen resulting in an immune response; subsequent exposure induces a much stronger response

Sensitization


Conjuntos de estudio relacionados

Seeley's Anatomy & Physiology 11th ed Chapter 7

View Set

Art Appreciation Ch.2: Developing Visual Literacy

View Set

ALL US History Regents 2012-2015

View Set

NPLEX II - Botanical Mnemonics for me

View Set

SOCI1000 Chapter 10 Practice Questions

View Set

OB Chapt 19 Nursing Management of Pregnancy at Risk

View Set

Unit 6 - The Sales Contract (Questions)

View Set

Earth Science, Chapter 6 - Earthquakes

View Set

Marketing Exam #2 Melissa Moore Mississippi State University

View Set